18.07.2014 Views

Topics in Inequalities - Theorems and Techniques Hojoo ... - Index of

Topics in Inequalities - Theorems and Techniques Hojoo ... - Index of

Topics in Inequalities - Theorems and Techniques Hojoo ... - Index of

SHOW MORE
SHOW LESS

You also want an ePaper? Increase the reach of your titles

YUMPU automatically turns print PDFs into web optimized ePapers that Google loves.

<strong>Topics</strong> <strong>in</strong> <strong>Inequalities</strong> - <strong>Theorems</strong> <strong>and</strong> <strong>Techniques</strong><br />

<strong>Hojoo</strong> Lee<br />

Introduction<br />

<strong>Inequalities</strong> are useful <strong>in</strong> all fields <strong>of</strong> Mathematics. The aim <strong>of</strong> this problem-oriented book is to present<br />

elementary techniques <strong>in</strong> the theory <strong>of</strong> <strong>in</strong>equalities. The readers will meet classical theorems <strong>in</strong>clud<strong>in</strong>g<br />

Schur’s <strong>in</strong>equality, Muirhead’s theorem, the Cauchy-Schwarz <strong>in</strong>equality, the Power Mean <strong>in</strong>equality, the AM-<br />

GM <strong>in</strong>equality, <strong>and</strong> Hölder’s theorem. I would greatly appreciate hear<strong>in</strong>g about comments <strong>and</strong> corrections<br />

from my readers. You can send email to me at ultrametric@gmail.com<br />

To Students<br />

My target readers are challeng<strong>in</strong>g high schools students <strong>and</strong> undergraduate students. The given techniques<br />

<strong>in</strong> this book are just the tip <strong>of</strong> the <strong>in</strong>equalities iceberg. Young students should f<strong>in</strong>d their own methods to<br />

attack various problems. A great Hungarian Mathematician Paul Erdös was fond <strong>of</strong> say<strong>in</strong>g that God has<br />

a transf<strong>in</strong>ite book with all the theorems <strong>and</strong> their best pro<strong>of</strong>s. I strongly encourage readers to send me their<br />

own creative solutions <strong>of</strong> the problems <strong>in</strong> this book. Have fun!<br />

Acknowledgement<br />

I’m <strong>in</strong>debted to Orl<strong>and</strong>o Döhr<strong>in</strong>g <strong>and</strong> Darij Gr<strong>in</strong>berg for provid<strong>in</strong>g me with TeX files <strong>in</strong>clud<strong>in</strong>g collections<br />

<strong>of</strong> <strong>in</strong>terest<strong>in</strong>g <strong>in</strong>equalities. I’d like to thank Marian Muresan for his excellent collection <strong>of</strong> problems.<br />

I’m also pleased that Cao M<strong>in</strong>h Quang sent me various vietnam problems <strong>and</strong> nice pro<strong>of</strong>s <strong>of</strong> Nesbitt’s<br />

<strong>in</strong>equality. I owe great debts to Stanley Rab<strong>in</strong>owitz who k<strong>in</strong>dly sent me his paper On The Computer<br />

Solution <strong>of</strong> Symmetric Homogeneous Triangle <strong>Inequalities</strong>.<br />

Resources on the Web<br />

1. MathL<strong>in</strong>ks, http://www.mathl<strong>in</strong>ks.ro<br />

2. Art <strong>of</strong> Problem Solv<strong>in</strong>g, http://www.art<strong>of</strong>problemsolv<strong>in</strong>g.com<br />

3. MathPro Press, http://www.mathpropress.com<br />

4. K. S. Kedlaya, A < B, http://www.unl.edu/amc/a-activities/a4-for-students/s-<strong>in</strong>dex.html<br />

5. T. J. Mildorf, Olympiad <strong>Inequalities</strong>, http://web.mit.edu/tmildorf/www<br />

I


Contents<br />

1 Geometric <strong>Inequalities</strong> 1<br />

1.1 Ravi Substitution . . . . . . . . . . . . . . . . . . . . . . . . . . . . . . . . . . . . . . . . . . . 1<br />

1.2 Trigonometric Methods . . . . . . . . . . . . . . . . . . . . . . . . . . . . . . . . . . . . . . . 7<br />

1.3 Applications <strong>of</strong> Complex Numbers . . . . . . . . . . . . . . . . . . . . . . . . . . . . . . . . . 11<br />

2 Four Basic <strong>Techniques</strong> 12<br />

2.1 Trigonometric Substitutions . . . . . . . . . . . . . . . . . . . . . . . . . . . . . . . . . . . . 12<br />

2.2 Algebraic Substitutions . . . . . . . . . . . . . . . . . . . . . . . . . . . . . . . . . . . . . . . 15<br />

2.3 Increas<strong>in</strong>g Function Theorem . . . . . . . . . . . . . . . . . . . . . . . . . . . . . . . . . . . . 20<br />

2.4 Establish<strong>in</strong>g New Bounds . . . . . . . . . . . . . . . . . . . . . . . . . . . . . . . . . . . . . . 22<br />

3 Homogenizations <strong>and</strong> Normalizations 26<br />

3.1 Homogenizations . . . . . . . . . . . . . . . . . . . . . . . . . . . . . . . . . . . . . . . . . . . 26<br />

3.2 Schur’s Inequality <strong>and</strong> Muirhead’s Theorem . . . . . . . . . . . . . . . . . . . . . . . . . . . . 29<br />

3.3 Normalizations . . . . . . . . . . . . . . . . . . . . . . . . . . . . . . . . . . . . . . . . . . . . 34<br />

3.4 Cauchy-Schwarz Inequality <strong>and</strong> Hölder’s Inequality . . . . . . . . . . . . . . . . . . . . . . . . 38<br />

4 Convexity 42<br />

4.1 Jensen’s Inequality . . . . . . . . . . . . . . . . . . . . . . . . . . . . . . . . . . . . . . . . . . 42<br />

4.2 Power Means . . . . . . . . . . . . . . . . . . . . . . . . . . . . . . . . . . . . . . . . . . . . . 45<br />

4.3 Majorization Inequality . . . . . . . . . . . . . . . . . . . . . . . . . . . . . . . . . . . . . . . 47<br />

4.4 Support<strong>in</strong>g L<strong>in</strong>e Inequality . . . . . . . . . . . . . . . . . . . . . . . . . . . . . . . . . . . . . 48<br />

5 Problems, Problems, Problems 50<br />

5.1 Multivariable <strong>Inequalities</strong> . . . . . . . . . . . . . . . . . . . . . . . . . . . . . . . . . . . . . . 50<br />

5.2 Problems for Putnam Sem<strong>in</strong>ar . . . . . . . . . . . . . . . . . . . . . . . . . . . . . . . . . . . 58<br />

II


Chapter 1<br />

Geometric <strong>Inequalities</strong><br />

It gives me the same pleasure when someone else proves a good theorem as when I do it myself.<br />

E. L<strong>and</strong>au<br />

1.1 Ravi Substitution<br />

Many <strong>in</strong>equalities are simplified by some suitable substitutions. We beg<strong>in</strong> with a classical <strong>in</strong>equality <strong>in</strong><br />

triangle geometry. What is the first 1 nontrivial geometric <strong>in</strong>equality ? In 1746, Chapple showed that<br />

Theorem 1.1.1. (Chapple 1746, Euler 1765) Let R <strong>and</strong> r denote the radii <strong>of</strong> the circumcircle <strong>and</strong><br />

<strong>in</strong>circle <strong>of</strong> the triangle ABC. Then, we have R ≥ 2r <strong>and</strong> the equality holds if <strong>and</strong> only if ABC is equilateral.<br />

Pro<strong>of</strong>. Let BC = a, CA = b, AB = c, s = a+b+c<br />

2<br />

<strong>and</strong> S = [ABC]. 2 Recall the well-known identities :<br />

S = abc<br />

4R , S = rs, S2 = s(s − a)(s − b)(s − c). Hence, R ≥ 2r is equivalent to abc<br />

4S ≥ 2 S S2<br />

s<br />

or abc ≥ 8<br />

s<br />

or<br />

abc ≥ 8(s − a)(s − b)(s − c). We need to prove the follow<strong>in</strong>g.<br />

Theorem 1.1.2. ([AP], A. Padoa) Let a, b, c be the lengths <strong>of</strong> a triangle. Then, we have<br />

abc ≥ 8(s − a)(s − b)(s − c) or abc ≥ (b + c − a)(c + a − b)(a + b − c)<br />

<strong>and</strong> the equality holds if <strong>and</strong> only if a = b = c.<br />

Pro<strong>of</strong>. We use the Ravi Substitution : S<strong>in</strong>ce a, b, c are the lengths <strong>of</strong> a triangle, there are positive reals x,<br />

y, z such that a = y + z, b = z + x, c = x + y. (Why?) Then, the <strong>in</strong>equality is (y + z)(z + x)(x + y) ≥ 8xyz<br />

for x, y, z > 0. However, we get (y + z)(z + x)(x + y) − 8xyz = x(y − z) 2 + y(z − x) 2 + z(x − y) 2 ≥ 0.<br />

Exercise 1. Let ABC be a right triangle. Show that R ≥ (1 + √ 2)r. When does the equality hold ?<br />

It’s natural to ask that the <strong>in</strong>equality <strong>in</strong> the theorem 2 holds for arbitrary positive reals a, b, c? Yes ! It’s<br />

possible to prove the <strong>in</strong>equality without the additional condition that a, b, c are the lengths <strong>of</strong> a triangle :<br />

Theorem 1.1.3. Let x, y, z > 0. Then, we have xyz ≥ (y + z − x)(z + x − y)(x + y − z). The equality<br />

holds if <strong>and</strong> only if x = y = z.<br />

Pro<strong>of</strong>. S<strong>in</strong>ce the <strong>in</strong>equality is symmetric <strong>in</strong> the variables, without loss <strong>of</strong> generality, we may assume that<br />

x ≥ y ≥ z. Then, we have x + y > z <strong>and</strong> z + x > y. If y + z > x, then x, y, z are the lengths <strong>of</strong> the sides<br />

<strong>of</strong> a triangle. In this case, by the theorem 2, we get the result. Now, we may assume that y + z ≤ x. Then,<br />

xyz > 0 ≥ (y + z − x)(z + x − y)(x + y − z).<br />

The <strong>in</strong>equality <strong>in</strong> the theorem 2 holds when some <strong>of</strong> x, y, z are zeros :<br />

Theorem 1.1.4. Let x, y, z ≥ 0. Then, we have xyz ≥ (y + z − x)(z + x − y)(x + y − z).<br />

1 The first geometric <strong>in</strong>equality is the Triangle Inequality : AB + BC ≥ AC<br />

2 In this book, [P ] st<strong>and</strong>s for the area <strong>of</strong> the polygon P .<br />

1


Pro<strong>of</strong>. S<strong>in</strong>ce x, y, z ≥ 0, we can f<strong>in</strong>d positive sequences {x n }, {y n }, {z n } for which<br />

Apply<strong>in</strong>g the theorem 2 yields<br />

lim x n = x,<br />

n→∞<br />

lim y n = y, lim z n = z.<br />

n→∞ n→∞<br />

x n y n z n ≥ (y n + z n − x n )(z n + x n − y n )(x n + y n − z n ).<br />

Now, tak<strong>in</strong>g the limits to both sides, we get the result.<br />

Clearly, the equality holds when x = y = z. However, xyz = (y+z −x)(z +x−y)(x+y−z) <strong>and</strong> x, y, z ≥ 0<br />

does not guarantee that x = y = z. In fact, for x, y, z ≥ 0, the equality xyz = (y +z −x)(z +x−y)(x+y −z)<br />

is equivalent to<br />

x = y = z or x = y, z = 0 or y = z, x = 0 or z = x, y = 0.<br />

It’s straightforward to verify the equality<br />

xyz − (y + z − x)(z + x − y)(x + y − z) = x(x − y)(x − z) + y(y − z)(y − x) + z(z − x)(z − y).<br />

Hence, the theorem 4 is a particular case <strong>of</strong> Schur’s <strong>in</strong>equality.<br />

Problem 1. (IMO 2000/2, Proposed by Titu Andreescu) Let a, b, c be positive numbers such that<br />

abc = 1. Prove that (<br />

a − 1 + 1 ) (<br />

b − 1 + 1 ) (<br />

c − 1 + 1 )<br />

≤ 1.<br />

b<br />

c<br />

a<br />

First Solution. S<strong>in</strong>ce abc = 1, we make the substitution a = x y , b = y z , c = z x<br />

for x, y, z > 0.3 We rewrite<br />

the given <strong>in</strong>equality <strong>in</strong> the terms <strong>of</strong> x, y, z :<br />

( x<br />

y − 1 + z ) (y<br />

y z − 1 + x ) ( z<br />

z x x)<br />

− 1 + y ≤ 1 ⇔ xyz ≥ (y + z − x)(z + x − y)(x + y − z).<br />

The Ravi Substitution is useful for <strong>in</strong>equalities for the lengths a, b, c <strong>of</strong> a triangle. After the Ravi<br />

Substitution, we can remove the condition that they are the lengths <strong>of</strong> the sides <strong>of</strong> a triangle.<br />

Problem 2. (IMO 1983/6) Let a, b, c be the lengths <strong>of</strong> the sides <strong>of</strong> a triangle. Prove that<br />

a 2 b(a − b) + b 2 c(b − c) + c 2 a(c − a) ≥ 0.<br />

First Solution. After sett<strong>in</strong>g a = y + z, b = z + x, c = x + y for x, y, z > 0, it becomes<br />

x 3 z + y 3 x + z 3 y ≥ x 2 yz + xy 2 z + xyz 2<br />

or x2<br />

y + y2<br />

z + z2<br />

x ≥ x + y + z,<br />

which follows from the Cauchy-Schwarz <strong>in</strong>equality<br />

( )<br />

x<br />

2<br />

(y + z + x)<br />

y + y2<br />

z + z2<br />

≥ (x + y + z) 2 .<br />

x<br />

Exercise 2. Let a, b, c be the lengths <strong>of</strong> a triangle. Show that<br />

a<br />

b + c +<br />

b<br />

c + a +<br />

c<br />

a + b < 2.<br />

3 For example, take x = 1, y = 1 a , z = 1 ab . 2


Exercise 3. (Darij Gr<strong>in</strong>berg) Let a, b, c be the lengths <strong>of</strong> a triangle. Show the <strong>in</strong>equalities<br />

a 3 + b 3 + c 3 + 3abc − 2b 2 a − 2c 2 b − 2a 2 c ≥ 0,<br />

<strong>and</strong><br />

3a 2 b + 3b 2 c + 3c 2 a − 3abc − 2b 2 a − 2c 2 b − 2a 2 c ≥ 0.<br />

We now discuss Weitzenböck’s <strong>in</strong>equality <strong>and</strong> related <strong>in</strong>equalities.<br />

Problem 3. (IMO 1961/2, Weitzenböck’s <strong>in</strong>equality) Let a, b, c be the lengths <strong>of</strong> a triangle with area<br />

S. Show that<br />

a 2 + b 2 + c 2 ≥ 4 √ 3S.<br />

Solution. Write a = y + z, b = z + x, c = x + y for x, y, z > 0. It’s equivalent to<br />

which can be obta<strong>in</strong>ed as follow<strong>in</strong>g :<br />

((y + z) 2 + (z + x) 2 + (x + y) 2 ) 2 ≥ 48(x + y + z)xyz,<br />

((y + z) 2 + (z + x) 2 + (x + y) 2 ) 2 ≥ 16(yz + zx + xy) 2 ≥ 16 · 3(xy · yz + yz · zx + xy · yz).<br />

Here, we used the well-known <strong>in</strong>equalities p 2 + q 2 ≥ 2pq <strong>and</strong> (p + q + r) 2 ≥ 3(pq + qr + rp).<br />

Theorem 1.1.5. (Hadwiger-F<strong>in</strong>sler <strong>in</strong>equality) For any triangle ABC with sides a, b, c <strong>and</strong> area F ,<br />

the follow<strong>in</strong>g <strong>in</strong>equality holds.<br />

2ab + 2bc + 2ca − (a 2 + b 2 + c 2 ) ≥ 4 √ 3F.<br />

First Pro<strong>of</strong>. After the substitution a = y + z, b = z + x, c = x + y, where x, y, z > 0, it becomes<br />

which follows from the identity<br />

xy + yz + zx ≥ √ 3xyz(x + y + z),<br />

(xy + yz + zx) 2 − 3xyz(x + y + z) = (xy − yz)2 + (yz − zx) 2 + (zx − xy) 2<br />

.<br />

2<br />

Second Pro<strong>of</strong>. We give a convexity pro<strong>of</strong>. There are many ways to deduce the follow<strong>in</strong>g identity:<br />

2ab + 2bc + 2ca − (a 2 + b 2 + c 2 )<br />

4F<br />

S<strong>in</strong>ce tan x is convex on ( 0, π 2<br />

)<br />

, Jensen’s <strong>in</strong>equality shows that<br />

2ab + 2bc + 2ca − (a 2 + b 2 + c 2 )<br />

4F<br />

= tan A 2 + tan B 2 + tan C 2 .<br />

≥ 3 tan<br />

(<br />

A<br />

2 + B 2 + C 2<br />

3<br />

)<br />

= √ 3.<br />

Ts<strong>in</strong>tsifas proved a simultaneous generalization <strong>of</strong> Weitzenböck’s <strong>in</strong>equality <strong>and</strong> Nesbitt’s <strong>in</strong>equality.<br />

Theorem 1.1.6. (Ts<strong>in</strong>tsifas) Let p, q, r be positive real numbers <strong>and</strong> let a, b, c denote the sides <strong>of</strong> a triangle<br />

with area F . Then, we have<br />

p<br />

q + r a2 +<br />

q<br />

r + p b2 +<br />

r<br />

p + q c2 ≥ 2 √ 3F.<br />

3


Pro<strong>of</strong>. (V. Pambuccian) By Hadwiger-F<strong>in</strong>sler <strong>in</strong>equality, it suffices to show that<br />

p<br />

q + r a2 +<br />

q<br />

r + p b2 +<br />

r<br />

p + q c2 ≥ 1 2 (a + b + c)2 − (a 2 + b 2 + c 2 )<br />

or<br />

( ) ( ) ( )<br />

p + q + r p + q + r p + q + r<br />

a 2 +<br />

b 2 +<br />

c 2 ≥ 1 (a + b + c)2<br />

q + r<br />

r + p<br />

p + q 2<br />

or<br />

( 1<br />

((q + r) + (r + p) + (p + q))<br />

q + r a2 + 1<br />

r + p b2 + 1 )<br />

p + q c2 ≥ (a + b + c) 2 .<br />

However, this is a straightforward consequence <strong>of</strong> the Cauchy-Schwarz <strong>in</strong>equality.<br />

Theorem 1.1.7. (Neuberg-Pedoe <strong>in</strong>equality) Let a 1 , b 1 , c 1 denote the sides <strong>of</strong> the triangle A 1 B 1 C 1 with<br />

area F 1 . Let a 2 , b 2 , c 2 denote the sides <strong>of</strong> the triangle A 2 B 2 C 2 with area F 2 . Then, we have<br />

a 1 2 (b 2 2 + c 2 2 − a 2 2 ) + b 1 2 (c 2 2 + a 2 2 − b 2 2 ) + c 1 2 (a 2 2 + b 2 2 − c 2 2 ) ≥ 16F 1 F 2 .<br />

Notice that it’s a generalization <strong>of</strong> Weitzenböck’s <strong>in</strong>equality.(Why?) In [GC], G. Chang proved Neuberg-<br />

Pedoe <strong>in</strong>equality by us<strong>in</strong>g complex numbers. For very <strong>in</strong>terest<strong>in</strong>g geometric observations <strong>and</strong> pro<strong>of</strong>s <strong>of</strong><br />

Neuberg-Pedoe <strong>in</strong>equality, see [DP] or [GI, pp.92-93]. Here, we <strong>of</strong>fer three algebraic pro<strong>of</strong>s.<br />

Lemma 1.1.1.<br />

a 1 2 (a 2 2 + b 2 2 − c 2 2 ) + b 1 2 (b 2 2 + c 2 2 − a 2 2 ) + c 1 2 (c 2 2 + a 2 2 − b 2 2 ) > 0.<br />

Pro<strong>of</strong>. Observe that it’s equivalent to<br />

(a 1 2 + b 1 2 + c 1 2 )(a 2 2 + b 2 2 + c 2 2 ) > 2(a 1 2 a 2 2 + b 1 2 b 2 2 + c 1 2 c 2 2 ).<br />

From Heron’s formula, we f<strong>in</strong>d that, for i = 1, 2,<br />

16F i 2 = (a i 2 + b i 2 + c i 2 ) 2 − 2(a i 4 + b i 4 + c i 4 ) > 0 or a i 2 + b i 2 + c i 2 ><br />

√<br />

2(a i4 + b i 4 + c i4 ) .<br />

The Cauchy-Schwarz <strong>in</strong>equality implies that<br />

√<br />

(a 2 1 + b 2 1 + c 2 1 )(a 2 2 + b 2 2 + c 2 2 ) > 2 (a 14 + b 4 1 + c 14 )(a 24 + b 4 2 + c 24 ) ≥ 2(a 2 1 a 2 2 + b 2 1 b 2 2 + c 2 1 c 2 2 ).<br />

First Pro<strong>of</strong>. ([LC1], Carlitz) By the lemma, we obta<strong>in</strong><br />

Hence, we need to show that<br />

L = a 1 2 (b 2 2 + c 2 2 − a 2 2 ) + b 1 2 (c 2 2 + a 2 2 − b 2 2 ) + c 1 2 (a 2 2 + b 2 2 − c 2 2 ) > 0,<br />

One may easily check the follow<strong>in</strong>g identity<br />

where<br />

Us<strong>in</strong>g the identity<br />

L 2 − (16F 1 2 )(16F 2 2 ) ≥ 0.<br />

L 2 − (16F 1 2 )(16F 2 2 ) = −4(UV + V W + W U),<br />

U = b 1 2 c 2 2 − b 2 2 c 1 2 , V = c 1 2 a 2 2 − c 2 2 a 1 2 <strong>and</strong> W = a 1 2 b 2 2 − a 2 2 b 1 2 .<br />

a 2 1 U + b 2 1 V + c 2 1 W = 0 or W = − a 1 2<br />

c<br />

2 U − b 1 2<br />

1 c<br />

2 V,<br />

1<br />

one may also deduce that<br />

UV + V W + W U = − a 1 2 (U − c 1 2 − a 2 2<br />

)2<br />

1 − b 1<br />

V − 4a 1 2 b 2 1 − (c 2 1 − a 2 1 − b 2 1 ) 2<br />

V 2 .<br />

c<br />

2 1 2a<br />

2 1 4a 12 c<br />

2 1<br />

It follows that<br />

UV + V W + W U = − a 1 2 (U − c 1 2 − a 2 2<br />

)2<br />

1 − b 1<br />

V − 16F 1 2<br />

c<br />

2 1 2a<br />

2 1 4a 12 c<br />

2 V 2 ≤ 0.<br />

1<br />

4


Carlitz also observed that the Neuberg-Pedoe <strong>in</strong>equality can be deduced from Aczél’s <strong>in</strong>equality.<br />

Theorem 1.1.8. (Aczél’s <strong>in</strong>equality) Let a 1 , · · · , a n , b 1 , · · · , b n be positive real numbers satisfy<strong>in</strong>g<br />

Then, the follow<strong>in</strong>g <strong>in</strong>equality holds.<br />

a 1 b 1 − (a 2 b 2 + · · · + a n b n ) ≥<br />

a 1 2 ≥ a 2 2 + · · · + a n 2 <strong>and</strong> b 1 2 ≥ b 2 2 + · · · + b n 2 .<br />

√<br />

(a 12 − (a 22 + · · · + a n2 )) ( b 1 2 − ( b 2 2 + · · · + b n<br />

2 ))<br />

Pro<strong>of</strong>. ([AI]) The Cauchy-Schwarz <strong>in</strong>equality shows that<br />

√<br />

a 1 b 1 ≥ (a 22 + · · · + a n2 )(b 2 2 + · · · + b 2 n ) ≥ a 2 b 2 + · · · + a n b n .<br />

Then, the above <strong>in</strong>equality is equivalent to<br />

(a 1 b 1 − (a 2 b 2 + · · · + a n b n )) 2 ≥ ( a 1 2 − ( a 2 2 + · · · + a n<br />

2 )) ( b 1 2 − ( b 2 2 + · · · + b n<br />

2 )) .<br />

In case a 2 1 − (a 2 2 + · · · + a 2 n ) = 0, it’s trivial. Hence, we now assume that a 2 1 − (a 2 2 + · · · + a 2 n ) > 0. The<br />

ma<strong>in</strong> trick is to th<strong>in</strong>k <strong>of</strong> the follow<strong>in</strong>g quadratic polynomial<br />

(<br />

) (<br />

) (<br />

)<br />

n∑<br />

n∑<br />

n∑<br />

n∑<br />

P (x) = (a 1 x − b 1 ) 2 − (a i x − b i ) 2 = a 2 2<br />

1 − a i x 2 + 2 a 1 b 1 − a i b i x + b 2 2<br />

1 − b i .<br />

i=2<br />

i=2<br />

S<strong>in</strong>ce P ( b 1<br />

a 1<br />

) = − ∑ ( ( ) ) 2<br />

n<br />

i=2<br />

a<br />

b1<br />

i a 1<br />

− b i ≤ 0 <strong>and</strong> s<strong>in</strong>ce the coefficient <strong>of</strong> x 2 <strong>in</strong> the quadratic polynomial P is<br />

positive, P should have at least one real root. Therefore, P has nonnegative discrim<strong>in</strong>ant. It follows that<br />

(<br />

2<br />

(<br />

a 1 b 1 −<br />

)) 2 (<br />

n∑<br />

a i b i − 4 a 2 1 −<br />

i=2<br />

n∑<br />

i=2<br />

a i<br />

2<br />

) (<br />

b 1 2 −<br />

i=2<br />

n∑<br />

i=2<br />

b i<br />

2<br />

)<br />

≥ 0.<br />

i=2<br />

Second Pro<strong>of</strong> <strong>of</strong> Neuberg-Pedoe <strong>in</strong>equality. ([LC2], Carlitz) We rewrite it <strong>in</strong> terms <strong>of</strong> a 1 , b 1 , c 1 , a 2 , b 2 , c 2 :<br />

(a 1 2 + b 1 2 + c 1 2 )(a 2 2 + b 2 2 + c 2 2 ) − 2(a 1 2 a 2 2 + b 1 2 b 2 2 + c 1 2 c 2 2 )<br />

√ ((a12<br />

≥ + b 2 1 + c ) (<br />

2 (a22<br />

2 1 − 2(a14 + b 4 1 + c 14 ))<br />

+ b 2 2 + c ) )<br />

2<br />

2 2 − 2(a24 + b 4 2 + c 24 ) .<br />

We employ the follow<strong>in</strong>g substitutions<br />

As <strong>in</strong> the pro<strong>of</strong> <strong>of</strong> the lemma 5, we have<br />

x 1 = a 1 2 + b 1 2 + c 1 2 , x 2 = √ 2 a 1 2 , x 3 = √ 2 b 1 2 , x 4 = √ 2 c 1 2 ,<br />

y 1 = a 2 2 + b 2 2 + c 2 2 , y 2 = √ 2 a 2 2 , y 3 = √ 2 b 2 2 , y 4 = √ 2 c 2 2 .<br />

We now apply Aczél’s <strong>in</strong>equality to get the <strong>in</strong>equality<br />

x 1 2 > x 2 2 + y 3 2 + x 4 2 <strong>and</strong> y 1 2 > y 2 2 + y 3 2 + y 4 2 .<br />

x 1 y 1 − x 2 y 2 − x 3 y 3 − x 4 y 4 ≥ √ (x 12 − (x 22 + y 32 + x 42 )) (y 12 − (y 22 + y 32 + y 42 )).<br />

We close this section with a very simple pro<strong>of</strong> by a former student <strong>in</strong> KMO 4 summer program.<br />

4 Korean Mathematical Olympiads<br />

5


Third Pro<strong>of</strong>. Toss two triangles △A 1 B 1 C 1 <strong>and</strong> △A 2 B 2 C 2 on R 2 :<br />

A 1 (0, p 1 ), B 1 (p 2 , 0), C 1 (p 3 , 0), A 2 (0, q 1 ), B 2 (q 2 , 0), <strong>and</strong> C 2 (q 3 , 0).<br />

It therefore follows from the <strong>in</strong>equality x 2 + y 2 ≥ 2|xy| that<br />

a 1 2 (b 2 2 + c 2 2 − a 2 2 ) + b 1 2 (c 2 2 + a 2 2 − b 2 2 ) + c 1 2 (a 2 2 + b 2 2 − c 2 2 )<br />

= (p 3 − p 2 ) 2 (2q 1 2 + 2q 1 q 2 ) + (p 1 2 + p 3 2 )(2q 2 2 − 2q 2 q 3 ) + (p 1 2 + p 2 2 )(2q 3 2 − 2q 2 q 3 )<br />

= 2(p 3 − p 2 ) 2 q 1 2 + 2(q 3 − q 2 ) 2 p 1 2 + 2(p 3 q 2 − p 2 q 3 ) 2<br />

≥ 2((p 3 − p 2 )q 1 ) 2 + 2((q 3 − q 2 )p 1 ) 2<br />

≥ 4|(p 3 − p 2 )q 1 | · |(q 3 − q 2 )p 1 |<br />

= 16F 1 F 2 .<br />

6


1.2 Trigonometric Methods<br />

In this section, we employ trigonometric methods to attack geometric <strong>in</strong>equalities.<br />

Theorem 1.2.1. (Erdös-Mordell Theorem) If from a po<strong>in</strong>t P <strong>in</strong>side a given triangle ABC perpendiculars<br />

P H 1 , P H 2 , P H 3 are drawn to its sides, then P A + P B + P C ≥ 2(P H 1 + P H 2 + P H 3 ).<br />

This was conjectured by Paul Erdös <strong>in</strong> 1935, <strong>and</strong> first proved by Mordell <strong>in</strong> the same year. Several pro<strong>of</strong>s<br />

<strong>of</strong> this <strong>in</strong>equality have been given, us<strong>in</strong>g Ptolemy’s theorem by André Avez, angular computations with<br />

similar triangles by Leon Bank<strong>of</strong>f, area <strong>in</strong>equality by V. Komornik, or us<strong>in</strong>g trigonometry by Mordell <strong>and</strong><br />

Barrow.<br />

Pro<strong>of</strong>. ([MB], Mordell) We transform it to a trigonometric <strong>in</strong>equality. Let h 1 = P H 1 , h 2 = P H 2 <strong>and</strong><br />

h 3 = P H 3 . Apply the S<strong>in</strong>ce Law <strong>and</strong> the Cos<strong>in</strong>e Law to obta<strong>in</strong><br />

√<br />

P A s<strong>in</strong> A = H 2 H 3 = h 2 2 + h 2 3 − 2h 2 h 3 cos(π − A),<br />

√<br />

P B s<strong>in</strong> B = H 3 H 1 = h 2 3 + h 2 1 − 2h 3 h 1 cos(π − B),<br />

√<br />

P C s<strong>in</strong> C = H 1 H 2 = h 2 1 + h 2 2 − 2h 1 h 2 cos(π − C).<br />

So, we need to prove that<br />

∑<br />

cyclic<br />

√<br />

1<br />

h 2 2 + h 2 3 − 2h 2 h 3 cos(π − A) ≥ 2(h 1 + h 2 + h 3 ).<br />

s<strong>in</strong> A<br />

The ma<strong>in</strong> trouble is that the left h<strong>and</strong> side has too heavy terms with square root expressions. Our strategy<br />

is to f<strong>in</strong>d a lower bound without square roots. To this end, we express the terms <strong>in</strong>side the square root as<br />

the sum <strong>of</strong> two squares.<br />

H 2 H 3<br />

2<br />

= h 2 2 + h 3 2 − 2h 2 h 3 cos(π − A)<br />

= h 2 2 + h 3 2 − 2h 2 h 3 cos(B + C)<br />

= h 2 2 + h 3 2 − 2h 2 h 3 (cos B cos C − s<strong>in</strong> B s<strong>in</strong> C).<br />

Us<strong>in</strong>g cos 2 B + s<strong>in</strong> 2 B = 1 <strong>and</strong> cos 2 C + s<strong>in</strong> 2 C = 1, one f<strong>in</strong>ds that<br />

H 2 H 3<br />

2<br />

= (h2 s<strong>in</strong> C + h 3 s<strong>in</strong> B) 2 + (h 2 cos C − h 3 cos B) 2 .<br />

S<strong>in</strong>ce (h 2 cos C − h 3 cos B) 2 is clearly nonnegative, we get H 2 H 3 ≥ h 2 s<strong>in</strong> C + h 3 s<strong>in</strong> B. It follows that<br />

√<br />

∑ h 2 2 + h 2 3 − 2h 2 h 3 cos(π − A)<br />

≥<br />

∑ h 2 s<strong>in</strong> C + h 3 s<strong>in</strong> B<br />

s<strong>in</strong> A<br />

s<strong>in</strong> A<br />

cyclic<br />

cyclic<br />

= ∑ ( s<strong>in</strong> B<br />

s<strong>in</strong> C + s<strong>in</strong> C )<br />

h 1<br />

s<strong>in</strong> B<br />

cyclic<br />

≥<br />

∑ √<br />

s<strong>in</strong> B<br />

2<br />

s<strong>in</strong> C · s<strong>in</strong> C<br />

s<strong>in</strong> B h 1<br />

cyclic<br />

= 2h 1 + 2h 2 + 2h 3 .<br />

We use the same techniques to attack the follow<strong>in</strong>g geometric <strong>in</strong>equality.<br />

Problem 4. (IMO Short-list 2005) In an acute triangle ABC, let D, E, F , P , Q, R be the feet <strong>of</strong><br />

perpendiculars from A, B, C, A, B, C to BC, CA, AB, EF , F D, DE, respectively. Prove that<br />

where p(T ) denotes the perimeter <strong>of</strong> triangle T .<br />

p(ABC)p(P QR) ≥ p(DEF ) 2 ,<br />

7


Solution. Let’s euler 5 this problem. Let ρ be the circumradius <strong>of</strong> the triangle ABC. It’s easy to show that<br />

BC = 2ρ s<strong>in</strong> A <strong>and</strong> EF = 2ρ s<strong>in</strong> A cos A. S<strong>in</strong>ce DQ = 2ρ s<strong>in</strong> C cos B cos A, DR = 2ρ s<strong>in</strong> B cos C cos A, <strong>and</strong><br />

∠F DE = π − 2A, the Cos<strong>in</strong>e Law gives us<br />

or<br />

where<br />

QR 2 = DQ 2 + DR 2 − 2DQ · DR cos(π − 2A)<br />

[<br />

]<br />

= 4ρ 2 cos 2 A (s<strong>in</strong> C cos B) 2 + (s<strong>in</strong> B cos C) 2 + 2 s<strong>in</strong> C cos B s<strong>in</strong> B cos C cos(2A)<br />

QR = 2ρ cos A √ f(A, B, C),<br />

f(A, B, C) = (s<strong>in</strong> C cos B) 2 + (s<strong>in</strong> B cos C) 2 + 2 s<strong>in</strong> C cos B s<strong>in</strong> B cos C cos(2A).<br />

So, what we need to attack is the follow<strong>in</strong>g <strong>in</strong>equality:<br />

⎛<br />

⎞ ⎛<br />

⎞ ⎛<br />

⎞<br />

⎝ ∑<br />

2ρ s<strong>in</strong> A⎠<br />

⎝ ∑<br />

2ρ cos A √ f(A, B, C) ⎠ ≥ ⎝ ∑<br />

2ρ s<strong>in</strong> A cos A⎠<br />

or<br />

cyclic<br />

cyclic<br />

cyclic<br />

cyclic<br />

cyclic<br />

⎛ ⎞ ⎛<br />

⎞ ⎛<br />

⎞<br />

⎝ ∑<br />

s<strong>in</strong> A⎠<br />

⎝ ∑<br />

cos A √ f(A, B, C) ⎠ ≥ ⎝ ∑<br />

2<br />

s<strong>in</strong> A cos A⎠<br />

.<br />

Our job is now to f<strong>in</strong>d a reasonable lower bound <strong>of</strong> √ f(A, B, C). Once aga<strong>in</strong>, we express f(A, B, C) as the<br />

sum <strong>of</strong> two squares. We observe that<br />

cyclic<br />

f(A, B, C) = (s<strong>in</strong> C cos B) 2 + (s<strong>in</strong> B cos C) 2 + 2 s<strong>in</strong> C cos B s<strong>in</strong> B cos C cos(2A)<br />

= (s<strong>in</strong> C cos B + s<strong>in</strong> B cos C) 2 + 2 s<strong>in</strong> C cos B s<strong>in</strong> B cos C [−1 + cos(2A)]<br />

= s<strong>in</strong> 2 (C + B) − 2 s<strong>in</strong> C cos B s<strong>in</strong> B cos C · 2 s<strong>in</strong> 2 A<br />

= s<strong>in</strong> 2 A [1 − 4 s<strong>in</strong> B s<strong>in</strong> C cos B cos C] .<br />

So, we shall express 1 − 4 s<strong>in</strong> B s<strong>in</strong> C cos B cos C as the sum <strong>of</strong> two squares. The trick is to replace 1 with<br />

(<br />

s<strong>in</strong> 2 B + cos 2 B ) ( s<strong>in</strong> 2 C + cos 2 C ) . Indeed, we get<br />

2<br />

1 − 4 s<strong>in</strong> B s<strong>in</strong> C cos B cos C = ( s<strong>in</strong> 2 B + cos 2 B ) ( s<strong>in</strong> 2 C + cos 2 C ) − 4 s<strong>in</strong> B s<strong>in</strong> C cos B cos C<br />

= (s<strong>in</strong> B cos C − s<strong>in</strong> C cos B) 2 + (cos B cos C − s<strong>in</strong> B s<strong>in</strong> C) 2<br />

= s<strong>in</strong> 2 (B − C) + cos 2 (B + C)<br />

= s<strong>in</strong> 2 (B − C) + cos 2 A.<br />

It therefore follows that<br />

f(A, B, C) = s<strong>in</strong> 2 A [ s<strong>in</strong> 2 (B − C) + cos 2 A ] ≥ s<strong>in</strong> 2 A cos 2 A<br />

so that<br />

∑<br />

cos A √ f(A, B, C) ≥ ∑<br />

s<strong>in</strong> A cos 2 A.<br />

cyclic<br />

cyclic<br />

So, we can complete the pro<strong>of</strong> if we establish that<br />

⎛ ⎞ ⎛<br />

⎞ ⎛<br />

⎞<br />

⎝ ∑<br />

s<strong>in</strong> A⎠<br />

⎝ ∑<br />

s<strong>in</strong> A cos 2 A⎠ ≥ ⎝ ∑<br />

2<br />

s<strong>in</strong> A cos A⎠<br />

.<br />

cyclic<br />

cyclic<br />

cyclic<br />

Indeed, one sees that it’s a direct consequence <strong>of</strong> the Cauchy-Schwarz <strong>in</strong>equality<br />

(p + q + r)(x + y + z) ≥ ( √ px + √ qy + √ rz) 2 ,<br />

where p, q, r, x, y <strong>and</strong> z are positive real numbers.<br />

5 euler v. (<strong>in</strong> Mathematics) transform the problems <strong>in</strong> triangle geometry to trigonometric ones<br />

8


Alternatively, one may obta<strong>in</strong> another lower bound <strong>of</strong> f(A, B, C):<br />

f(A, B, C) = (s<strong>in</strong> C cos B) 2 + (s<strong>in</strong> B cos C) 2 + 2 s<strong>in</strong> C cos B s<strong>in</strong> B cos C cos(2A)<br />

= (s<strong>in</strong> C cos B − s<strong>in</strong> B cos C) 2 + 2 s<strong>in</strong> C cos B s<strong>in</strong> B cos C [1 + cos(2A)]<br />

= s<strong>in</strong> 2 (B − C) + 2 s<strong>in</strong>(2B) · s<strong>in</strong>(2C) · 2 cos 2 A<br />

2 2<br />

≥ cos 2 A s<strong>in</strong>(2B) s<strong>in</strong>(2C).<br />

Then, we can use this to <strong>of</strong>fer a lower bound <strong>of</strong> the perimeter <strong>of</strong> triangle P QR:<br />

p(P QR) = ∑<br />

2ρ cos A √ f(A, B, C) ≥ ∑<br />

2ρ cos 2 A √ s<strong>in</strong> 2B s<strong>in</strong> 2C<br />

cyclic<br />

cyclic<br />

So, one may consider the follow<strong>in</strong>g <strong>in</strong>equality:<br />

p(ABC) ∑<br />

2ρ cos 2 A √ s<strong>in</strong> 2B s<strong>in</strong> 2C ≥ p(DEF ) 2<br />

or<br />

or<br />

cyclic<br />

cyclic<br />

cyclic<br />

⎛<br />

⎞ ⎛<br />

⎞ ⎛<br />

⎞<br />

⎝2ρ ∑<br />

s<strong>in</strong> A⎠<br />

⎝ ∑<br />

2ρ cos 2 A √ s<strong>in</strong> 2B s<strong>in</strong> 2C⎠ ≥ ⎝2ρ ∑<br />

2<br />

s<strong>in</strong> A cos A⎠<br />

.<br />

cyclic<br />

cyclic<br />

cyclic<br />

cyclic<br />

⎛ ⎞ ⎛<br />

⎞ ⎛<br />

⎞<br />

⎝ ∑<br />

s<strong>in</strong> A⎠<br />

⎝ ∑<br />

cos 2 A √ s<strong>in</strong> 2B s<strong>in</strong> 2C⎠ ≥ ⎝ ∑<br />

2<br />

s<strong>in</strong> A cos A⎠<br />

.<br />

However, it turned out that this doesn’t hold. Try to disprove this!<br />

Problem 5. (IMO 2001/1) Let ABC be an acute-angled triangle with O as its circumcenter. Let P on l<strong>in</strong>e<br />

BC be the foot <strong>of</strong> the altitude from A. Assume that ∠BCA ≥ ∠ABC + 30 ◦ . Prove that ∠CAB + ∠COP <<br />

90 ◦ .<br />

Pro<strong>of</strong>. The angle <strong>in</strong>equality ∠CAB + ∠COP < 90 ◦ can be written as ∠COP < ∠P CO. This can be shown<br />

if we establish the length <strong>in</strong>equality OP > P C. S<strong>in</strong>ce the power <strong>of</strong> P with respect to the circumcircle <strong>of</strong> ABC<br />

is OP 2 = R 2 − BP · P C, where R is the circumradius <strong>of</strong> the triangle ABC, it becomes R 2 − BP · P C > P C 2<br />

or R 2 > BC · P C. We euler this. It’s an easy job to get BC = 2R s<strong>in</strong> A <strong>and</strong> P C = 2R s<strong>in</strong> B cos C. Hence,<br />

we show the <strong>in</strong>equality R 2 > 2R s<strong>in</strong> A · 2R s<strong>in</strong> B cos C or s<strong>in</strong> A s<strong>in</strong> B cos C < 1 4<br />

. S<strong>in</strong>ce s<strong>in</strong> A < 1, it suffices<br />

to show that s<strong>in</strong> A s<strong>in</strong> B cos C < 1 4 . F<strong>in</strong>ally, we use the angle condition ∠C ≥ ∠B + 30◦ to obta<strong>in</strong> the<br />

trigonometric <strong>in</strong>equality<br />

s<strong>in</strong> B cos C =<br />

s<strong>in</strong>(B + C) − s<strong>in</strong>(C − B)<br />

2<br />

≤<br />

1 − s<strong>in</strong>(C − B)<br />

2<br />

≤<br />

1 − s<strong>in</strong> 30◦<br />

2<br />

= 1 4 .<br />

We close this section with Barrows’ <strong>in</strong>equality stronger than Erdös-Mordell Theorem.<br />

follow<strong>in</strong>g trigonometric <strong>in</strong>equality:<br />

We need the<br />

Proposition 1.2.1. Let x, y, z, θ 1 , θ 2 , θ 3 be real numbers with θ 1 + θ 2 + θ 3 = π. Then,<br />

x 2 + y 2 + z 2 ≥ 2(yz cos θ 1 + zx cos θ 2 + xy cos θ 3 ).<br />

Pro<strong>of</strong>. Us<strong>in</strong>g θ 3 = π − (θ 1 + θ 2 ), it’s an easy job to check the follow<strong>in</strong>g identity<br />

x 2 + y 2 + z 2 − 2(yz cos θ 1 + zx cos θ 2 + xy cos θ 3 ) = (z − (x cos θ 2 + y cos θ 1 )) 2 + (x s<strong>in</strong> θ 2 − y s<strong>in</strong> θ 1 ) 2 .<br />

9


Corollary 1.2.1. Let p, q, <strong>and</strong> r be positive real numbers. Let θ 1 , θ 2 , <strong>and</strong> θ 3 be real numbers satisfy<strong>in</strong>g<br />

θ 1 + θ 2 + θ 3 = π. Then, the follow<strong>in</strong>g <strong>in</strong>equality holds.<br />

p cos θ 1 + q cos θ 2 + r cos θ 3 ≤ 1 ( qr<br />

2 p + rp<br />

q + pq )<br />

.<br />

r<br />

(√ √<br />

Pro<strong>of</strong>. Take (x, y, z) = qr<br />

p , rp<br />

q , √ )<br />

pq<br />

r<br />

<strong>and</strong> apply the above proposition.<br />

Theorem 1.2.2. (Barrow’s Inequality) Let P be an <strong>in</strong>terior po<strong>in</strong>t <strong>of</strong> a triangle ABC <strong>and</strong> let U, V , W<br />

be the po<strong>in</strong>ts where the bisectors <strong>of</strong> angles BP C, CP A, AP B cut the sides BC,CA,AB respectively. Prove<br />

that P A + P B + P C ≥ 2(P U + P V + P W ).<br />

Pro<strong>of</strong>. ([MB] <strong>and</strong> [AK]) Let d 1 = P A, d 2 = P B, d 3 = P C, l 1 = P U, l 2 = P V , l 3 = P W , 2θ 1 = ∠BP C,<br />

2θ 2 = ∠CP A, <strong>and</strong> 2θ 3 = ∠AP B. We need to show that d 1 + d 2 + d 3 ≥ 2(l 1 + l 2 + l 3 ). It’s easy to deduce<br />

the follow<strong>in</strong>g identities<br />

l 1 = 2d 2d 3<br />

d 2 + d 3<br />

cos θ 1 , l 2 = 2d 3d 1<br />

d 3 + d 1<br />

cos θ 2 , <strong>and</strong> l 3 = 2d 1d 2<br />

d 1 + d 2<br />

cos θ 3 ,<br />

By the AM-GM <strong>in</strong>equality <strong>and</strong> the above corollary, this means that<br />

l 1 + l 2 + l 3 ≤ √ d 2 d 3 cos θ 1 + √ d 3 d 1 cos θ 2 + √ d 1 d 2 cos θ 3 ≤ 1 2 (d 1 + d 2 + d 3 ) .<br />

As another application <strong>of</strong> the above trigonometric proposition, we establish the follow<strong>in</strong>g <strong>in</strong>equality<br />

Corollary 1.2.2. ([AK], Abi-Khuzam) Let x 1 , · · · , x 4 be positive real numbers. Let θ 1 , · · · , θ 4 be real<br />

numbers such that θ 1 + · · · + θ 4 = π. Then,<br />

√<br />

(x 1 x 2 + x 3 x 4 )(x 1 x 3 + x 2 x 4 )(x 1 x 4 + x 2 x 3 )<br />

x 1 cos θ 1 + x 2 cos θ 2 + x 3 cos θ 3 + x 4 cos θ 4 ≤<br />

.<br />

x 1 x 2 x 3 x 4<br />

√<br />

Pro<strong>of</strong>. Let p = x12 2<br />

+x 2<br />

2x 1 x 2<br />

+ x32 2<br />

+x 4<br />

2x 3 x 4<br />

q = x1x2+x3x4<br />

2<br />

<strong>and</strong> λ = p<br />

q . In the view <strong>of</strong> θ 1 + θ 2 + (θ 3 + θ 4 ) = π <strong>and</strong><br />

θ 3 + θ 4 + (θ 1 + θ 2 ) = π, the proposition implies that<br />

x 1 cos θ 1 + x 2 cos θ 2 + λ cos(θ 3 + θ 4 ) ≤ pλ = √ pq,<br />

<strong>and</strong><br />

x 3 cos θ 3 + x 4 cos θ 4 + λ cos(θ 1 + θ 2 ) ≤ q λ = √ pq.<br />

S<strong>in</strong>ce cos(θ 3 + θ 4 ) + cos(θ 1 + θ 2 ) = 0, add<strong>in</strong>g these two above <strong>in</strong>equalities yields<br />

√<br />

x 1 cos θ 1 + x 2 cos θ 2 + x 3 cos θ 3 + x 4 cos θ 4 ≤ 2 √ (x 1 x 2 + x 3 x 4 )(x 1 x 3 + x 2 x 4 )(x 1 x 4 + x 2 x 3 )<br />

pq =<br />

.<br />

x 1 x 2 x 3 x 4<br />

10


1.3 Applications <strong>of</strong> Complex Numbers<br />

In this section, we discuss some applications <strong>of</strong> complex numbers to geometric <strong>in</strong>equality. Every complex<br />

number corresponds to a unique po<strong>in</strong>t <strong>in</strong> the complex plane. The st<strong>and</strong>ard symbol for the set <strong>of</strong> all complex<br />

numbers is C, <strong>and</strong> we also refer to the complex plane as C. The ma<strong>in</strong> tool is applications <strong>of</strong> the follow<strong>in</strong>g<br />

fundamental <strong>in</strong>equality.<br />

Theorem 1.3.1. If z 1 , · · · , z n ∈ C, then |z 1 | + · · · + |z n | ≥ |z 1 + · · · + z n |.<br />

Pro<strong>of</strong>. Use <strong>in</strong>duction on n with the triangle <strong>in</strong>equality.<br />

Theorem 1.3.2. (Ptolemy’s Inequality) For any po<strong>in</strong>ts A, B, C, D <strong>in</strong> the plane, we have<br />

AB · CD + BC · DA ≥ AC · BD.<br />

Pro<strong>of</strong>. Let a, b, c <strong>and</strong> 0 be complex numbers that correspond to A, B, C, D <strong>in</strong> the complex plane. It becomes<br />

|a − b| · |c| + |b − c| · |a| ≥ |a − c| · |b|.<br />

Apply<strong>in</strong>g the Triangle Inequality to the identity (a − b)c + (b − c)a = (a − c)b, we get the result.<br />

Problem 6. ([TD]) Let P be an arbitrary po<strong>in</strong>t <strong>in</strong> the plane <strong>of</strong> a triangle ABC with the centroid G. Show<br />

the follow<strong>in</strong>g <strong>in</strong>equalities<br />

(1) BC · P B · P C + AB · P A · P B + CA · P C · P A ≥ BC · CA · AB <strong>and</strong><br />

(2) P A 3 · BC + P B 3 · CA + P C 3 · AB ≥ 3P G · BC · CA · AB.<br />

Solution. We only check the first <strong>in</strong>equality. Regard A, B, C, P as complex numbers <strong>and</strong> assume that P<br />

corresponds to 0. We’re required to prove that<br />

|(B − C)BC| + |(A − B)AB| + |(C − A)CA| ≥ |(B − C)(C − A)(A − B)|.<br />

It rema<strong>in</strong>s to apply the Triangle Inequality to the identity<br />

(B − C)BC + (A − B)AB + (C − A)CA = −(B − C)(C − A)(A − B).<br />

Problem 7. (IMO Short-list 2002) Let ABC be a triangle for which there exists an <strong>in</strong>terior po<strong>in</strong>t F<br />

such that ∠AF B = ∠BF C = ∠CF A. Let the l<strong>in</strong>es BF <strong>and</strong> CF meet the sides AC <strong>and</strong> AB at D <strong>and</strong> E,<br />

respectively. Prove that AB + AC ≥ 4DE.<br />

Solution. Let AF = x, BF = y, CF = z <strong>and</strong> let ω = cos 2π 3<br />

+ i s<strong>in</strong> 2π 3<br />

. We can toss the pictures on C so that<br />

the po<strong>in</strong>ts F , A, B, C, D, <strong>and</strong> E are represented by the complex numbers 0, x, yω, zω 2 , d, <strong>and</strong> e. It’s an<br />

easy exercise to establish that DF =<br />

xz<br />

x+z<br />

<strong>and</strong> EF =<br />

xy<br />

xz<br />

xy<br />

x+y<br />

. This means that d = −<br />

x+z<br />

ω <strong>and</strong> e = −<br />

x+y ω.<br />

We’re now required to prove that<br />

|x − yω| + |zω 2 − x| ≥ 4<br />

−zx<br />

∣z + x ω + xy ∣ ∣∣∣<br />

x + y ω2 .<br />

S<strong>in</strong>ce |ω| = 1 <strong>and</strong> ω 3 = 1, we have |zω 2 − x| = |ω(zω 2 − x)| = |z − xω|. Therefore, we need to prove<br />

|x − yω| + |z − xω| ≥<br />

4zx<br />

∣z + x − 4xy ∣ ∣∣∣<br />

x + y ω .<br />

∣ More strongly, we establish that |(x − yω) + (z − xω)| ≥ ∣ 4zx<br />

z+x − 4xy ∣∣<br />

x+y ω or |p − qω| ≥ |r − sω|, where<br />

p = z + x, q = y + x, r = 4zx<br />

4xy<br />

z+x<br />

<strong>and</strong> s =<br />

x+y<br />

. It’s clear that p ≥ r > 0 <strong>and</strong> q ≥ s > 0. It follows that<br />

|p − qω| 2 − |r − sω| 2 = (p − qω)(p − qω) − (r − sω)(r − sω) = (p 2 − r 2 ) + (pq − rs) + (q 2 − s 2 ) ≥ 0.<br />

It’s easy to check that the equality holds if <strong>and</strong> only if △ABC is equilateral.<br />

11


Chapter 2<br />

Four Basic <strong>Techniques</strong><br />

Differentiate!<br />

Shi<strong>in</strong>g-shen Chern<br />

2.1 Trigonometric Substitutions<br />

If you are faced with an <strong>in</strong>tegral that conta<strong>in</strong>s square root expressions such as<br />

∫ √1<br />

− x2 dx,<br />

∫ √1<br />

+ y2 dy,<br />

∫ √z2<br />

− 1 dz<br />

then trigonometric substitutions such as x = s<strong>in</strong> t, y = tan t, z = sec t are very useful. We will learn that<br />

mak<strong>in</strong>g a suitable trigonometric substitution simplifies the given <strong>in</strong>equality.<br />

Problem 8. (APMO 2004/5) Prove that, for all positive real numbers a, b, c,<br />

(a 2 + 2)(b 2 + 2)(c 2 + 2) ≥ 9(ab + bc + ca).<br />

First Solution. Choose A, B, C ∈ ( ) √ √ √<br />

0, π 2 with a = 2 tan A, b = 2 tan B, <strong>and</strong> c = 2 tan C. Us<strong>in</strong>g the<br />

well-known trigonometric identity 1 + tan 2 θ = 1<br />

cos 2 θ<br />

, one may rewrite it as<br />

4<br />

≥ cos A cos B cos C (cos A s<strong>in</strong> B s<strong>in</strong> C + s<strong>in</strong> A cos B s<strong>in</strong> C + s<strong>in</strong> A s<strong>in</strong> B cos C) .<br />

9<br />

One may easily check the follow<strong>in</strong>g trigonometric identity<br />

cos(A + B + C) = cos A cos B cos C − cos A s<strong>in</strong> B s<strong>in</strong> C − s<strong>in</strong> A cos B s<strong>in</strong> C − s<strong>in</strong> A s<strong>in</strong> B cos C.<br />

Then, the above trigonometric <strong>in</strong>equality takes the form<br />

4<br />

≥ cos A cos B cos C (cos A cos B cos C − cos(A + B + C)) .<br />

9<br />

Let θ = A+B+C<br />

3<br />

. Apply<strong>in</strong>g the AM-GM <strong>in</strong>equality <strong>and</strong> Jesen’s <strong>in</strong>equality, we have<br />

( ) 3 cos A + cos B + cos C<br />

cos A cos B cos C ≤<br />

≤ cos 3 θ.<br />

3<br />

We now need to show that<br />

Us<strong>in</strong>g the trigonometric identity<br />

4<br />

9 ≥ cos3 θ(cos 3 θ − cos 3θ).<br />

cos 3θ = 4 cos 3 θ − 3 cos θ or cos 3 dgnsθ − cos 3θ = 3 cos θ − 3 cos 3 θ,<br />

it becomes<br />

4<br />

27 ≥ cos4 θ ( 1 − cos 2 θ ) ,<br />

12


which follows from the AM-GM <strong>in</strong>equality<br />

( cos 2 θ<br />

2<br />

· cos2 θ<br />

2<br />

· (1<br />

− cos 2 θ )) 1<br />

3<br />

≤ 1 ( cos 2 θ<br />

+ cos2 θ<br />

+ ( 1 − cos 2 θ )) = 1 3 2 2<br />

3 .<br />

One f<strong>in</strong>d that the equality holds if <strong>and</strong> only if tan A = tan B = tan C = 1 √<br />

2<br />

if <strong>and</strong> only if a = b = c = 1.<br />

Problem 9. (Latvia 2002) Let a, b, c, d be the positive real numbers such that<br />

Prove that abcd ≥ 3.<br />

1<br />

1 + a 4 + 1<br />

1 + b 4 + 1<br />

1 + c 4 + 1<br />

1 + d 4 = 1.<br />

Solution. We can write a 2 = tan A, b 2 = tan B, c 2 = tan C, d 2 = tan D, where A, B, C, D ∈ ( 0, π 2<br />

)<br />

. Then,<br />

the algebraic identity becomes the follow<strong>in</strong>g trigonometric identity :<br />

Apply<strong>in</strong>g the AM-GM <strong>in</strong>equality, we obta<strong>in</strong><br />

Similarly, we obta<strong>in</strong><br />

cos 2 A + cos 2 B + cos 2 C + cos 2 D = 1.<br />

s<strong>in</strong> 2 A = 1 − cos 2 A = cos 2 B + cos 2 C + cos 2 D ≥ 3 (cos B cos C cos D) 2 3<br />

.<br />

s<strong>in</strong> 2 B ≥ 3 (cos C cos D cos A) 2 3<br />

, s<strong>in</strong> 2 C ≥ 3 (cos D cos A cos B) 2 3<br />

, <strong>and</strong> s<strong>in</strong> 2 D ≥ 3 (cos A cos B cos C) 2 3<br />

.<br />

Multiply<strong>in</strong>g these four <strong>in</strong>equalities, we get the result!<br />

Problem 10. (Korea 1998) Let x, y, z be the positive reals with x + y + z = xyz. Show that<br />

1<br />

√ + 1<br />

√ + 1<br />

√ ≤ 3<br />

1 + x<br />

2 1 + y<br />

2 1 + z<br />

2 2 .<br />

S<strong>in</strong>ce the function f is not concave on R + , we cannot apply Jensen’s <strong>in</strong>equality to the function f(t) =<br />

1 √<br />

1+t 2 . However, the function f(tan θ) is concave on ( 0, π 2<br />

)<br />

!<br />

First Solution. We can write x = tan A, y = tan B, z = tan C, where A, B, C ∈ ( 0, π 2<br />

)<br />

. Us<strong>in</strong>g the fact that<br />

1 + tan 2 θ = ( 1<br />

cos θ<br />

) 2, we rewrite it <strong>in</strong> the terms <strong>of</strong> A, B, C :<br />

cos A + cos B + cos C ≤ 3 2 .<br />

It follows from tan(π − C) = −z = x+y<br />

1−xy<br />

= tan(A + B) <strong>and</strong> from π − C, A + B ∈ (0, π) that π − C = A + B<br />

or A + B + C = π. Hence, it suffices to show the follow<strong>in</strong>g.<br />

Theorem 2.1.1. In any acute triangle ABC, we have cos A + cos B + cos C ≤ 3 2 .<br />

Pro<strong>of</strong>. S<strong>in</strong>ce cos x is concave on ( 0, π 2<br />

)<br />

, it’s a direct consequence <strong>of</strong> Jensen’s <strong>in</strong>equality.<br />

We note that the function cos x is not concave on (0, π). In fact, it’s convex on ( π<br />

2 , π) . One may th<strong>in</strong>k<br />

that the <strong>in</strong>equality cos A + cos B + cos C ≤ 3 2<br />

doesn’t hold for any triangles. However, it’s known that it<br />

holds for all triangles.<br />

Theorem 2.1.2. In any triangle ABC, we have cos A + cos B + cos C ≤ 3 2 .<br />

First Pro<strong>of</strong>. It follows from π − C = A + B that cos C = − cos(A + B) = − cos A cos B + s<strong>in</strong> A s<strong>in</strong> B or<br />

3 − 2(cos A + cos B + cos C) = (s<strong>in</strong> A − s<strong>in</strong> B) 2 + (cos A + cos B − 1) 2 ≥ 0.<br />

13


Second Pro<strong>of</strong>. Let BC = a, CA = b, AB = c. Use the Cos<strong>in</strong>e Law to rewrite the given <strong>in</strong>equality <strong>in</strong> the<br />

terms <strong>of</strong> a, b, c :<br />

b 2 + c 2 − a 2<br />

2bc<br />

Clear<strong>in</strong>g denom<strong>in</strong>ators, this becomes<br />

+ c2 + a 2 − b 2<br />

2ca<br />

+ a2 + b 2 − c 2<br />

2ab<br />

≤ 3 2 .<br />

3abc ≥ a(b 2 + c 2 − a 2 ) + b(c 2 + a 2 − b 2 ) + c(a 2 + b 2 − c 2 ),<br />

which is equivalent to abc ≥ (b + c − a)(c + a − b)(a + b − c) <strong>in</strong> the theorem 2.<br />

In the first chapter, we found that the geometric <strong>in</strong>equality R ≥ 2r is equivalent to the algebraic <strong>in</strong>equality<br />

abc ≥ (b + c − a)(c + a − b)(a + b − c). We now f<strong>in</strong>d that, <strong>in</strong> the pro<strong>of</strong> <strong>of</strong> the above theorem, abc ≥<br />

(b + c − a)(c + a − b)(a + b − c) is equivalent to the trigonometric <strong>in</strong>equality cos A + cos B + cos C ≤ 3 2 . One<br />

may ask that<br />

In any triangles ABC, is there a natural relation between cos A + cos B + cos C <strong>and</strong> R r , where R<br />

<strong>and</strong> r are the radii <strong>of</strong> the circumcircle <strong>and</strong> <strong>in</strong>circle <strong>of</strong> ABC ?<br />

Theorem 2.1.3. Let R <strong>and</strong> r denote the radii <strong>of</strong> the circumcircle <strong>and</strong> <strong>in</strong>circle <strong>of</strong> the triangle ABC. Then,<br />

we have cos A + cos B + cos C = 1 + r R .<br />

Pro<strong>of</strong>. Use the identity a(b 2 +c 2 −a 2 )+b(c 2 +a 2 −b 2 )+c(a 2 +b 2 −c 2 ) = 2abc+(b+c−a)(c+a−b)(a+b−c).<br />

We leave the details for the readers.<br />

Exercise 4. (a) Let p, q, r be the positive real numbers such that p 2 + q 2 + r 2 + 2pqr = 1. Show that there<br />

exists an acute triangle ABC such that p = cos A, q = cos B, r = cos C.<br />

(b) Let p, q, r ≥ 0 with p 2 + q 2 + r 2 + 2pqr = 1. Show that there are A, B, C ∈ [ 0, π 2<br />

]<br />

with p = cos A,<br />

q = cos B, r = cos C, <strong>and</strong> A + B + C = π.<br />

Problem 11. (USA 2001) Let a, b, <strong>and</strong> c be nonnegative real numbers such that a 2 + b 2 + c 2 + abc = 4.<br />

Prove that 0 ≤ ab + bc + ca − abc ≤ 2.<br />

Solution. Notice that a, b, c > 1 implies that a 2 +b 2 +c 2 +abc > 4. If a ≤ 1, then we have ab+bc+ca−abc ≥<br />

(1 − a)bc ≥ 0. We now prove that ab + bc + ca − abc ≤ 2. Lett<strong>in</strong>g a = 2p, b = 2q, c = 2r, we get<br />

p 2 + q 2 + r 2 + 2pqr = 1. By the above exercise, we can write<br />

[<br />

a = 2 cos A, b = 2 cos B, c = 2 cos C for some A, B, C ∈ 0, π ]<br />

with A + B + C = π.<br />

2<br />

We are required to prove<br />

cos A cos B + cos B cos C + cos C cos A − 2 cos A cos B cos C ≤ 1 2 .<br />

One may assume that A ≥ π 3<br />

or 1 − 2 cos A ≥ 0. Note that<br />

cos A cos B + cos B cos C + cos C cos A − 2 cos A cos B cos C = cos A(cos B + cos C) + cos B cos C(1 − 2 cos A).<br />

We apply Jensen’s <strong>in</strong>equality to deduce cos B + cos C ≤ 3 2<br />

− cos A. Note that 2 cos B cos C = cos(B − C) +<br />

cos(B + C) ≤ 1 − cos A. These imply that<br />

( ) ( )<br />

3 1 − cos A<br />

cos A(cos B + cos C) + cos B cos C(1 − 2 cos A) ≤ cos A<br />

2 − cos A +<br />

(1 − 2 cos A).<br />

2<br />

However, it’s easy to verify that cos A ( 3<br />

2 − cos A) + ( )<br />

1−cos A<br />

2 (1 − 2 cos A) =<br />

1<br />

2 .<br />

14


2.2 Algebraic Substitutions<br />

We know that some <strong>in</strong>equalities <strong>in</strong> triangle geometry can be treated by the Ravi substitution <strong>and</strong> trigonometric<br />

substitutions. We can also transform the given <strong>in</strong>equalities <strong>in</strong>to easier ones through some clever algebraic<br />

substitutions.<br />

Problem 12. (IMO 2001/2) Let a, b, c be positive real numbers. Prove that<br />

a<br />

√<br />

a2 + 8bc +<br />

b<br />

√<br />

b2 + 8ca +<br />

c<br />

√<br />

c2 + 8ab ≥ 1.<br />

First Solution. To remove the square roots, we make the follow<strong>in</strong>g substitution :<br />

x =<br />

a<br />

√<br />

a2 + 8bc , y =<br />

b<br />

√<br />

b2 + 8ca , z =<br />

c<br />

√<br />

c2 + 8ab .<br />

Clearly, x, y, z ∈ (0, 1). Our aim is to show that x + y + z ≥ 1. We notice that<br />

a 2<br />

8bc =<br />

x2<br />

1 − x 2 , b 2<br />

8ac =<br />

Hence, we need to show that<br />

y2<br />

1 − y 2 , c 2<br />

8ab =<br />

z2<br />

1 − z 2 =⇒ 1 ( ) ( ) ( )<br />

x<br />

2 y<br />

2 z<br />

2<br />

512 = 1 − x 2 1 − y 2 1 − z 2 .<br />

x + y + z ≥ 1, where 0 < x, y, z < 1 <strong>and</strong> (1 − x 2 )(1 − y 2 )(1 − z 2 ) = 512(xyz) 2 .<br />

However, 1 > x + y + z implies that, by the AM-GM <strong>in</strong>equality,<br />

(1 − x 2 )(1 − y 2 )(1 − z 2 ) > ((x + y + z) 2 − x 2 )((x + y + z) 2 − y 2 )((x + y + z) 2 − z 2 ) = (x + x + y + z)(y + z)<br />

(x + y + y + z)(z + x)(x + y + z + z)(x + y) ≥ 4(x 2 yz) 1 1<br />

4 · 2(yz) 2 · 4(y 2 zx) 1 1<br />

4 · 2(zx) 2 · 4(z 2 xy) 1 1<br />

4 · 2(xy) 2<br />

= 512(xyz) 2 . This is a contradiction !<br />

Problem 13. (IMO 1995/2) Let a, b, c be positive numbers such that abc = 1. Prove that<br />

1<br />

a 3 (b + c) + 1<br />

b 3 (c + a) + 1<br />

c 3 (a + b) ≥ 3 2 .<br />

First Solution. After the substitution a = 1 x , b = 1 y , c = 1 z<br />

, we get xyz = 1. The <strong>in</strong>equality takes the form<br />

x 2<br />

y + z +<br />

y2<br />

z + x +<br />

It follows from the Cauchy-Schwarz <strong>in</strong>equality that<br />

( x<br />

2<br />

[(y + z) + (z + x) + (x + y)]<br />

y + z +<br />

so that, by the AM-GM <strong>in</strong>equality,<br />

x 2<br />

y + z +<br />

y2<br />

z + x +<br />

z2<br />

x + y ≥ 3 2 .<br />

y2<br />

z + x +<br />

z2<br />

x + y ≥ x + y + z ≥ 3(xyz) 1 3<br />

2 2<br />

)<br />

z2<br />

≥ (x + y + z) 2<br />

x + y<br />

= 3 2 .<br />

(Korea 1998) Let x, y, z be the positive reals with x + y + z = xyz. Show that<br />

1<br />

√ + 1<br />

√ + 1<br />

√ ≤ 3<br />

1 + x<br />

2 1 + y<br />

2 1 + z<br />

2 2 .<br />

15


Second Solution. The start<strong>in</strong>g po<strong>in</strong>t is lett<strong>in</strong>g a = 1 x , b = 1 y , c = 1 z<br />

. We f<strong>in</strong>d that a + b + c = abc is equivalent<br />

to 1 = xy + yz + zx. The <strong>in</strong>equality becomes<br />

x<br />

√<br />

x2 + 1 +<br />

y<br />

√<br />

y2 + 1 +<br />

z<br />

√<br />

z2 + 1 ≤ 3 2<br />

or<br />

or<br />

x<br />

√<br />

x2 + xy + yz + zx +<br />

x<br />

√<br />

(x + y)(x + z)<br />

+<br />

y<br />

√<br />

y2 + xy + yz + zx +<br />

y<br />

√<br />

(y + z)(y + x)<br />

+<br />

z<br />

√<br />

z2 + xy + yz + zx ≤ 3 2<br />

z<br />

√<br />

(z + x)(z + y)<br />

≤ 3 2 .<br />

By the AM-GM <strong>in</strong>equality, we have<br />

x<br />

√ = x√ (x + y)(x + z)<br />

≤ 1 x[(x + y) + (x + z)]<br />

= 1 ( x<br />

(x + y)(x + z) (x + y)(x + z) 2 (x + y)(x + z) 2 x + z + x )<br />

.<br />

x + z<br />

In a like manner, we obta<strong>in</strong><br />

y<br />

√<br />

(y + z)(y + x)<br />

≤ 1 2<br />

( y<br />

y + z +<br />

Add<strong>in</strong>g these three yields the required result.<br />

y )<br />

y + x<br />

<strong>and</strong><br />

z<br />

√ ≤ 1 ( z<br />

(z + x)(z + y) 2 z + x + z )<br />

.<br />

z + y<br />

We now prove a classical theorem <strong>in</strong> various ways.<br />

Theorem 2.2.1. (Nesbitt, 1903) For all positive real numbers a, b, c, we have<br />

a<br />

b + c +<br />

b<br />

c + a +<br />

c<br />

a + b ≥ 3 2 .<br />

Pro<strong>of</strong> 1. After the substitution x = b + c, y = c + a, z = a + b, it becomes<br />

∑<br />

cyclic<br />

y + z − x<br />

2x<br />

which follows from the AM-GM <strong>in</strong>equality as follow<strong>in</strong>g:<br />

≥ 3 2 or ∑<br />

cyclic<br />

y + z<br />

x<br />

≥ 6,<br />

∑<br />

cyclic<br />

y + z<br />

x<br />

= y x + z x + z y + x y + x z + y ( y<br />

z ≥ 6 x · z<br />

x · z<br />

y · x<br />

y · x<br />

z · y ) 1<br />

6<br />

= 6.<br />

z<br />

Pro<strong>of</strong> 2. We make the substitution<br />

It follows that<br />

x =<br />

∑<br />

f(x) = ∑<br />

cyclic<br />

a<br />

b + c , y =<br />

cyclic<br />

b<br />

c + a , z =<br />

c<br />

a + b .<br />

a<br />

= 1, where f(t) =<br />

t<br />

a + b + c<br />

S<strong>in</strong>ce f is concave on (0, ∞), Jensen’s <strong>in</strong>equality shows that<br />

( 1<br />

f =<br />

2)<br />

1 3 = 1 ∑<br />

( ) x + y + z<br />

f(x) ≤ f<br />

3<br />

3<br />

cyclic<br />

1 + t .<br />

( ( )<br />

1 x + y + z<br />

or f ≤ f<br />

.<br />

2)<br />

3<br />

S<strong>in</strong>ce f is monotone <strong>in</strong>creas<strong>in</strong>g, this implies that<br />

1<br />

2 ≤ x + y + z<br />

3<br />

or<br />

∑<br />

cyclic<br />

a<br />

b + c = x + y + z ≥ 3 2 .<br />

16


Pro<strong>of</strong> 3. As <strong>in</strong> the previous pro<strong>of</strong>, it suffices to show that<br />

T ≥ 1 2 , where T = x + y + z<br />

3<br />

<strong>and</strong><br />

∑<br />

cyclic<br />

x<br />

1 + x = 1.<br />

One can easily check that the condition<br />

∑<br />

cyclic<br />

x<br />

1 + x = 1<br />

becomes 1 = 2xyz + xy + yz + zx. By the AM-GM <strong>in</strong>equality, we have<br />

1 = 2xyz + xy + yz + zx ≤ 2T 3 + 3T 2 ⇒ 2T 3 + 3T 2 − 1 ≥ 0 ⇒ (2T − 1)(T + 1) 2 ≥ 0 ⇒ T ≥ 1 2 .<br />

(IMO 2000/2) Let a, b, c be positive numbers such that abc = 1. Prove that<br />

(<br />

a − 1 + 1 ) (<br />

b − 1 + 1 ) (<br />

c − 1 + 1 )<br />

≤ 1.<br />

b<br />

c<br />

a<br />

Second Solution. ([IV], Ilan Vardi) S<strong>in</strong>ce abc = 1, we may assume that a ≥ 1 ≥ b. 1 It follows that<br />

(<br />

1 − a − 1 + 1 ) (<br />

b − 1 + 1 ) (<br />

c − 1 + 1 )<br />

=<br />

(c + 1 ) (a<br />

b<br />

c<br />

a c − 2 + 1 )<br />

b − 1 (a − 1)(1 − b)<br />

+ . 2<br />

a<br />

Third Solution. As <strong>in</strong> the first solution, after the substitution a = x y , b = y z , c = z x<br />

for x, y, z > 0, we<br />

can rewrite it as xyz ≥ (y + z − x)(z + x − y)(x + y − z). Without loss <strong>of</strong> generality, we can assume that<br />

z ≥ y ≥ x. Set y − x = p <strong>and</strong> z − x = q with p, q ≥ 0. It’s straightforward to verify that<br />

xyz − (y + z − x)(z + x − y)(x + y − z) = (p 2 − pq + q 2 )x + (p 3 + q 3 − p 2 q − pq 2 ).<br />

S<strong>in</strong>ce p 2 − pq + q 2 ≥ (p − q) 2 ≥ 0 <strong>and</strong> p 3 + q 3 − p 2 q − pq 2 = (p − q) 2 (p + q) ≥ 0, we get the result.<br />

Fourth Solution. (From the IMO 2000 Short List) Us<strong>in</strong>g the condition abc = 1, it’s straightforward to<br />

verify the equalities<br />

2 = 1 (<br />

a − 1 + 1 ) (<br />

+ c b − 1 + 1 )<br />

,<br />

a b<br />

c<br />

2 = 1 b<br />

2 = 1 c<br />

(<br />

b − 1 + 1 ) (<br />

+ a c − 1 + 1 )<br />

,<br />

c<br />

a<br />

(<br />

c − 1 + 1 ) (<br />

+ b a − 1 + 1 )<br />

.<br />

a<br />

c<br />

In particular, they show that at most one <strong>of</strong> the numbers u = a − 1 + 1 b , v = b − 1 + 1 c , w = c − 1 + 1 a is<br />

negative. If there is such a number, we have<br />

(<br />

a − 1 + 1 ) (<br />

b − 1 + 1 ) (<br />

c − 1 + 1 )<br />

= uvw < 0 < 1.<br />

b<br />

c<br />

a<br />

And if u, v, w ≥ 0, the AM-GM <strong>in</strong>equality yields<br />

2 = 1 a u + cv ≥ 2 √ c<br />

a uv, 2 = 1 b v + aw ≥ 2 √ a<br />

b vw, 2 = 1 c w + aw ≥ 2 √<br />

b<br />

c wu.<br />

Thus, uv ≤ a c , vw ≤ b a , wu ≤ c b , so (uvw)2 ≤ a c · b<br />

a · c<br />

b<br />

= 1. S<strong>in</strong>ce u, v, w ≥ 0, this completes the pro<strong>of</strong>.<br />

1 Why? Note that the <strong>in</strong>equality is not symmetric <strong>in</strong> the three variables. Check it!<br />

2 For a verification <strong>of</strong> the identity, see [IV].<br />

17


Problem 14. Let a, b, c be positive real numbers satisfy<strong>in</strong>g a + b + c = 1. Show that<br />

Solution. We want to establish that<br />

Set x =<br />

√<br />

bc<br />

a , y = √ ca<br />

b , z = √<br />

a<br />

a + bc + b<br />

√<br />

abc<br />

b + ca + c + ab ≤ 1 + 3√ 3<br />

4 .<br />

1<br />

1 + bc a<br />

+ 1<br />

1 + ca b<br />

+<br />

√<br />

ab<br />

c<br />

1 + ab<br />

c<br />

ab<br />

c<br />

. We need to prove that<br />

≤ 1 + 3√ 3<br />

4 .<br />

1<br />

1 + x 2 + 1<br />

1 + y 2 + z<br />

1 + z 2 ≤ 1 + 3√ 3<br />

4 ,<br />

where x, y, z > 0 <strong>and</strong> xy + yz + zx = 1. It’s not hard to show that there exists A, B, C ∈ (0, π) with<br />

The <strong>in</strong>equality becomes<br />

x = tan A 2 , y = tan B 2 , z = tan C , <strong>and</strong> A + B + C = π.<br />

2<br />

or<br />

or<br />

1<br />

1 + ( 1<br />

)<br />

tan A 2<br />

+<br />

2<br />

1 + ( tan C 2<br />

)<br />

tan B 2<br />

+<br />

2<br />

1 + ( tan C 2<br />

1 + 1 2 (cos A + cos B + s<strong>in</strong> C) ≤ 1 + 3√ 3<br />

4<br />

cos A + cos B + s<strong>in</strong> C ≤ 3√ 3<br />

2 .<br />

) 2<br />

≤ 1 + 3√ 3<br />

4<br />

Note that cos A + cos B = 2 cos ( ) (<br />

A+B<br />

2 cos A−B<br />

) ∣ 2 . S<strong>in</strong>ce A−B ∣<br />

2<br />

< π 2<br />

, this means that<br />

( ) ( )<br />

A + B<br />

π − C<br />

cos A + cos B ≤ 2 cos = 2 cos .<br />

2<br />

2<br />

It will be enough to show that<br />

( ) π − C<br />

2 cos + s<strong>in</strong> C ≤ 3√ 3<br />

2<br />

2 ,<br />

where C ∈ (0, π). This is a one-variable <strong>in</strong>equality. 3 It’s left as an exercise for the reader.<br />

Problem 15. (Iran 1998) Prove that, for all x, y, z > 1 such that 1 x + 1 y + 1 z = 2,<br />

√ x + y + z ≥<br />

√<br />

x − 1 +<br />

√<br />

y − 1 +<br />

√<br />

z − 1.<br />

First Solution. We beg<strong>in</strong> with the algebraic substitution a = √ x − 1, b = √ y − 1, c = √ z − 1. Then, the<br />

condition becomes<br />

1<br />

1 + a 2 + 1<br />

1 + b 2 + 1<br />

1 + c 2 = 2 ⇔ a2 b 2 + b 2 c 2 + c 2 a 2 + 2a 2 b 2 c 2 = 1<br />

<strong>and</strong> the <strong>in</strong>equality is equivalent to<br />

√<br />

a2 + b 2 + c 2 + 3 ≥ a + b + c ⇔ ab + bc + ca ≤ 3 2 .<br />

Let p = bc, q = ca, r = ab. Our job is to prove that p + q + r ≤ 3 2 where p2 + q 2 + r 2 + 2pqr = 1. By the<br />

exercise 7, we can make the trigonometric substitution<br />

(<br />

p = cos A, q = cos B, r = cos C for some A, B, C ∈ 0, π )<br />

with A + B + C = π.<br />

2<br />

What we need to show is now that cos A + cos B + cos C ≤ 3 2<br />

. It follows from Jensen’s <strong>in</strong>equality.<br />

3<br />

Differentiate! Shi<strong>in</strong>g-shen Chern<br />

18


Problem 16. (IMO Short-list 2001) Let x 1 , · · · , x n be arbitrary real numbers. Prove the <strong>in</strong>equality.<br />

x 1<br />

1 + x 1<br />

2 + x 2<br />

1 + x 12 + x 2<br />

2 + · · · + x n<br />

1 + x 12 + · · · + x n<br />

2 < √ n.<br />

First Solution. We only consider the case when x 1 , · · · , x n are all nonnegative real numbers.(Why?) 4 Let<br />

x 0 = 1. After the substitution y i = x 2 0 + · · · + x 2 i for all i = 0, · · · , n, we obta<strong>in</strong> x i = √ y i − y i−1 . We need<br />

to prove the follow<strong>in</strong>g <strong>in</strong>equality<br />

n∑<br />

√<br />

yi − y i−1<br />

< √ n.<br />

y i<br />

S<strong>in</strong>ce y i ≥ y i−1 for all i = 1, · · · , n, we have an upper bound <strong>of</strong> the left h<strong>and</strong> side:<br />

n∑<br />

√<br />

yi − y i−1<br />

n∑<br />

√ √<br />

yi − y i−1<br />

n∑ 1<br />

≤ √ =<br />

− 1 y i<br />

yi y i−1 y i−1 y i<br />

i=0<br />

i=0<br />

i=0<br />

We now apply the Cauchy-Schwarz <strong>in</strong>equality to give an upper bound <strong>of</strong> the last term:<br />

√<br />

n∑ 1<br />

− 1 ∑<br />

≤ √ n ( 1 n<br />

− 1 ) √ ( 1<br />

= n − 1 )<br />

.<br />

y i−1 y i y i−1 y i y 0 y n<br />

S<strong>in</strong>ce y 0 = 1 <strong>and</strong> y n > 0, this yields the desired upper bound √ n.<br />

i=0<br />

i=0<br />

i=0<br />

Second Solution. We may assume that x 1 , · · · , x n are all nonnegative real numbers. Let x 0 = 0. We make<br />

the follow<strong>in</strong>g algebraic substitution<br />

t i =<br />

x i<br />

√<br />

x02 + · · · + x i<br />

2 , c i =<br />

1<br />

√<br />

1 + ti<br />

2 <strong>and</strong> s i =<br />

t i<br />

√<br />

1 + ti<br />

2<br />

for all i = 0, · · · , n. It’s an easy exercise to show that<br />

desired <strong>in</strong>equality becomes<br />

x i<br />

x 0 2 +···+x i<br />

2<br />

c 0 c 1<br />

√<br />

1 − c12 + c 0 c 1 c 2<br />

√<br />

1 − c22 + · · · + c 0 c 1 · · · c n<br />

√<br />

1 − cn2 < √ n.<br />

S<strong>in</strong>ce 0 < c i ≤ 1 for all i = 1, · · · , n, we have<br />

n∑ √<br />

c 0 · · · c i 1 − ci2 ≤<br />

i=1<br />

n∑ √<br />

c 0 · · · c i−1 1 − ci2 =<br />

i=1<br />

= c 0 · · · c i s i . S<strong>in</strong>ce s i = √ 1 − c i2 , the<br />

n∑ √<br />

(c0 · · · c i−1 ) 2 − (c 0 · · · c i−1 c i ) 2 .<br />

S<strong>in</strong>ce c 0 = 1, by the Cauchy-Schwarz <strong>in</strong>equality, we obta<strong>in</strong><br />

n∑ √ ∑<br />

(c0 · · · c i−1 ) 2 − (c 0 · · · c i−1 c i ) 2 ≤ √ n n [(c 0 · · · c i−1 ) 2 − (c 0 · · · c i−1 c i ) 2 ] = √ n [1 − (c 0 · · · c n ) 2 ].<br />

i=1<br />

i=1<br />

i=1<br />

4 x 1<br />

1+x 1<br />

2 +<br />

x 2<br />

1+x 1 2 +x 2<br />

2 + · · · +<br />

x n<br />

1+x 2 1 +···+x 2 ≤ |x 1|<br />

n 1+x 2 +<br />

1<br />

|x 2 |<br />

1+x 1 2 +x 2<br />

2 + · · · +<br />

|x n |<br />

1+x 1 2 +···+x n<br />

2 .<br />

19


2.3 Increas<strong>in</strong>g Function Theorem<br />

Theorem 2.3.1. (Increas<strong>in</strong>g Function Theorem) Let f : (a, b) −→ R be a differentiable function. If<br />

f ′ (x) ≥ 0 for all x ∈ (a, b), then f is monotone <strong>in</strong>creas<strong>in</strong>g on (a, b). If f ′ (x) > 0 for all x ∈ (a, b), then f is<br />

strictly <strong>in</strong>creas<strong>in</strong>g on (a, b).<br />

Pro<strong>of</strong>. We first consider the case when f ′ (x) > 0 for all x ∈ (a, b). Let a < x 1 < x 2 < b. We want<br />

to show that f(x 1 ) < f(x 2 ). Apply<strong>in</strong>g the Mean Value Theorem, we f<strong>in</strong>d some c ∈ (x 1 , x 2 ) such that<br />

f(x 2 ) − f(x 1 ) = f ′ (c)(x 2 − x 1 ). S<strong>in</strong>ce f ′ (c) > 0, this equation means that f(x 2 ) − f(x 1 ) > 0. In case when<br />

f ′ (x) ≥ 0 for all x ∈ (a, b), we can also apply the Mean Value Theorem to get the result.<br />

Problem 17. (Irel<strong>and</strong> 2000) Let x, y ≥ 0 with x + y = 2. Prove that x 2 y 2 (x 2 + y 2 ) ≤ 2.<br />

First Solution. After homogeniz<strong>in</strong>g it, we need to prove<br />

( ) 6 x + y<br />

2 ≥ x 2 y 2 (x 2 + y 2 ) or (x + y) 6 ≥ 32x 2 y 2 (x 2 + y 2 ).<br />

2<br />

(Now, forget the constra<strong>in</strong>t x+y = 2!) In case xy = 0, it clearly holds. We now assume that xy ≠ 0. Because<br />

<strong>of</strong> the homogeneity <strong>of</strong> the <strong>in</strong>equality, this means that we may normalize to xy = 1. Then, it becomes<br />

(<br />

x +<br />

x) 1 6<br />

≥ 32<br />

(x 2 + 1 )<br />

x 2<br />

or p 3 ≥ 32(p − 2).<br />

where p = ( x + x) 1 2<br />

≥ 4. Our job is now to m<strong>in</strong>imize F (p) = p 3 − 32(p − 2) on [4, ∞). S<strong>in</strong>ce F ′ (p) =<br />

√<br />

3p 2 32<br />

− 32 ≥ 0, where p ≥<br />

3<br />

, F is (monotone) <strong>in</strong>creas<strong>in</strong>g on [4, ∞). So, F (p) ≥ F (4) = 0 for all p ≥ 4.<br />

Second Solution. As <strong>in</strong> the first solution, we prove that (x + y) 6 ≥ 32(x 2 + y 2 )(xy) 2 for all x, y ≥ 0. In case<br />

x = y = 0, it’s clear. Now, if x 2 + y 2 > 0, then we may normalize to x 2 + y 2 = 2. Sett<strong>in</strong>g p = xy, we have<br />

0 ≤ p ≤ x2 +y 2<br />

2<br />

= 1 <strong>and</strong> (x + y) 2 = x 2 + y 2 + 2xy = 2 + 2p. It now becomes<br />

(2 + 2p) 3 ≥ 64p 2 or p 3 − 5p 2 + 3p + 1 ≥ 0.<br />

We want to m<strong>in</strong>imize F (p) = p 3 − 5p 2 + 3p + 1 on [0, 1]. We compute F ′ (p) = 3 ( p − 3) 1 (p − 3). We f<strong>in</strong>d<br />

that F is monotone <strong>in</strong>creas<strong>in</strong>g on [0, 1 3 ] <strong>and</strong> monotone decreas<strong>in</strong>g on [ 1 3<br />

, 1]. S<strong>in</strong>ce F (0) = 1 <strong>and</strong> F (1) = 0,<br />

we conclude that F (p) ≥ F (1) = 0 for all p ∈ [0, 1].<br />

Third Solution. We show that (x + y) 6 ≥ 32(x 2 + y 2 )(xy) 2 where x ≥ y ≥ 0. We make the substitution<br />

u = x + y <strong>and</strong> v = x − y. Then, we have u ≥ v ≥ 0. It becomes<br />

( u<br />

u 6 2 + v 2 ) ( u 2 − v 2 ) 2<br />

≥ 32<br />

or u 6 ≥ (u 2 + v 2 )(u 2 − v 2 ) 2 .<br />

2 4<br />

Note that u 4 ≥ u 4 − v 4 ≥ 0 <strong>and</strong> that u 2 ≥ u 2 − v 2 ≥ 0. So, u 6 ≥ (u 4 − v 4 )(u 2 − v 2 ) = (u 2 + v 2 )(u 2 − v 2 ) 2 .<br />

Problem 18. (IMO 1984/1) Let x, y, z be nonnegative real numbers such that x + y + z = 1. Prove that<br />

0 ≤ xy + yz + zx − 2xyz ≤ 7<br />

27 .<br />

First Solution. Let f(x, y, z) = xy + yz + zx − 2xyz. We may assume that 0 ≤ x ≤ y ≤ z ≤ 1. S<strong>in</strong>ce<br />

x + y + z = 1, this implies that x ≤ 1 3<br />

. It follows that f(x, y, z) = (1 − 3x)yz + xyz + zx + xy ≥ 0. Apply<strong>in</strong>g<br />

the AM-GM <strong>in</strong>equality, we obta<strong>in</strong> yz ≤ ( )<br />

y+z 2 (<br />

2 = 1−x<br />

) 2.<br />

2 S<strong>in</strong>ce 1 − 2x ≥ 0, this implies that<br />

( ) 2 1 − x<br />

f(x, y, z) = x(y + z) + yz(1 − 2x) ≤ x(1 − x) + (1 − 2x) = −2x3 + x 2 + 1<br />

.<br />

2<br />

4<br />

Our job is now to maximize a one-variable function F (x) = 1 4 (−2x3 + x 2 + 1), where x ∈ [ 0, 3] 1 . S<strong>in</strong>ce<br />

F ′ (x) = 3 2 x ( 1<br />

3 − x) ≥ 0 on [ ]<br />

0, 1 3 , we conclude that F (x) ≤ F (<br />

1<br />

3 ) = 7<br />

27 for all x ∈ [ 0, 3] 1 .<br />

20


(IMO 2000/2) Let a, b, c be positive numbers such that abc = 1. Prove that<br />

(<br />

a − 1 + 1 ) (<br />

b − 1 + 1 ) (<br />

c − 1 + 1 )<br />

≤ 1.<br />

b<br />

c<br />

a<br />

Fifth Solution. (based on work by an IMO 2000 contestant from Japan) S<strong>in</strong>ce abc = 1, at least one <strong>of</strong> a, b,<br />

c is greater than or equal to 1. Say b ≥ 1. Putt<strong>in</strong>g c = 1 ab<br />

, it becomes<br />

(<br />

a − 1 + 1 )<br />

( 1<br />

(b − 1 + ab)<br />

b<br />

ab a)<br />

− 1 + 1 ≤ 1<br />

or<br />

Sett<strong>in</strong>g x = ab, it becomes f b (x) ≥ 0, where<br />

a 3 b 3 − a 2 b 3 − ab 3 − a 2 b 2 + 3ab 2 − ab + b 3 − b 2 − b + 1 ≥ 0.<br />

f b (t) = t 3 + b 3 − b 2 t − bt 2 + 3bt − t 2 − b 2 − t − b + 1.<br />

Fix a positive number b ≥ 1. We need to show that F (t) := f b (t) ≥ 0 for all t ≥ 0. It follows from b ≥ 1<br />

that the cubic polynomial F ′ (t) = 3t 2 − 2(b + 1)t − (b 2 − 3b + 1) has two real roots<br />

b + 1 − √ 4b 2 − 7b + 4<br />

3<br />

<strong>and</strong> λ = b + 1 + √ 4b 2 − 7b + 4<br />

.<br />

3<br />

S<strong>in</strong>ce F has a local m<strong>in</strong>imum at t = λ, we f<strong>in</strong>d that F (t) ≥ M<strong>in</strong> {F (0), F (λ)} for all t ≥ 0. We have to<br />

prove that F (0) ≥ 0 <strong>and</strong> F (λ) ≥ 0. We have F (0) = b 3 − b 2 − b + 1 = (b − 1) 2 (b + 1) ≥ 0. It rema<strong>in</strong>s to show<br />

that F (λ) ≥ 0. Notice that λ is a root <strong>of</strong> F / (t). After long division, we get<br />

F (t) = F ′ (t)<br />

( 1<br />

3 t − b + 1<br />

9<br />

)<br />

+ 1 (<br />

(−8b 2 + 14b − 8)t + 8b 3 − 7b 2 − 7b + 8 ) .<br />

9<br />

Putt<strong>in</strong>g t = λ, we have<br />

F (λ) = 1 9<br />

(<br />

(−8b 2 + 14b − 8)λ + 8b 3 − 7b 2 − 7b + 8 ) .<br />

Thus, our job is now to establish that, for all b ≥ 0,<br />

(<br />

(−8b 2 b + 1 + √ )<br />

4b<br />

+ 14b − 8)<br />

2 − 7b + 4<br />

+ 8b 3 − 7b 2 − 7b + 8 ≥ 0,<br />

3<br />

which is equivalent to<br />

16b 3 − 15b 2 − 15b + 16 ≥ (8b 2 − 14b + 8) √ 4b 2 − 7b + 4 .<br />

S<strong>in</strong>ce both 16b 3 − 15b 2 − 15b + 16 <strong>and</strong> 8b 2 − 14b + 8 are positive, 5 it’s equivalent to<br />

or<br />

(16b 3 − 15b 2 − 15b + 16) 2 ≥ (8b 2 − 14b + 8) 2 (4b 2 − 7b + 4)<br />

864b 5 − 3375b 4 + 5022b 3 − 3375b 2 + 864b ≥ 0 or 864b 4 − 3375b 3 + 5022b 2 − 3375b + 864 ≥ 0.<br />

Let G(x) = 864x 4 − 3375x 3 + 5022x 2 − 3375x + 864. We prove that G(x) ≥ 0 for all x ∈ R. We f<strong>in</strong>d that<br />

G ′ (x) = 3456x 3 − 10125x 2 + 10044x − 3375 = (x − 1)(3456x 2 − 6669x + 3375).<br />

S<strong>in</strong>ce 3456x 2 − 6669x + 3375 > 0 for all x ∈ R, we f<strong>in</strong>d that G(x) <strong>and</strong> x − 1 have the same sign. It follows<br />

that G is monotone decreas<strong>in</strong>g on (−∞, 1] <strong>and</strong> monotone <strong>in</strong>creas<strong>in</strong>g on [1, ∞). We conclude that G has the<br />

global m<strong>in</strong>imum at x = 1. Hence, G(x) ≥ G(1) = 0 for all x ∈ R.<br />

5 It’s easy to check that 16b 3 − 15b 2 − 15b + 16 = 16(b 3 − b 2 − b + 1) + b 2 + b > 16(b 2 − 1)(b − 1) ≥ 0 <strong>and</strong> 8b 2 − 14b + 8 =<br />

8(b − 1) 2 + 2b > 0.<br />

21


2.4 Establish<strong>in</strong>g New Bounds<br />

We first give two alternative ways to prove Nesbitt’s <strong>in</strong>equality.<br />

(Nesbitt) For all positive real numbers a, b, c, we have<br />

a<br />

b + c +<br />

(<br />

2<br />

a<br />

Pro<strong>of</strong> 4. From<br />

b+c 2) − 1 ≥ 0, we deduce that<br />

It follows that<br />

Pro<strong>of</strong> 5. We claim that<br />

a<br />

b + c ≥ 1 4 ·<br />

∑<br />

cyclic<br />

b<br />

c + a +<br />

8a<br />

b+c − 1<br />

a<br />

b+c + 1 =<br />

a<br />

b + c ≥ ∑<br />

cyclic<br />

c<br />

a + b ≥ 3 2 .<br />

8a − b − c<br />

4(a + b + c) .<br />

8a − b − c<br />

4(a + b + c) = 3 2 .<br />

a<br />

b + c ≥ 3a 3 (<br />

)<br />

2<br />

(<br />

) or 2 a 3 3 3<br />

2 + b 2 + c 2 ≥ 3a 1 2 (b + c).<br />

2 a 3 2 + b 3 2 + c 3 2<br />

The AM-GM <strong>in</strong>equality gives a 3 2 +b 3 2 +b 3 2 ≥ 3a 1 2 b <strong>and</strong> a 3 2 +c 3 2 +c 3 2 ≥ 3a 1 2<br />

(<br />

)<br />

c . Add<strong>in</strong>g these two <strong>in</strong>equalities<br />

yields 2 a 3 2 + b 3 2 + c 3 2 ≥ 3a 1 2 (b + c), as desired. Therefore, we have<br />

∑<br />

cyclic<br />

a<br />

b + c ≥ 3 2<br />

∑<br />

cyclic<br />

a 3 2<br />

a 3 2 + b 3 2 + c 3 2<br />

= 3 2 .<br />

Some cyclic <strong>in</strong>equalities can be proved by f<strong>in</strong>d<strong>in</strong>g new bounds. Suppose that we want to establish that<br />

∑<br />

F (x, y, z) ≥ C.<br />

If a function G satisfies<br />

cyclic<br />

(1) F (x, y, z) ≥ G(x, y, z) for all x, y, z > 0, <strong>and</strong><br />

(2) ∑ cyclic<br />

G(x, y, z) = C for all x, y, z > 0,<br />

then, we deduce that<br />

∑<br />

F (x, y, z) ≥ ∑<br />

G(x, y, z) = C.<br />

cyclic<br />

For example, if a function F satisfies<br />

F (x, y, z) ≥<br />

cyclic<br />

x<br />

x + y + z<br />

for all x, y, z > 0, then, tak<strong>in</strong>g the cyclic sum yields<br />

∑<br />

F (x, y, z) ≥ 1.<br />

cyclic<br />

As we saw <strong>in</strong> the above two pro<strong>of</strong>s <strong>of</strong> Nesbitt’s <strong>in</strong>equality, there are various lower bounds.<br />

Problem 19. Let a, b, c be the lengths <strong>of</strong> a triangle. Show that<br />

a<br />

b + c +<br />

b<br />

c + a +<br />

c<br />

a + b < 2.<br />

22


Pro<strong>of</strong>. We don’t employ the Ravi substitution. It follows from the triangle <strong>in</strong>equality that<br />

∑ a<br />

b + c < ∑ a<br />

1<br />

= 2.<br />

2<br />

(a + b + c)<br />

cyclic<br />

cyclic<br />

One day, I tried f<strong>in</strong>d<strong>in</strong>g a new lower bound <strong>of</strong> (x + y + z) 2 where x, y, z > 0 . There are well-known lower<br />

bounds such as 3(xy + yz + zx) <strong>and</strong> 9(xyz) 2 3 . But I wanted to f<strong>in</strong>d quite different one. I tried break<strong>in</strong>g the<br />

symmetry <strong>of</strong> the three variables x, y, z. Note that<br />

(x + y + z) 2 = x 2 + y 2 + z 2 + xy + xy + yz + yz + zx + zx.<br />

I applied the AM-GM <strong>in</strong>equality to the right h<strong>and</strong> side except the term x 2 :<br />

It follows that<br />

y 2 + z 2 + xy + xy + yz + yz + zx + zx ≥ 8x 1 2 y<br />

3<br />

4 z<br />

3<br />

4 .<br />

(<br />

)<br />

(x + y + z) 2 ≥ x 2 + 8x 1 3 3 1<br />

2 y 4 z 4 = x 2 x 3 3 3<br />

2 + 8y 4 z 4 .<br />

(IMO 2001/2) Let a, b, c be positive real numbers. Prove that<br />

a<br />

√<br />

a2 + 8bc +<br />

b<br />

√<br />

b2 + 8ca +<br />

c<br />

√<br />

c2 + 8ab ≥ 1.<br />

Second Solution. We f<strong>in</strong>d that the above <strong>in</strong>equality also gives another lower bound <strong>of</strong> x + y + z, that is,<br />

√ (<br />

)<br />

x + y + z ≥ x 1 2 x 3 2 + 8y 3 4 z 3 4 .<br />

It follows that<br />

∑<br />

cyclic<br />

x 3 4<br />

√<br />

x 3 2 + 8y 3 4 z 3 4<br />

≥ ∑<br />

cyclic<br />

x<br />

x + y + z = 1.<br />

After the substitution x = a 4 3 , y = b 4 3 , <strong>and</strong> z = c 4 3 , it now becomes<br />

∑ a<br />

√<br />

a2 + 8bc ≥ 1.<br />

cyclic<br />

Problem 20. (IMO 2005/3) Let x, y, <strong>and</strong> z be positive numbers such that xyz ≥ 1. Prove that<br />

x 5 − x 2<br />

x 5 + y 2 + z 2 + y5 − y 2<br />

y 5 + z 2 + x 2 + z5 − z 2<br />

z 5 + x 2 + y 2 ≥ 0.<br />

First Solution. It’s equivalent to the follow<strong>in</strong>g <strong>in</strong>equality<br />

( x 2 − x 5 )<br />

x 5 + y 2 + z 2 + 1 +<br />

( y 2 − y 5<br />

y 5 + z 2 + x 2 + 1 )<br />

+<br />

( z 2 − z 5<br />

z 5 + x 2 + y 2 + 1 )<br />

≤ 3<br />

or<br />

x 2 + y 2 + z 2<br />

x 5 + y 2 + z 2 + x2 + y 2 + z 2<br />

y 5 + z 2 + x 2 + x2 + y 2 + z 2<br />

z 5 + x 2 + y 2 ≤ 3.<br />

With the Cauchy-Schwarz <strong>in</strong>equality <strong>and</strong> the fact that xyz ≥ 1, we have<br />

(x 5 + y 2 + z 2 )(yz + y 2 + z 2 ) ≥ (x 2 + y 2 + z 2 ) 2 or x2 + y 2 + z 2<br />

x 5 + y 2 + z 2 ≤ yz + y2 + z 2<br />

x 2 + y 2 + z 2 .<br />

Tak<strong>in</strong>g the cyclic sum <strong>and</strong> x 2 + y 2 + z 2 ≥ xy + yz + zx give us<br />

x 2 + y 2 + z 2<br />

x 5 + y 2 + z 2 + x2 + y 2 + z 2<br />

y 5 + z 2 + x 2 + x2 + y 2 + z 2 xy + yz + zx<br />

z 5 + x 2 ≤ 2 +<br />

+ y2 x 2 + y 2 + z 2 ≤ 3.<br />

23


Second Solution. The ma<strong>in</strong> idea is to th<strong>in</strong>k <strong>of</strong> 1 as follows :<br />

x 5<br />

x 5 + y 2 + z 2 + y 5<br />

y 5 + z 2 + x 2 + z 5<br />

z 5 + x 2 + y 2 ≥ 1 ≥ x 2<br />

x 5 + y 2 + z 2 + y 2<br />

y 5 + z 2 + x 2 + z 2<br />

z 5 + x 2 + y 2 .<br />

We first show the left-h<strong>and</strong>. It follows from y 4 + z 4 ≥ y 3 z + yz 3 = yz(y 2 + z 2 ) that<br />

x(y 4 + z 4 ) ≥ xyz(y 2 + z 2 ) ≥ y 2 + z 2<br />

or<br />

x 5<br />

x 5 + y 2 + z 2 ≥ x 5<br />

x 5 + xy 4 + xz 4 = x 4<br />

x 4 + y 4 + z 4 .<br />

Tak<strong>in</strong>g the cyclic sum, we have the required <strong>in</strong>equality. It rema<strong>in</strong>s to show the right-h<strong>and</strong>.<br />

[First Way] As <strong>in</strong> the first solution, the Cauchy-Schwarz <strong>in</strong>equality <strong>and</strong> xyz ≥ 1 imply that<br />

(x 5 + y 2 + z 2 )(yz + y 2 + z 2 ) ≥ (x 2 + y 2 + z 2 ) 2 or x2 (yz + y 2 + z 2 )<br />

(x 2 + y 2 + z 2 ) 2 ≥ x 2<br />

x 5 + y 2 + z 2 .<br />

Tak<strong>in</strong>g the cyclic sum, we have<br />

∑<br />

cyclic<br />

x 2 (yz + y 2 + z 2 )<br />

(x 2 + y 2 + z 2 ) 2 ≥ ∑<br />

cyclic<br />

Our job is now to establish the follow<strong>in</strong>g homogeneous <strong>in</strong>equality<br />

1 ≥ ∑<br />

cyclic<br />

x 2 (yz + y 2 + z 2 )<br />

(x 2 + y 2 + z 2 ) 2 ⇔ (x2 + y 2 + z 2 ) 2 ≥ 2 ∑<br />

However, by the AM-GM <strong>in</strong>equality, we obta<strong>in</strong><br />

cyclic<br />

cyclic<br />

[Second Way] We claim that<br />

cyclic<br />

∑<br />

x 4 = ∑ x 4 + y 4<br />

≥ ∑<br />

x 2 y 2 = ∑<br />

2<br />

cyclic<br />

cyclic<br />

x 2<br />

x 5 + y 2 + z 2 .<br />

x 2 y 2 + ∑<br />

x 2 yz ⇔ ∑<br />

x 4 ≥ ∑<br />

x 2 yz.<br />

cyclic<br />

cyclic<br />

( y<br />

x 2 2 + z 2 )<br />

≥ ∑<br />

x 2 yz.<br />

2<br />

cyclic<br />

2x 4 + y 4 + z 4 + 4x 2 y 2 + 4x 2 z 2 x 2<br />

4(x 2 + y 2 + z 2 ) 2 ≥<br />

x 5 + y 2 + z 2 .<br />

We do this by prov<strong>in</strong>g<br />

2x 4 + y 4 + z 4 + 4x 2 y 2 + 4x 2 z 2 x 2 yz<br />

4(x 2 + y 2 + z 2 ) 2 ≥<br />

x 4 + y 3 z + yz 3<br />

because xyz ≥ 1 implies that<br />

Hence, we need to show the homogeneous <strong>in</strong>equality<br />

x 2 yz<br />

x 4 + y 3 z + yz 3 = x 2<br />

x 5<br />

xyz + y2 + z ≥ x 2<br />

2 x 5 + y 2 + z 2 .<br />

(2x 4 + y 4 + z 4 + 4x 2 y 2 + 4x 2 z 2 )(x 4 + y 3 z + yz 3 ) ≥ 4x 2 yz(x 2 + y 2 + z 2 ) 2 .<br />

However, this is a straightforward consequence <strong>of</strong> the AM-GM <strong>in</strong>equality.<br />

(2x 4 + y 4 + z 4 + 4x 2 y 2 + 4x 2 z 2 )(x 4 + y 3 z + yz 3 ) − 4x 2 yz(x 2 + y 2 + z 2 ) 2<br />

cyclic<br />

= (x 8 + x 4 y 4 + x 6 y 2 + x 6 y 2 + y 7 z + y 3 z 5 ) + (x 8 + x 4 z 4 + x 6 z 2 + x 6 z 2 + yz 7 + y 5 z 3 )<br />

+2(x 6 y 2 + x 6 z 2 ) − 6x 4 y 3 z − 6x 4 yz 3 − 2x 6 yz<br />

≥ 6 6√ x 8 · x 4 y 4 · x 6 y 2 · x 6 y 2 · y 7 z · y 3 z 5 + 6 6√ x 8 · x 4 z 4 · x 6 z 2 · x 6 z 2 · yz 7 · y 5 z 3<br />

= 0.<br />

+2 √ x 6 y 2 · x 6 z 2 − 6x 4 y 3 z − 6x 4 yz 3 − 2x 6 yz<br />

Tak<strong>in</strong>g the cyclic sum, we obta<strong>in</strong><br />

1 = ∑<br />

cyclic<br />

2x 4 + y 4 + z 4 + 4x 2 y 2 + 4x 2 z 2<br />

4(x 2 + y 2 + z 2 ) 2 ≥ ∑<br />

cyclic<br />

x 2<br />

x 5 + y 2 + z 2 .<br />

24


Third Solution. (by an IMO 2005 contestant Iurie Boreico 6 from Moldova) We establish that<br />

It follows immediately from the identity<br />

x 5 − x 2<br />

x 5 + y 2 + z 2 ≥ x 5 − x 2<br />

x 3 (x 2 + y 2 + z 2 ) .<br />

x 5 − x 2<br />

x 5 + y 2 + z 2 − x 5 − x 2<br />

x 3 (x 2 + y 2 + z 2 ) = (x 3 − 1) 2 x 2 (y 2 + z 2 )<br />

x 3 (x 2 + y 2 + z 2 )(x 5 + y 2 + z 2 ) .<br />

Tak<strong>in</strong>g the cyclic sum <strong>and</strong> us<strong>in</strong>g xyz ≥ 1, we have<br />

∑<br />

cyclic<br />

x 5 − x 2<br />

x 5 + y 2 + z 2 ≥ 1<br />

x 5 + y 2 + z 2<br />

∑<br />

cyclic<br />

(<br />

x 2 − 1 )<br />

x<br />

≥<br />

1<br />

x 5 + y 2 + z 2<br />

∑ (<br />

x 2 − yz ) ≥ 0.<br />

cyclic<br />

Exercise 5. (USAMO Summer Program 2002) Let a, b, c be positive real numbers. Prove that<br />

(<br />

(H<strong>in</strong>t. [TJM]) Establish the <strong>in</strong>equality<br />

( ) 2 ( ) 2 ( ) 2<br />

2a<br />

3 2b<br />

3 2c<br />

3<br />

+ + ≥ 3.<br />

b + c c + a a + b<br />

2a<br />

b+c<br />

Exercise 6. (APMO 2005) (abc = 8, a, b, c > 0)<br />

) 2<br />

3<br />

≥ 3<br />

(<br />

a<br />

a+b+c<br />

a 2<br />

√<br />

(1 + a3 )(1 + b 3 ) + b 2<br />

√<br />

(1 + b3 )(1 + c 3 ) + c 2<br />

√<br />

(1 + c3 )(1 + a 3 ) ≥ 4 3<br />

)<br />

.<br />

(H<strong>in</strong>t.) Use the <strong>in</strong>equality<br />

1 √<br />

1+x 3 ≥ 2<br />

2+x 2 to give a lower bound <strong>of</strong> the left h<strong>and</strong> side.<br />

6 He received the special prize for this solution.<br />

25


Chapter 3<br />

Homogenizations <strong>and</strong> Normalizations<br />

Every Mathematician Has Only a Few Tricks. A long time ago an older <strong>and</strong> well-known number theorist made<br />

some disparag<strong>in</strong>g remarks about Paul Erdös ′ s work. You admire Erdos ′ s contributions to mathematics as much as I<br />

do, <strong>and</strong> I felt annoyed when the older mathematician flatly <strong>and</strong> def<strong>in</strong>itively stated that all <strong>of</strong> Erdos ′ s work could be<br />

reduced to a few tricks which Erdös repeatedly relied on <strong>in</strong> his pro<strong>of</strong>s. What the number theorist did not realize is<br />

that other mathematicians, even the very best, also rely on a few tricks which they use over <strong>and</strong> over. Take Hilbert.<br />

The second volume <strong>of</strong> Hilbert ′ s collected papers conta<strong>in</strong>s Hilbert ′ s papers <strong>in</strong> <strong>in</strong>variant theory. I have made a po<strong>in</strong>t <strong>of</strong><br />

read<strong>in</strong>g some <strong>of</strong> these papers with care. It is sad to note that some <strong>of</strong> Hilbert ′ s beautiful results have been completely<br />

forgotten. But on read<strong>in</strong>g the pro<strong>of</strong>s <strong>of</strong> Hilbert ′ s strik<strong>in</strong>g <strong>and</strong> deep theorems <strong>in</strong> <strong>in</strong>variant theory, it was surpris<strong>in</strong>g to<br />

verify that Hilbert ′ s pro<strong>of</strong>s relied on the same few tricks. Even Hilbert had only a few tricks! Gian-Carlo Rota,<br />

Ten Lessons I Wish I Had Been Taught, Notices <strong>of</strong> the AMS, January 1997<br />

3.1 Homogenizations<br />

Many <strong>in</strong>equality problems come with constra<strong>in</strong>ts such as ab = 1, xyz = 1, x+y +z = 1. A non-homogeneous<br />

symmetric <strong>in</strong>equality can be transformed <strong>in</strong>to a homogeneous one. Then we apply two powerful theorems :<br />

Shur’s <strong>in</strong>equality <strong>and</strong> Muirhead’s theorem. We beg<strong>in</strong> with a simple example.<br />

Problem 21. (Hungary 1996) Let a <strong>and</strong> b be positive real numbers with a + b = 1. Prove that<br />

a 2<br />

a + 1 +<br />

b2<br />

b + 1 ≥ 1 3 .<br />

Solution. Us<strong>in</strong>g the condition a + b = 1, we can reduce the given <strong>in</strong>equality to homogeneous one, i. e.,<br />

1<br />

3 ≤ a 2<br />

(a + b)(a + (a + b)) + b 2<br />

(a + b)(b + (a + b)) or a2 b + ab 2 ≤ a 3 + b 3 ,<br />

which follows from (a 3 +b 3 )−(a 2 b+ab 2 ) = (a−b) 2 (a+b) ≥ 0. The equality holds if <strong>and</strong> only if a = b = 1 2 .<br />

The above <strong>in</strong>equality a 2 b + ab 2 ≤ a 3 + b 3 can be generalized as follow<strong>in</strong>g :<br />

Theorem 3.1.1. Let a 1 , a 2 , b 1 , b 2 be positive real numbers such that a 1 + a 2 = b 1 + b 2 <strong>and</strong> max(a 1 , a 2 ) ≥<br />

max(b 1 , b 2 ). Let x <strong>and</strong> y be nonnegative real numbers. Then, we have x a1 y a2 + x a2 y a1 ≥ x b1 y b2 + x b2 y b1 .<br />

Pro<strong>of</strong>. Without loss <strong>of</strong> generality, we can assume that a 1 ≥ a 2 , b 1 ≥ b 2 , a 1 ≥ b 1 . If x or y is zero, then<br />

it clearly holds. So, we assume that both x <strong>and</strong> y are nonzero. It follows from a 1 + a 2 = b 1 + b 2 that<br />

a 1 − a 2 = (b 1 − a 2 ) + (b 2 − a 2 ). It’s easy to check<br />

x a 1<br />

y a 2<br />

+ x a 2<br />

y a 1<br />

− x b 1<br />

y b 2<br />

− x b 2<br />

y b 1<br />

= x a 2<br />

y a 2 ( x a 1−a 2<br />

+ y a 1−a 2<br />

− x b 1−a 2<br />

y b 2−a 2<br />

− x b 2−a 2<br />

y b 1−a 2<br />

)<br />

= x a2 y ( a2 x b1−a2 − y b1−a2) ( x b2−a2 − y b2−a2)<br />

1 (<br />

=<br />

x a 2 y<br />

a 2<br />

x<br />

b 1<br />

− y b ) ( 1<br />

x b 2<br />

− y b ) 2<br />

≥ 0.<br />

26


Remark 3.1.1. When does the equality hold <strong>in</strong> the theorem 8?<br />

We now <strong>in</strong>troduce two summation notations ∑ cyclic <strong>and</strong> ∑ sym<br />

. Let P (x, y, z) be a three variables function<br />

<strong>of</strong> x, y, z. Let us def<strong>in</strong>e :<br />

∑<br />

P (x, y, z) = P (x, y, z) + P (y, z, x) + P (z, x, y),<br />

cyclic<br />

∑<br />

P (x, y, z) = P (x, y, z) + P (x, z, y) + P (y, x, z) + P (y, z, x) + P (z, x, y) + P (z, y, x).<br />

sym<br />

For example, we know that<br />

∑<br />

cyclic<br />

x 3 y = x 3 y + y 3 z + z 3 x,<br />

∑<br />

x 3 = 2(x 3 + y 3 + z 3 )<br />

sym<br />

∑<br />

x 2 y = x 2 y + x 2 z + y 2 z + y 2 x + z 2 x + z 2 y,<br />

sym<br />

∑<br />

xyz = 6xyz.<br />

Problem 22. (IMO 1984/1) Let x, y, z be nonnegative real numbers such that x + y + z = 1. Prove that<br />

0 ≤ xy + yz + zx − 2xyz ≤ 7<br />

27 .<br />

Second Solution. Us<strong>in</strong>g the condition x + y + z = 1, we reduce the given <strong>in</strong>equality to homogeneous one, i.<br />

e.,<br />

0 ≤ (xy + yz + zx)(x + y + z) − 2xyz ≤ 7<br />

27 (x + y + z)3 .<br />

The left h<strong>and</strong> side <strong>in</strong>equality is trivial because it’s equivalent to<br />

0 ≤ xyz + ∑ sym<br />

x 2 y.<br />

sym<br />

The right h<strong>and</strong> side <strong>in</strong>equality simplifies to<br />

7 ∑<br />

In the view <strong>of</strong><br />

7 ∑<br />

cyclic<br />

x 3 + 15xyz − 6 ∑ sym<br />

cyclic<br />

x 3 + 15xyz − 6 ∑ sym<br />

⎛<br />

x 2 y = ⎝2 ∑<br />

cyclic<br />

x 3 − ∑ sym<br />

x 2 y ≥ 0.<br />

⎞ ⎛<br />

x 2 y⎠ + 5 ⎝3xyz + ∑<br />

cyclic<br />

x 3 − ∑ sym<br />

⎞<br />

x 2 y⎠ ,<br />

it’s enough to show that<br />

2 ∑<br />

x 3 ≥ ∑ x 2 y <strong>and</strong> 3xyz + ∑<br />

x 3 ≥ ∑ x 2 y.<br />

cyclic sym<br />

cyclic sym<br />

We note that<br />

2 ∑<br />

cyclic<br />

x 3 − ∑ sym<br />

x 2 y = ∑<br />

(x 3 + y 3 ) − ∑<br />

(x 2 y + xy 2 ) = ∑<br />

(x 3 + y 3 − x 2 y − xy 2 ) ≥ 0.<br />

cyclic<br />

The second <strong>in</strong>equality can be rewritten as<br />

∑<br />

cyclic<br />

cyclic<br />

x(x − y)(x − z) ≥ 0,<br />

which is a particular case <strong>of</strong> Schur’s theorem <strong>in</strong> the next section.<br />

cyclic<br />

After homogeniz<strong>in</strong>g, sometimes we can f<strong>in</strong>d the right approach to see the <strong>in</strong>equalities:<br />

27


(Iran 1998) Prove that, for all x, y, z > 1 such that 1 x + 1 y + 1 z = 2,<br />

√ x + y + z ≥<br />

√<br />

x − 1 +<br />

√<br />

y − 1 +<br />

√<br />

z − 1.<br />

Second Solution. After the algebraic substitution a = 1 x , b = 1 y , c = 1 z<br />

, we are required to prove that<br />

√<br />

1<br />

a + 1 b + 1 c ≥ √<br />

1 − a<br />

a<br />

√ √<br />

1 − b 1 − c<br />

+ + ,<br />

b c<br />

where a, b, c ∈ (0, 1) <strong>and</strong> a+b+c = 2. Us<strong>in</strong>g the constra<strong>in</strong>t a+b+c = 2, we obta<strong>in</strong> a homogeneous <strong>in</strong>equality<br />

or<br />

√<br />

√<br />

(<br />

1<br />

1<br />

2 (a + b + c) a + 1 b + 1 )<br />

≥<br />

c<br />

√<br />

a+b+c<br />

2<br />

− a<br />

+<br />

a<br />

√<br />

a+b+c<br />

2<br />

− b<br />

+<br />

b<br />

a+b+c<br />

2<br />

− c<br />

c<br />

√ ( 1<br />

(a + b + c)<br />

a + 1 b + 1 ) √ √ √<br />

b + c − a c + a − b a + b − c<br />

≥<br />

+<br />

+<br />

,<br />

c a<br />

b<br />

c<br />

which immediately follows from the Cauchy-Schwarz <strong>in</strong>equality<br />

√<br />

( 1<br />

[(b + c − a) + (c + a − b) + (a + b − c)]<br />

a + 1 b + 1 ) √ √ √<br />

b + c − a c + a − b a + b − c<br />

≥<br />

+<br />

+<br />

.<br />

c a<br />

b<br />

c<br />

28


3.2 Schur’s Inequality <strong>and</strong> Muirhead’s Theorem<br />

Theorem 3.2.1. (Schur) Let x, y, z be nonnegative real numbers. For any r > 0, we have<br />

∑<br />

x r (x − y)(x − z) ≥ 0.<br />

cyclic<br />

Pro<strong>of</strong>. S<strong>in</strong>ce the <strong>in</strong>equality is symmetric <strong>in</strong> the three variables, we may assume without loss <strong>of</strong> generality<br />

that x ≥ y ≥ z. Then the given <strong>in</strong>equality may be rewritten as<br />

(x − y)[x r (x − z) − y r (y − z)] + z r (x − z)(y − z) ≥ 0,<br />

<strong>and</strong> every term on the left-h<strong>and</strong> side is clearly nonnegative.<br />

Remark 3.2.1. When does the equality hold <strong>in</strong> Schur’s Inequality?<br />

Exercise 7. Disprove the follow<strong>in</strong>g proposition: For all a, b, c, d ≥ 0 <strong>and</strong> r > 0, we have<br />

a r (a − b)(a − c)(a − d) + b r (b − c)(b − d)(b − a) + c r (c − a)(c − c)(a − d) + d r (d − a)(d − b)(d − c) ≥ 0.<br />

The follow<strong>in</strong>g special case <strong>of</strong> Schur’s <strong>in</strong>equality is useful :<br />

∑<br />

x(x − y)(x − z) ≥ 0 ⇔ 3xyz + ∑<br />

cyclic<br />

cyclic<br />

x 3 ≥ ∑ sym<br />

x 2 y ⇔ ∑ sym<br />

xyz + ∑ sym<br />

x 3 ≥ 2 ∑ sym<br />

Corollary 3.2.1. Let x, y, z be nonnegative real numbers. Then, we have<br />

(<br />

)<br />

3xyz + x 3 + y 3 + z 3 ≥ 2 (xy) 3 3<br />

3<br />

2 + (yz) 2 + (zx) 2 .<br />

Pro<strong>of</strong>. By Schur’s <strong>in</strong>equality <strong>and</strong> the AM-GM <strong>in</strong>equality, we have<br />

3xyz + ∑<br />

x 3 ≥ ∑<br />

x 2 y + xy 2 ≥ ∑<br />

2(xy) 3 2 .<br />

cyclic<br />

cyclic<br />

cyclic<br />

x 2 y.<br />

We now use Schur’s <strong>in</strong>equality to give an alternative solution <strong>of</strong><br />

(APMO 2004/5) Prove that, for all positive real numbers a, b, c,<br />

Second Solution. After exp<strong>and</strong><strong>in</strong>g, it becomes<br />

8 + (abc) 2 + 2 ∑<br />

(a 2 + 2)(b 2 + 2)(c 2 + 2) ≥ 9(ab + bc + ca).<br />

cyclic<br />

a 2 b 2 + 4 ∑<br />

a 2 ≥ 9 ∑<br />

ab.<br />

cyclic<br />

From the <strong>in</strong>equality (ab − 1) 2 + (bc − 1) 2 + (ca − 1) 2 ≥ 0, we obta<strong>in</strong><br />

6 + 2 ∑<br />

a 2 b 2 ≥ 4 ∑<br />

ab.<br />

Hence, it will be enough to show that<br />

cyclic<br />

cyclic<br />

2 + (abc) 2 + 4 ∑<br />

a 2 ≥ 5 ∑<br />

ab.<br />

cyclic<br />

cyclic<br />

S<strong>in</strong>ce 3(a 2 + b 2 + c 2 ) ≥ 3(ab + bc + ca), it will be enough to show that<br />

2 + (abc) 2 + ∑<br />

a 2 ≥ 2 ∑<br />

ab,<br />

cyclic<br />

which is a particular case <strong>of</strong> the follow<strong>in</strong>g result for t = 1.<br />

cyclic<br />

cyclic<br />

29


Corollary 3.2.2. Let t ∈ (0, 3]. For all a, b, c ≥ 0, we have<br />

∑<br />

(3 − t) + t(abc) 2 t + a 2 ≥ 2 ∑<br />

ab.<br />

In particular, we obta<strong>in</strong> non-homogeneous <strong>in</strong>equalities<br />

cyclic<br />

cyclic<br />

5<br />

2 + 1 2 (abc)4 + a 2 + b 2 + c 2 ≥ 2(ab + bc + ca),<br />

2 + (abc) 2 + a 2 + b 2 + c 2 ≥ 2(ab + bc + ca),<br />

1 + 2abc + a 2 + b 2 + c 2 ≥ 2(ab + bc + ca).<br />

Pro<strong>of</strong>. After sett<strong>in</strong>g x = a 2 3 , y = b 2 3 , z = c 2 3 , it becomes<br />

3 − t + t(xyz) 3 t +<br />

∑<br />

By the corollary 1, it will be enough to show that<br />

cyclic<br />

x 3 ≥ 2 ∑<br />

(xy) 3 2 .<br />

cyclic<br />

3 − t + t(xyz) 3 t ≥ 3xyz,<br />

which is a straightforward consequence <strong>of</strong> the weighted AM-GM <strong>in</strong>equality :<br />

3 − t<br />

3<br />

· 1 + t 3 (xyz) 3 3−t<br />

t ≥ 1 3<br />

One may check that the equality holds if <strong>and</strong> only if a = b = c = 1.<br />

( )<br />

(xyz) 3 t<br />

3<br />

t = 3xyz.<br />

(IMO 2000/2) Let a, b, c be positive numbers such that abc = 1. Prove that<br />

(<br />

a − 1 + 1 ) (<br />

b − 1 + 1 ) (<br />

c − 1 + 1 )<br />

≤ 1.<br />

b<br />

c<br />

a<br />

Second Solution. It is equivalent to the follow<strong>in</strong>g homogeneous <strong>in</strong>equality 1 :<br />

(a − (abc) 1/3 + (abc)2/3<br />

b<br />

) (b − (abc) 1/3 + (abc)2/3<br />

c<br />

) )<br />

(c − (abc) 1/3 + (abc)2/3 ≤ abc.<br />

a<br />

After the substitution a = x 3 , b = y 3 , c = z 3 with x, y, z > 0, it becomes<br />

) (<br />

) (<br />

)<br />

(x 3 − xyz + (xyz)2<br />

y 3 y 3 − xyz + (xyz)2<br />

z 3 z 3 − xyz + (xyz)2<br />

x 3 ≤ x 3 y 3 z 3 ,<br />

which simplifies to<br />

(<br />

x 2 y − y 2 z + z 2 x ) ( y 2 z − z 2 x + x 2 y ) ( z 2 x − x 2 y + y 2 z ) ≤ x 3 y 3 z 3<br />

or<br />

3x 3 y 3 z 3 + ∑<br />

x 6 y 3 ≥ ∑<br />

x 4 y 4 z + ∑<br />

x 5 y 2 z 2<br />

cyclic cyclic<br />

cyclic<br />

or<br />

3(x 2 y)(y 2 z)(z 2 x) +<br />

cyclic(x ∑ 2 y) 3 ≥ ∑ (x 2 y) 2 (y 2 z)<br />

sym<br />

which is a special case <strong>of</strong> Schur’s <strong>in</strong>equality.<br />

Here is another <strong>in</strong>equality problem with the constra<strong>in</strong>t abc = 1.<br />

1 For an alternative homogenization, see the problem 1 <strong>in</strong> the chapter 2.<br />

30


Problem 23. (Tournament <strong>of</strong> Towns 1997) Let a, b, c be positive numbers such that abc = 1. Prove that<br />

1<br />

a + b + 1 + 1<br />

b + c + 1 + 1<br />

c + a + 1 ≤ 1.<br />

Solution. We can rewrite the given <strong>in</strong>equality as follow<strong>in</strong>g :<br />

1<br />

a + b + (abc) + 1<br />

1/3 b + c + (abc) + 1<br />

1/3 c + a + (abc) ≤ 1<br />

1/3 (abc) . 1/3<br />

We make the substitution a = x 3 , b = y 3 , c = z 3 with x, y, z > 0. Then, it becomes<br />

1<br />

x 3 + y 3 + xyz + 1<br />

y 3 + z 3 + xyz + 1<br />

z 3 + x 3 + xyz ≤ 1<br />

xyz<br />

which is equivalent to<br />

xyz ∑<br />

(x 3 + y 3 + xyz)(y 3 + z 3 + xyz) ≤ (x 3 + y 3 + xyz)(y 3 + z 3 + xyz)(z 3 + x 3 + xyz)<br />

or<br />

cyclic<br />

We apply the theorem 9 to obta<strong>in</strong><br />

∑<br />

sym<br />

x 6 y 3 ≥ ∑ sym<br />

x 5 y 2 z 2 !<br />

∑<br />

x 6 y 3 = ∑<br />

x 6 y 3 + y 6 x 3<br />

sym<br />

≥<br />

cyclic<br />

∑<br />

x 5 y 4 + y 5 x 4<br />

cyclic<br />

= ∑<br />

x 5 (y 4 + z 4 )<br />

≥<br />

cyclic<br />

∑<br />

x 5 (y 2 z 2 + y 2 z 2 )<br />

cyclic<br />

= ∑ sym<br />

x 5 y 2 z 2 .<br />

Exercise 8. ([TZ], pp.142) Prove that for any acute triangle ABC,<br />

cot 3 A + cot 3 B + cot 3 C + 6 cot A cot B cot C ≥ cot A + cot B + cot C.<br />

Exercise 9. (Korea 1998) Let I be the <strong>in</strong>center <strong>of</strong> a triangle ABC. Prove that<br />

IA 2 + IB 2 + IC 2 ≥ BC2 + CA 2 + AB 2<br />

.<br />

3<br />

Exercise 10. ([IN], pp.103) Let a, b, c be the lengths <strong>of</strong> a triangle. Prove that<br />

a 2 b + a 2 c + b 2 c + b 2 a + c 2 a + c 2 b > a 3 + b 3 + c 3 + 2abc.<br />

Exercise 11. (Surányi’s <strong>in</strong>equality)) Show that, for all x 1 , · · · , x n ≥ 0,<br />

(n − 1) (x n 1 + · · · x n n ) + nx 1 · · · x n ≥ (x 1 + · · · x n ) ( x n−1 n−1<br />

1 + · · · x ) n .<br />

Theorem 3.2.2. (Muirhead) Let a 1 , a 2 , a 3 , b 1 , b 2 , b 3 be real numbers such that<br />

a 1 ≥ a 2 ≥ a 3 ≥ 0, b 1 ≥ b 2 ≥ b 3 ≥ 0, a 1 ≥ b 1 , a 1 + a 2 ≥ b 1 + b 2 , a 1 + a 2 + a 3 = b 1 + b 2 + b 3 .<br />

Let x, y, z be positive real numbers. Then, we have ∑ sym xa 1<br />

y a 2<br />

z a 3<br />

≥ ∑ sym xb 1<br />

y b 2<br />

z b 3<br />

.<br />

31


Pro<strong>of</strong>. Case 1. b 1 ≥ a 2 : It follows from a 1 ≥ a 1 +a 2 −b 1 <strong>and</strong> from a 1 ≥ b 1 that a 1 ≥ max(a 1 +a 2 −b 1 , b 1 ) so<br />

that max(a 1 , a 2 ) = a 1 ≥ max(a 1 +a 2 −b 1 , b 1 ). From a 1 +a 2 −b 1 ≥ b 1 +a 3 −b 1 = a 3 <strong>and</strong> a 1 +a 2 −b 1 ≥ b 2 ≥ b 3 ,<br />

we have max(a 1 + a 2 − b 1 , a 3 ) ≥ max(b 2 , b 3 ). Apply the theorem 8 twice to obta<strong>in</strong><br />

∑<br />

x a 1<br />

y a 2<br />

z a 3<br />

= ∑<br />

z a 3<br />

(x a 1<br />

y a 2<br />

+ x a 2<br />

y a 1<br />

)<br />

sym<br />

≥<br />

cyclic<br />

∑<br />

z a3 (x a1+a2−b1 y b1 + x b1 y a1+a2−b1 )<br />

cyclic<br />

= ∑<br />

x b 1<br />

(y a 1+a 2 −b 1<br />

z a 3<br />

+ y a 3<br />

z a 1+a 2 −b 1<br />

)<br />

≥<br />

cyclic<br />

∑<br />

x b 1<br />

(y b 2<br />

z b 3<br />

+ y b 3<br />

z b 2<br />

)<br />

cyclic<br />

= ∑ sym<br />

x b1 y b2 z b3 .<br />

Case 2. b 1 ≤ a 2 : It follows from 3b 1 ≥ b 1 + b 2 + b 3 = a 1 + a 2 + a 3 ≥ b 1 + a 2 + a 3 that b 1 ≥ a 2 + a 3 − b 1<br />

<strong>and</strong> that a 1 ≥ a 2 ≥ b 1 ≥ a 2 + a 3 − b 1 . Therefore, we have max(a 2 , a 3 ) ≥ max(b 1 , a 2 + a 3 − b 1 ) <strong>and</strong><br />

max(a 1 , a 2 + a 3 − b 1 ) ≥ max(b 2 , b 3 ). Apply the theorem 8 twice to obta<strong>in</strong><br />

∑<br />

x a 1<br />

y a 2<br />

z a 3<br />

= ∑<br />

x a 1<br />

(y a 2<br />

z a 3<br />

+ y a 3<br />

z a 2<br />

)<br />

sym<br />

≥<br />

cyclic<br />

∑<br />

x a1 (y b1 z a2+a3−b1 + y a2+a3−b1 z b1 )<br />

cyclic<br />

= ∑<br />

y b1 (x a1 z a2+a3−b1 + x a2+a3−b1 z a1 )<br />

≥<br />

cyclic<br />

∑<br />

y b 1<br />

(x b 2<br />

z b 3<br />

+ x b 3<br />

z b 2<br />

)<br />

cyclic<br />

= ∑ sym<br />

x b1 y b2 z b3 .<br />

Remark 3.2.2. The equality holds if <strong>and</strong> only if x = y = z. However, if we allow x = 0 or y = 0 or z = 0,<br />

then one may easily check that the equality holds when a 1 , a 2 , a 3 > 0 <strong>and</strong> b 1 , b 2 , b 3 > 0 if <strong>and</strong> only if<br />

x = y = z or x = y, z = 0 or y = z, x = 0 or z = x, y = 0.<br />

We can use Muirhead’s theorem to prove Nesbitt’s <strong>in</strong>equality.<br />

(Nesbitt) For all positive real numbers a, b, c, we have<br />

a<br />

b + c +<br />

b<br />

c + a +<br />

c<br />

a + b ≥ 3 2 .<br />

Pro<strong>of</strong> 6. Clear<strong>in</strong>g the denom<strong>in</strong>ators <strong>of</strong> the <strong>in</strong>equality, it becomes<br />

2 ∑<br />

cyclic<br />

a(a + b)(a + c) ≥ 3(a + b)(b + c)(c + a) or ∑ sym<br />

a 3 ≥ ∑ sym<br />

(IMO 1995) Let a, b, c be positive numbers such that abc = 1. Prove that<br />

1<br />

a 3 (b + c) + 1<br />

b 3 (c + a) + 1<br />

c 3 (a + b) ≥ 3 2 .<br />

a 2 b.<br />

32


Second Solution. It’s equivalent to<br />

1<br />

a 3 (b + c) + 1<br />

b 3 (c + a) + 1<br />

c 3 (a + b) ≥ 3<br />

2(abc) . 4/3<br />

Set a = x 3 , b = y 3 , c = z 3 with x, y, z > 0. Then, it becomes ∑ cyclic<br />

denom<strong>in</strong>ators, this becomes<br />

∑<br />

or<br />

sym<br />

x 12 y 12 + 2 ∑ sym<br />

x 12 y 9 z 3 + ∑ sym<br />

x 9 y 9 z 6 ≥ 3 ∑ sym<br />

1<br />

3<br />

x 9 (y 3 +z 3 )<br />

≥<br />

2x 4 y 4 z<br />

. 4<br />

x 11 y 8 z 5 + 6x 8 y 8 z 8<br />

Clear<strong>in</strong>g<br />

( ∑<br />

x 12 y 12 − ∑ )<br />

x 11 y 8 z 5 + 2<br />

sym<br />

sym<br />

x 12 y 9 z 3 − ∑ sym<br />

x 11 y 8 z 5 )<br />

+<br />

x 9 y 9 z 6 − ∑ sym<br />

x 8 y 8 z 8 )<br />

≥ 0,<br />

( ∑<br />

sym<br />

( ∑<br />

sym<br />

<strong>and</strong> every term on the left h<strong>and</strong> side is nonnegative by Muirhead’s theorem.<br />

Problem 24. (Iran 1996) Let x, y, z be positive real numbers. Prove that<br />

(<br />

)<br />

1<br />

(xy + yz + zx)<br />

(x + y) 2 + 1<br />

(y + z) 2 + 1<br />

(z + x) 2 ≥ 9 4 .<br />

Pro<strong>of</strong>. It’s equivalent to<br />

4 ∑ x 5 y + 2 ∑<br />

x 4 yz + 6x 2 y 2 z 2 − ∑ x 4 y 2 − 6 ∑<br />

x 3 y 3 − 2 ∑ x 3 y 2 z ≥ 0.<br />

sym<br />

cyclic sym<br />

cyclic sym<br />

We rewrite this as follow<strong>in</strong>g<br />

( ∑<br />

x 5 y − ∑ ) ( ∑<br />

x 4 y 2 + 3 x 5 y − ∑ ) ⎛<br />

⎞<br />

x 3 y 3 + 2xyz ⎝3xyz + ∑<br />

x 3 − ∑ x 2 y⎠ ≥ 0.<br />

sym sym<br />

sym sym<br />

cyclic sym<br />

By Muirhead’s theorem <strong>and</strong> Schur’s <strong>in</strong>equality, it’s a sum <strong>of</strong> three nonnegative terms.<br />

Problem 25. Let x, y, z be nonnegative real numbers with xy + yz + zx = 1. Prove that<br />

1<br />

x + y + 1<br />

y + z + 1<br />

z + x ≥ 5 2 .<br />

Pro<strong>of</strong>. Us<strong>in</strong>g xy + yz + zx = 1, we homogenize the given <strong>in</strong>equality as follow<strong>in</strong>g :<br />

or<br />

or<br />

( 1<br />

(xy + yz + zx)<br />

x + y + 1<br />

y + z + 1 ) 2 ( 2 5<br />

≥<br />

z + x 2)<br />

4 ∑ x 5 y + ∑ x 4 yz + 14 ∑ x 3 y 2 z + 38x 2 y 2 z 2 ≥ ∑ x 4 y 2 + 3 ∑ x 3 y 3<br />

sym sym<br />

sym<br />

sym<br />

sym<br />

( ∑<br />

x 5 y − ∑ ) ( ∑<br />

x 4 y 2 + 3 x 5 y − ∑ ) ( ∑<br />

x 3 y 3 + xyz x 3 + 14 ∑ )<br />

x 2 y + 38xyz ≥ 0.<br />

sym sym<br />

sym sym<br />

sym<br />

sym<br />

By Muirhead’s theorem, we get the result. In the above <strong>in</strong>equality, without the condition xy + yz + zx = 1,<br />

the equality holds if <strong>and</strong> only if x = y, z = 0 or y = z, x = 0 or z = x, y = 0. S<strong>in</strong>ce xy + yz + zx = 1, the<br />

equality occurs when (x, y, z) = (1, 1, 0), (1, 0, 1), (0, 1, 1).<br />

33


3.3 Normalizations<br />

In the previous sections, we transformed non-homogeneous <strong>in</strong>equalities <strong>in</strong>to homogeneous ones. On the other<br />

h<strong>and</strong>, homogeneous <strong>in</strong>equalities also can be normalized <strong>in</strong> various ways. We <strong>of</strong>fer two alternative solutions<br />

<strong>of</strong> the problem 8 by normalizations :<br />

(IMO 2001/2) Let a, b, c be positive real numbers. Prove that<br />

a<br />

√<br />

a2 + 8bc +<br />

b<br />

√<br />

b2 + 8ca +<br />

c<br />

√<br />

c2 + 8ab ≥ 1.<br />

Third Solution. We make the substitution x =<br />

a<br />

a+b+c , y =<br />

b<br />

a+b+c , z =<br />

xf(x 2 + 8yz) + yf(y 2 + 8zx) + zf(z 2 + 8xy) ≥ 1,<br />

c<br />

a+b+c .2 The problem is<br />

where f(t) = 1 √<br />

t<br />

. S<strong>in</strong>ce f is convex on R + <strong>and</strong> x + y + z = 1, we apply (the weighted) Jensen’s <strong>in</strong>equality<br />

to obta<strong>in</strong><br />

xf(x 2 + 8yz) + yf(y 2 + 8zx) + zf(z 2 + 8xy) ≥ f(x(x 2 + 8yz) + y(y 2 + 8zx) + z(z 2 + 8xy)).<br />

Note that f(1) = 1. S<strong>in</strong>ce the function f is strictly decreas<strong>in</strong>g, it suffices to show that<br />

1 ≥ x(x 2 + 8yz) + y(y 2 + 8zx) + z(z 2 + 8xy).<br />

Us<strong>in</strong>g x + y + z = 1, we homogenize it as (x + y + z) 3 ≥ x(x 2 + 8yz) + y(y 2 + 8zx) + z(z 2 + 8xy). However,<br />

this is easily seen from<br />

(x + y + z) 3 − x(x 2 + 8yz) − y(y 2 + 8zx) − z(z 2 + 8xy) = 3[x(y − z) 2 + y(z − x) 2 + z(x − y) 2 ] ≥ 0.<br />

In the above solution, we normalized to x + y + z = 1. We now prove it by normaliz<strong>in</strong>g to xyz = 1.<br />

Fourth Solution. We make the substitution x = bc<br />

a<br />

, y = ca<br />

2 b<br />

, z = ab<br />

2 c<br />

. Then, we get xyz = 1 <strong>and</strong> the <strong>in</strong>equality<br />

2<br />

becomes<br />

1 1 1<br />

√ + √ + √ ≥ 1<br />

1 + 8x 1 + 8y 1 + 8z<br />

which is equivalent to<br />

∑<br />

√<br />

(1 + 8x)(1 + 8y) ≥<br />

√<br />

(1 + 8x)(1 + 8y)(1 + 8z).<br />

cyclic<br />

After squar<strong>in</strong>g both sides, it’s equivalent to<br />

8(x + y + z) + 2 √ (1 + 8x)(1 + 8y)(1 + 8z) ∑ √<br />

1 + 8x ≥ 510.<br />

Recall that xyz = 1. The AM-GM <strong>in</strong>equality gives us x + y + z ≥ 3,<br />

(1 + 8x)(1 + 8y)(1 + 8z) ≥ 9x 8 9 · 9y<br />

8<br />

9 · 9z<br />

8<br />

9 = 729 <strong>and</strong><br />

∑<br />

Us<strong>in</strong>g these three <strong>in</strong>equalities, we get the result.<br />

cyclic<br />

cyclic<br />

√<br />

1 + 8x ≥<br />

∑<br />

cyclic<br />

(IMO 1983/6) Let a, b, c be the lengths <strong>of</strong> the sides <strong>of</strong> a triangle. Prove that<br />

a 2 b(a − b) + b 2 c(b − c) + c 2 a(c − a) ≥ 0.<br />

2 Divid<strong>in</strong>g by a + b + c gives the equivalent <strong>in</strong>equalityPcyclic<br />

a<br />

r<br />

a+b+c<br />

a 2<br />

(a+b+c) 2 + 8bc<br />

(a+b+c) 2 ≥ 1.<br />

√<br />

9x 8 9 ≥ 9(xyz) 4<br />

27 = 9.<br />

34


Second Solution. After sett<strong>in</strong>g a = y + z, b = z + x, c = x + y for x, y, z > 0, it becomes<br />

x 3 z + y 3 x + z 3 y ≥ x 2 yz + xy 2 z + xyz 2<br />

or x2<br />

y + y2<br />

z + z2<br />

x ≥ x + y + z.<br />

S<strong>in</strong>ce it’s homogeneous, we can restrict our attention to the case x + y + z = 1. Then, it becomes<br />

( ) x<br />

( y<br />

) ( z<br />

yf + zf + xf ≥ 1,<br />

y z x)<br />

where f(t) = t 2 . S<strong>in</strong>ce f is convex on R, we apply (the weighted) Jensen’s <strong>in</strong>equality to obta<strong>in</strong><br />

( ) x<br />

( y<br />

) ( ( z<br />

yf + zf + xf ≥ f y ·<br />

y z x) x<br />

y + z · y<br />

z + x · z )<br />

= f(1) = 1.<br />

x<br />

Problem 26. (KMO W<strong>in</strong>ter Program Test 2001) Prove that, for all a, b, c > 0,<br />

√<br />

(a2 b + b 2 c + c 2 a) (ab 2 + bc 2 + ca 2 ) ≥ abc + 3√ (a 3 + abc) (b 3 + abc) (c 3 + abc)<br />

First Solution. Divid<strong>in</strong>g by abc, it becomes<br />

√ (a<br />

c + b a + c ) ( √ (a c<br />

b a + a b c)<br />

+ b ) ( ) ( )<br />

2 b<br />

≥ abc + 3 bc + 1 2 c<br />

ca + 1 2<br />

ab + 1 .<br />

After the substitution x = a b , y = b c , z = c a<br />

, we obta<strong>in</strong> the constra<strong>in</strong>t xyz = 1. It takes the form<br />

√<br />

√ (x ) (<br />

(x + y + z) (xy + yz + zx) ≥ 1 +<br />

3<br />

y<br />

) ( )<br />

z + 1 x + 1 z<br />

y + 1 .<br />

From the constra<strong>in</strong>t xyz = 1, we f<strong>in</strong>d two identities<br />

( x<br />

) ( y<br />

) ( ) ( ) ( ) ( )<br />

z + 1 x + 1 z x + z y + x z + y<br />

y + 1 =<br />

= (z + x)(x + y)(y + z),<br />

z x y<br />

(x + y + z) (xy + yz + zx) = (x + y)(y + z)(z + x) + xyz = (x + y)(y + z)(z + x) + 1.<br />

Lett<strong>in</strong>g p = 3√ (x + y)(y + z)(z + x), the <strong>in</strong>equality now becomes √ p 3 + 1 ≥ 1 + p. Apply<strong>in</strong>g the AM-GM<br />

<strong>in</strong>equality, we have p ≥ 3 √2 √ xy · 2 √ yz · 2 √ zx = 2. It follows that (p 3 +1)−(1+p) 2 = p(p+1)(p−2) ≥ 0.<br />

Problem 27. (IMO 1999/2) Let n be an <strong>in</strong>teger with n ≥ 2.<br />

(a) Determ<strong>in</strong>e the least constant C such that the <strong>in</strong>equality<br />

∑<br />

1≤i 0. S<strong>in</strong>ce the <strong>in</strong>equality is homogeneous, we may normalize to x 1 +· · ·+x n = 1.<br />

We denote<br />

F (x 1 , · · · , x n ) =<br />

∑<br />

x i x j (x 2 i + x 2 j).<br />

3 I slightly modified his solution <strong>in</strong> [Au99].<br />

1≤i


From the assumption x 1 + · · · + x n = 1, we have<br />

F (x 1 , · · · , x n ) =<br />

∑<br />

x 3 i x j +<br />

∑<br />

1≤i


Pro<strong>of</strong> 8. (Cao M<strong>in</strong>h Quang) Assume that a + b + c = 1. Note that ab + bc + ca ≤ 1 3 (a + b + c)2 = 1 3 . More<br />

strongly, we establish that<br />

a<br />

b + c + b<br />

c + a + c<br />

a + b ≥ 3 − 9 (ab + bc + ca)<br />

2<br />

or ( ) ( ) ( )<br />

a 9a(b + c) b 9b(c + a) c 9c(a + b)<br />

+ + + + + ≥ 3.<br />

b + c 4 c + a 4 a + b 4<br />

The AM-GM <strong>in</strong>equality shows that<br />

∑<br />

cyclic<br />

a 9a(b + c)<br />

+ ≥ ∑<br />

2<br />

b + c 4<br />

cyclic<br />

√<br />

a 9a(b + c) · = ∑<br />

3a = 3.<br />

b + c 4<br />

cyclic<br />

Pro<strong>of</strong> 9. We now break the symmetry by a suitable normalization. S<strong>in</strong>ce the <strong>in</strong>equality is symmetric <strong>in</strong> the<br />

three variables, we may assume that a ≥ b ≥ c. After the substitution x = a c , y = b c<br />

, we have x ≥ y ≥ 1. It<br />

becomes<br />

a b<br />

c<br />

b<br />

c + 1 + c<br />

a<br />

c + 1 + 1<br />

a<br />

c + b ≥ 3 2 or x<br />

y + 1 + y<br />

x + 1 ≥ 3 2 − 1<br />

x + y .<br />

c<br />

We apply the AM-GM <strong>in</strong>equality to obta<strong>in</strong><br />

It’s enough to show that<br />

x + 1<br />

y + 1 + y + 1<br />

x + 1 ≥ 2 or x<br />

y + 1 + y<br />

x + 1 ≥ 2 − 1<br />

y + 1 + 1<br />

x + 1 .<br />

2 − 1<br />

y + 1 + 1<br />

x + 1 ≥ 3 2 − 1<br />

x + y<br />

⇔ 1 2 − 1<br />

y + 1 ≥ 1<br />

x + 1 − 1<br />

x + y<br />

⇔ y − 1<br />

2(1 + y) ≥ y − 1<br />

(x + 1)(x + y) .<br />

However, the last <strong>in</strong>equality clearly holds for x ≥ y ≥ 1.<br />

Pro<strong>of</strong> 10. As <strong>in</strong> the previous pro<strong>of</strong>, we may normalize to c = 1 with the assumption a ≥ b ≥ 1. We prove<br />

Let A = a + b <strong>and</strong> B = ab. It becomes<br />

a<br />

b + 1 + b<br />

a + 1 + 1<br />

a + b ≥ 3 2 .<br />

a 2 + b 2 + a + b<br />

(a + 1)(b + 1) + 1<br />

a + b ≥ 3 2 or A2 − 2B + A<br />

A + B + 1 + 1 A ≥ 3 2 or 2A3 − A 2 − A + 2 ≥ B(7A − 2).<br />

S<strong>in</strong>ce 7A − 2 > 2(a + b − 1) > 0 <strong>and</strong> A 2 = (a + b) 2 ≥ 4ab = 4B, it’s enough to show that<br />

4(2A 3 − A 2 − A + 2) ≥ A 2 (7A − 2) ⇔ A 3 − 2A 2 − 4A + 8 ≥ 0.<br />

However, it’s easy to check that A 3 − 2A 2 − 4A + 8 = (A − 2) 2 (A + 2) ≥ 0.<br />

37


3.4 Cauchy-Schwarz Inequality <strong>and</strong> Hölder’s Inequality<br />

We beg<strong>in</strong> with the follow<strong>in</strong>g famous theorem:<br />

Theorem 3.4.1. (The Cauchy-Schwarz <strong>in</strong>equality) Let a 1 , · · · , a n , b 1 , · · · , b n be real numbers. Then,<br />

(a 1 2 + · · · + a n 2 )(b 1 2 + · · · + b n 2 ) ≥ (a 1 b 1 + · · · + a n b n ) 2 .<br />

Pro<strong>of</strong>. Let A = √ a 12 + · · · + a n2 <strong>and</strong> B = √ b 1 2 + · · · + b n 2 . In the case when A = 0, we get a 1 = · · · =<br />

a n = 0. Thus, the given <strong>in</strong>equality clearly holds. So, we may assume that A, B > 0. We may normalize to<br />

Hence, we need to to show that<br />

We now apply the AM-GM <strong>in</strong>equality to deduce<br />

1 = a 1 2 + · · · + a n 2 = b 1 2 + · · · + b n 2 .<br />

|a 1 b 1 + · · · + a n b n | ≤ 1.<br />

|x 1 y 1 + · · · + x n y n | ≤ |x 1 y 1 | + · · · + |x n y n | ≤ x 1 2 + y 1<br />

2<br />

2<br />

+ · · · + x n 2 + y n<br />

2<br />

2<br />

= 1.<br />

Exercise 13. Prove the Lagrange identity :<br />

( n<br />

∑<br />

i=1<br />

a i<br />

2<br />

) ( n<br />

∑<br />

i=1<br />

b i<br />

2<br />

) ( n<br />

) 2<br />

∑<br />

− a i b i =<br />

i=1<br />

∑<br />

1≤i a k b k − a k b k<br />

4<br />

k=1 k=1<br />

k=1<br />

k=1<br />

Exercise 15. ([PF], S. S. Wagner) Let a 1 , · · · , a n , b 1 , · · · , b n be real numbers. Suppose that x ∈ [0, 1].<br />

Show that ⎛<br />

⎞ ⎛<br />

⎞ ⎛<br />

⎞<br />

n∑<br />

⎝ a 2 i + 2x ∑ n∑<br />

a i a j<br />

⎠ ⎝ b 2 i + 2x ∑ n∑<br />

b i b j<br />

⎠ ≥ ⎝ a i b i + x ∑ 2<br />

a i b j<br />

⎠ .<br />

i=1 i


(Iran 1998) Prove that, for all x, y, z > 1 such that 1 x + 1 y + 1 z = 2,<br />

√ x + y + z ≥<br />

√<br />

x − 1 +<br />

√<br />

y − 1 +<br />

√<br />

z − 1.<br />

Third Solution. We note that x−1<br />

x<br />

√<br />

√ x + y + z =<br />

+ y−1<br />

y<br />

+ z−1<br />

z<br />

( x − 1<br />

(x + y + z)<br />

x<br />

= 1. Apply the Cauchy-Schwarz <strong>in</strong>equality to deduce<br />

+ y − 1<br />

y<br />

+ z − 1 )<br />

≥ √ x − 1 + √ y − 1 + √ z − 1.<br />

z<br />

We now apply the Cauchy-Schwarz <strong>in</strong>equality to prove Nesbitt’s <strong>in</strong>equality.<br />

(Nesbitt) For all positive real numbers a, b, c, we have<br />

a<br />

b + c +<br />

b<br />

c + a +<br />

c<br />

a + b ≥ 3 2 .<br />

Pro<strong>of</strong> 11. Apply<strong>in</strong>g the Cauchy-Schwarz <strong>in</strong>equality, we have<br />

( 1<br />

((b + c) + (c + a) + (a + b))<br />

b + c + 1<br />

c + a + 1 )<br />

≥ 3 2 .<br />

a + b<br />

It follows that<br />

a + b + c<br />

b + c<br />

+ a + b + c<br />

c + a<br />

+ a + b + c<br />

a + b<br />

Pro<strong>of</strong> 12. The Cauchy-Schwarz <strong>in</strong>equality yields<br />

∑<br />

cyclic<br />

a<br />

b + c<br />

⎛ ⎞<br />

∑<br />

a(b + c) ≥ ⎝ ∑<br />

a⎠<br />

cyclic<br />

cyclic<br />

2<br />

or<br />

≥ 9 2 or 3 + ∑<br />

∑<br />

cyclic<br />

cyclic<br />

a<br />

b + c ≥ 9 2 .<br />

a (a + b + c)2<br />

≥<br />

b + c 2(ab + bc + ca) ≥ 3 2 .<br />

Problem 28. (Gazeta Matematicã) Prove that, for all a, b, c > 0,<br />

√<br />

a4 + a 2 b 2 + b 4 + √ b 4 + b 2 c 2 + c 4 + √ c 4 + c 2 a 2 + a 4 ≥ a √ 2a 2 + bc + b √ 2b 2 + ca + c √ 2c 2 + ab.<br />

Solution. We obta<strong>in</strong> the cha<strong>in</strong> <strong>of</strong> equalities <strong>and</strong> <strong>in</strong>equalities<br />

∑ √<br />

a4 + a 2 b 2 + b 4 = ∑<br />

√ ( ) ( )<br />

a 4 + a2 b 2<br />

+ b<br />

2<br />

4 + a2 b 2<br />

2<br />

cyclic<br />

cyclic<br />

(√<br />

≥ √ 1<br />

√ )<br />

∑<br />

a 4 + a2 b 2<br />

+ b 2 2<br />

4 + a2 b 2<br />

2<br />

cyclic<br />

(√<br />

= √ 1<br />

√ )<br />

∑<br />

a 4 + a2 b 2<br />

+ a 2 2<br />

4 + a2 c 2<br />

2<br />

cyclic<br />

≥<br />

√ 2 ∑<br />

√ ( ) ( )<br />

4<br />

a 4 + a2 b 2<br />

a<br />

2<br />

4 + a2 c 2<br />

2<br />

cyclic<br />

≥<br />

√ 2 ∑ √<br />

a 4 + a2 bc<br />

2<br />

cyclic<br />

= ∑ √<br />

2a4 + a 2 bc .<br />

cyclic<br />

(Cauchy − Schwarz)<br />

(AM − GM)<br />

(Cauchy − Schwarz)<br />

39


Here is an <strong>in</strong>genious solution <strong>of</strong><br />

(KMO W<strong>in</strong>ter Program Test 2001) Prove that, for all a, b, c > 0,<br />

√<br />

(a2 b + b 2 c + c 2 a) (ab 2 + bc 2 + ca 2 ) ≥ abc + 3√ (a 3 + abc) (b 3 + abc) (c 3 + abc)<br />

Second Solution. (based on work by an w<strong>in</strong>ter program participant) We obta<strong>in</strong><br />

√<br />

(a2 b + b 2 c + c 2 a) (ab 2 + bc 2 + ca 2 )<br />

=<br />

2√ 1 [b(a2 + bc) + c(b 2 + ca) + a(c 2 + ab)] [c(a 2 + bc) + a(b 2 + ca) + b(c 2 + ab)]<br />

≥ 1 (√<br />

bc(a 2 + bc) + √ ca(b 2 + ca) + √ )<br />

ab(c 2 + ab)<br />

(Cauchy − Schwarz)<br />

2<br />

≥ 3 2<br />

√<br />

3 √bc(a2<br />

+ bc) · √ca(b 2 + ca) · √ab(c 2 + ab)<br />

(AM − GM)<br />

= 1 √<br />

3<br />

(a3 + abc) (b<br />

2<br />

3 + abc) (c 3 + abc) + 3√ (a 3 + abc) (b 3 + abc) (c 3 + abc)<br />

≥ 1 √<br />

3<br />

2 √ a<br />

2<br />

3 · abc · 2 √ b 3 · abc · 2 √ c 3 · abc + 3√ (a 3 + abc) (b 3 + abc) (c 3 + abc) (AM − GM)<br />

= abc + 3√ (a 3 + abc) (b 3 + abc) (c 3 + abc).<br />

We now illustrate normalization techniques to establish classical theorems. Us<strong>in</strong>g the same idea <strong>in</strong> the<br />

pro<strong>of</strong> <strong>of</strong> the Cauchy-Schwarz <strong>in</strong>equality, we f<strong>in</strong>d a natural generalization :<br />

Theorem 3.4.2. Let a ij (i, j = 1, · · · , n) be positive real numbers. Then, we have<br />

(a 11 n + · · · + a 1n n ) · · · (a n1 n + · · · + a nn n ) ≥ (a 11 a 21 · · · a n1 + · · · + a 1n a 2n · · · a nn ) n .<br />

Pro<strong>of</strong>. S<strong>in</strong>ce the <strong>in</strong>equality is homogeneous, as <strong>in</strong> the pro<strong>of</strong> <strong>of</strong> the theorem 11, we can normalize to<br />

(a i1 n + · · · + a <strong>in</strong> n ) 1 n<br />

= 1 or a i1 n + · · · + a <strong>in</strong> n = 1 (i = 1, · · · , n).<br />

Then, the <strong>in</strong>equality takes the form a 11 a 21 · · · a n1 + · · · + a 1n a 2n · · · a nn ≤ 1 or ∑ n<br />

i=1 a i1 · · · a <strong>in</strong> ≤ 1. Hence,<br />

it suffices to show that, for all i = 1, · · · , n,<br />

a i1 · · · a <strong>in</strong> ≤ 1 n , where a i1 n + · · · + a <strong>in</strong> n = 1.<br />

To f<strong>in</strong>ish the pro<strong>of</strong>, it rema<strong>in</strong>s to show the follow<strong>in</strong>g homogeneous <strong>in</strong>equality :<br />

Theorem 3.4.3. (AM-GM <strong>in</strong>equality) Let a 1 , · · · , a n be positive real numbers. Then, we have<br />

a 1 + · · · + a n<br />

n<br />

≥ n√ a 1 · · · a n .<br />

Pro<strong>of</strong>. S<strong>in</strong>ce it’s homogeneous, we may rescale a 1 , · · · , a n<br />

so that a 1 · · · a n = 1. 4 We want to show that<br />

a 1 · · · a n = 1 =⇒ a 1 + · · · + a n ≥ n.<br />

The pro<strong>of</strong> is by <strong>in</strong>duction on n. If n = 1, it’s trivial. If n = 2, then we get a 1 + a 2 − 2 = a 1 + a 2 − 2 √ a 1 a 2 =<br />

( √ a 1 − √ a 2 ) 2 ≥ 0. Now, we assume that it holds for some positive <strong>in</strong>teger n ≥ 2. And let a 1 , · · · , a n+1<br />

be positive numbers such that a 1 · · · a n a n+1 =1. We may assume that a 1 ≥ 1 ≥ a 2 . (Why?) It follows that<br />

a 1 a 2 + 1 − a 1 − a 2 = (a 1 − 1)(a 2 − 1) ≤ 0 so that a 1 a 2 + 1 ≤ a 1 + a 2 . S<strong>in</strong>ce (a 1 a 2 )a 3 · · · a n = 1, by the<br />

<strong>in</strong>duction hypothesis, we have a 1 a 2 + a 3 + · · · + a n+1 ≥ n. Hence, a 1 + a 2 − 1 + a 3 + · · · + a n+1 ≥ n.<br />

The follow<strong>in</strong>g simple observation is not tricky :<br />

a i<br />

4 Set x i =<br />

(a 1···a n ) n<br />

1<br />

(i = 1, · · · , n). Then, we get x 1 · · · x n = 1 <strong>and</strong> it becomes x 1 + · · · + x n ≥ n.<br />

40


Let a, b > 0 <strong>and</strong> m, n ∈ N. Take x 1 = · · · = x m = a <strong>and</strong> x m+1 = · · · = x xm+n = b. Apply<strong>in</strong>g the<br />

AM-GM <strong>in</strong>equality to x 1 , · · · , x m+n > 0, we obta<strong>in</strong><br />

ma + nb<br />

m + n ≥ (am b n ) 1<br />

m+n<br />

or<br />

m<br />

m + n a + n<br />

m + n b ≥ a m n<br />

m+n b m+n .<br />

Hence, for all positive rationals ω 1 <strong>and</strong> ω 2 with ω 1 + ω 2 = 1, we get<br />

We immediately have<br />

ω 1 a + ω 2 b ≥ a ω 1<br />

b ω 2<br />

.<br />

Theorem 3.4.4. Let ω 1 , ω 2 > 0 with ω 1 + ω 2 = 1. For all x, y > 0, we have<br />

ω 1 x + ω 2 y ≥ x ω1 y ω2 .<br />

Pro<strong>of</strong>. We can choose a positive rational sequence a 1 , a 2 , a 3 , · · · such that<br />

And lett<strong>in</strong>g b i = 1 − a i , we get<br />

From the previous observation, we have<br />

lim a n = ω 1 .<br />

n→∞<br />

lim b n = ω 2 .<br />

n→∞<br />

a n x + b n y ≥ x an y bn<br />

By tak<strong>in</strong>g the limits to both sides, we get the result.<br />

Modify<strong>in</strong>g slightly the above arguments, we see that the AM-GM <strong>in</strong>equality implies that<br />

Theorem 3.4.5. (Weighted AM-GM <strong>in</strong>equality) Let ω 1 , · · · , ω n > 0 with ω 1 + · · · + ω n = 1. For all<br />

x 1 , · · · , x n > 0, we have<br />

ω 1 x 1 + · · · + ω n x n ≥ x 1<br />

ω1<br />

· · · x n<br />

ω n<br />

.<br />

Alternatively, we f<strong>in</strong>d that it is a straightforward consequence <strong>of</strong> the concavity <strong>of</strong> ln x. Indeed, the weighted<br />

Jensen’s <strong>in</strong>equality says that ln(ω 1 x 1 + · · · + ω n x n ) ≥ ω 1 ln(x 1 ) + · · · + ω n ln(x n ) = ln(x 1<br />

ω1<br />

· · · x n<br />

ω n<br />

).<br />

Recall that the AM-GM <strong>in</strong>equality is used to deduce the theorem 18, which is a generalization <strong>of</strong> the<br />

Cauchy-Schwarz <strong>in</strong>equality. S<strong>in</strong>ce we now get the weighted version <strong>of</strong> the AM-GM <strong>in</strong>equality, we establish<br />

weighted version <strong>of</strong> the Cauchy-Schwarz <strong>in</strong>equality.<br />

Theorem 3.4.6. (Hölder) Let x ij (i = 1, · · · , m, j = 1, · · · n) be positive real numbers. Suppose that<br />

ω 1 , · · · , ω n are positive real numbers satisfy<strong>in</strong>g ω 1 + · · · + ω n = 1. Then, we have<br />

(<br />

n∏ ∑ m<br />

) ⎛ ⎞<br />

ωj<br />

m∑ n∏<br />

x ij ≥ ⎝ ω<br />

x j ij<br />

⎠ .<br />

j=1<br />

i=1<br />

Pro<strong>of</strong>. Because <strong>of</strong> the homogeneity <strong>of</strong> the <strong>in</strong>equality, as <strong>in</strong> the pro<strong>of</strong> <strong>of</strong> the theorem 12, we may rescale<br />

x 1j , · · · , x mj so that x 1j + · · · + x mj = 1 for each j ∈ {1, · · · , n}. Then, we need to show that<br />

n∏ m∑ n∏<br />

m∑ n∏<br />

1 ω j ω<br />

≥ x j ω<br />

ij or 1 ≥ x j ij .<br />

j=1<br />

i=1 j=1<br />

The weighted AM-GM <strong>in</strong>equality provides that<br />

n∑<br />

n∏<br />

ω<br />

ω j x ij ≥ x j ij (i ∈ {1, · · · , m}) =⇒<br />

j=1<br />

j=1<br />

However, we immediately have<br />

m∑ n∑<br />

ω j x ij =<br />

i=1 j=1<br />

n∑<br />

j=1 i=1<br />

m∑<br />

ω j x ij =<br />

i=1<br />

j=1<br />

i=1 j=1<br />

m∑ n∑<br />

m∑ n∏<br />

ω<br />

ω j x ij ≥ x j ij .<br />

i=1 j=1<br />

(<br />

n∑ m<br />

)<br />

∑<br />

ω j x ij =<br />

j=1<br />

i=1<br />

i=1 j=1<br />

n∑<br />

ω j = 1.<br />

j=1<br />

41


Chapter 4<br />

Convexity<br />

Any good idea can be stated <strong>in</strong> fifty words or less. S. M. Ulam<br />

4.1 Jensen’s Inequality<br />

In the previous chapter, we deduced the weighted AM-GM <strong>in</strong>equality from the AM-GM <strong>in</strong>equality. We use<br />

the same idea to study the follow<strong>in</strong>g functional <strong>in</strong>equalities.<br />

Proposition 4.1.1. Let f : [a, b] −→ R be a cont<strong>in</strong>uous function. Then, the follow<strong>in</strong>gs are equivalent.<br />

(1) For all n ∈ N, the follow<strong>in</strong>g <strong>in</strong>equality holds.<br />

ω 1 f(x 1 ) + · · · + ω n f(x n ) ≥ f(ω 1 x 1 + · · · + ω n x n )<br />

for all x 1 , · · · , x n ∈ [a, b] <strong>and</strong> ω 1 , · · · , ω n > 0 with ω 1 + · · · + ω n = 1.<br />

(2) For all n ∈ N, the follow<strong>in</strong>g <strong>in</strong>equality holds.<br />

r 1 f(x 1 ) + · · · + r n f(x n ) ≥ f(r 1 x 1 + · · · + r n x n )<br />

for all x 1 , · · · , x n ∈ [a, b] <strong>and</strong> r 1 , · · · , r n ∈ Q + with r 1 + · · · + r n = 1.<br />

(3) For all N ∈ N, the follow<strong>in</strong>g <strong>in</strong>equality holds.<br />

( )<br />

f(y 1 ) + · · · + f(y N ) y1 + · · · + y N<br />

≥ f<br />

N<br />

N<br />

for all y 1 , · · · , y N ∈ [a, b].<br />

(4) For all k ∈ {0, 1, 2, · · · }, the follow<strong>in</strong>g <strong>in</strong>equality holds.<br />

f(y 1 ) + · · · + f(y 2 k)<br />

2 k ≥ f<br />

( )<br />

y1 + · · · + y 2 k<br />

2 k<br />

for all y 1 , · · · , y 2 k ∈ [a, b].<br />

(5) We have 1 2 f(x) + 1 2 f(y) ≥ f ( )<br />

x+y<br />

2 for all x, y ∈ [a, b].<br />

(6) We have λf(x) + (1 − λ)f(y) ≥ f (λx + (1 − λ)y) for all x, y ∈ [a, b] <strong>and</strong> λ ∈ (0, 1).<br />

Pro<strong>of</strong>. (1) ⇒ (2) ⇒ (3) ⇒ (4) ⇒ (5) is obvious.<br />

(2) ⇒ (1) : Let x 1 , · · · , x n ∈ [a, b] <strong>and</strong> ω 1 , · · · , ω n > 0 with ω 1 + · · · + ω n = 1. One may see that there<br />

exist positive rational sequences {r k (1)} k∈N , · · · , {r k (n)} k∈N satisfy<strong>in</strong>g<br />

lim r k(j) = w j (1 ≤ j ≤ n) <strong>and</strong> r k (1) + · · · + r k (n) = 1 for all k ∈ N.<br />

k→∞<br />

By the hypothesis <strong>in</strong> (2), we obta<strong>in</strong> r k (1)f(x 1 ) + · · · + r k (n)f(x n ) ≥ f(r k (1) x 1 + · · · + r k (n) x n ). S<strong>in</strong>ce f is<br />

cont<strong>in</strong>uous, tak<strong>in</strong>g k → ∞ to both sides yields the <strong>in</strong>equality<br />

ω 1 f(x 1 ) + · · · + ω n f(x n ) ≥ f(ω 1 x 1 + · · · + ω n x n ).<br />

42


(3) ⇒ (2) : Let x 1 , · · · , x n ∈ [a, b] <strong>and</strong> r 1 , · · · , r n ∈ Q + with r 1 + · · · + r n = 1. We can f<strong>in</strong>d a positive<br />

<strong>in</strong>teger N ∈ N so that Nr 1 , · · · , Nr n ∈ N. For each i ∈ {1, · · · , n}, we can write r i = pi<br />

N , where p i ∈ N. It<br />

follows from r 1 + · · · + r n = 1 that N = p 1 + · · · + p n . Then, (3) implies that<br />

=<br />

≥<br />

r 1 f(x 1 ) + · · · + r n f(x n )<br />

p 1 terms<br />

p n terms<br />

{ }} { { }} {<br />

f(x 1 ) + · · · + f(x 1 ) + · · · + f(x n ) + · · · + f(x n )<br />

⎛<br />

N<br />

⎞<br />

p 1 terms<br />

p n terms<br />

{ }} { { }} {<br />

x 1 + · · · + x 1 + · · · + x n + · · · + x n<br />

f ⎜<br />

⎟<br />

⎝<br />

N<br />

⎠<br />

= f(r 1 x 1 + · · · + r n x n ).<br />

(4) ⇒ (3) : Let y 1 , · · · , y N ∈ [a, b]. Take a large k ∈ N so that 2 k > N. Let a = y 1+···+y N<br />

N<br />

. Then, (4)<br />

implies that<br />

f(y 1 ) + · · · + f(y N ) + (2 k − n)f(a)<br />

2 k<br />

(2 k − N) terms<br />

{ }} {<br />

= f(y 1) + · · · + f(y N ) + f(a) + · · · + f(a)<br />

⎛<br />

2 k<br />

⎞<br />

(2 k − N) terms<br />

{ }} {<br />

≥ f<br />

y 1 + · · · + y N + a + · · · + a<br />

⎜<br />

⎝<br />

2 k ⎟<br />

⎠<br />

so that<br />

= f(a)<br />

(<br />

y1 + · · · + y N<br />

f(y 1 ) + · · · + f(y N ) ≥ Nf(a) = Nf<br />

N<br />

(5) ⇒ (4) : We use <strong>in</strong>duction on k. In case k = 0, 1, 2, it clearly holds. Suppose that (4) holds for some<br />

k ≥ 2. Let y 1 , · · · , y 2 k+1 ∈ [a, b]. By the <strong>in</strong>duction hypothesis, we obta<strong>in</strong><br />

)<br />

.<br />

f(y 1 ) + · · · + f(y 2 k) + f(y 2 +1) + · · · + f(y k 2 k+1)<br />

( ) ( )<br />

≥ 2 k y1 + · · · + y<br />

f<br />

2 k<br />

2 k + 2 k y2<br />

f<br />

k +1 + · · · + y 2 k+1<br />

2<br />

( ) ( k<br />

f<br />

y1 +···+ y 2 k<br />

y2<br />

)<br />

k<br />

+ f +1 +···+ y 2 k+1<br />

= 2 k+1 2 k 2 k<br />

2<br />

( y1 +···+ y 2 k<br />

≥ 2 k+1 + y 2 k +1 +···+ y )<br />

2 k+1<br />

2<br />

f<br />

k 2 k<br />

2<br />

( )<br />

= 2 k+1 y1 + · · · + y<br />

f<br />

2 k+1<br />

2 k+1 .<br />

Hence, (4) holds for k + 1. This completes the <strong>in</strong>duction.<br />

So far, we’ve established that (1), (2), (3), (4), (5) are all equivalent. S<strong>in</strong>ce (1) ⇒ (6) ⇒ (5) is obvious,<br />

this completes the pro<strong>of</strong>.<br />

Def<strong>in</strong>ition 4.1.1. A real valued function f is said to be convex on [a, b] if<br />

for all x, y ∈ [a, b] <strong>and</strong> λ ∈ (0, 1).<br />

λf(x) + (1 − λ)f(y) ≥ f (λx + (1 − λ)y)<br />

43


The above proposition says that<br />

Corollary 4.1.1. (Jensen’s <strong>in</strong>equality) Let f : [a, b] −→ R be a cont<strong>in</strong>uous convex function. For all<br />

x 1 , · · · , x n ∈ [a, b], we have<br />

( )<br />

f(x 1 ) + · · · + f(x n ) x1 + · · · + x n<br />

≥ f<br />

.<br />

n<br />

n<br />

Corollary 4.1.2. (Weighted Jensen’s <strong>in</strong>equality) Let f : [a, b] −→ R be a cont<strong>in</strong>uous convex function.<br />

Let ω 1 , · · · , ω n > 0 with ω 1 + · · · + ω n = 1. For all x 1 , · · · , x n ∈ [a, b], we have<br />

ω 1 f(x 1 ) + · · · + ω n f(x n ) ≥ f(ω 1 x 1 + · · · + ω n x n ).<br />

In fact, we can almost drop the cont<strong>in</strong>uity <strong>of</strong> f. As an exercise, show that every convex function on [a, b]<br />

is cont<strong>in</strong>uous on (a, b). So, every convex function on R is cont<strong>in</strong>uous on R. By the proposition aga<strong>in</strong>, we get<br />

Corollary 4.1.3. (Convexity Criterion I) Let f : [a, b] −→ R be a cont<strong>in</strong>uous function. Suppose that<br />

( )<br />

f(x) + f(y) x + y<br />

≥ f<br />

2<br />

2<br />

for all x, y ∈ [a, b]. Then, f is a convex function on [a, b].<br />

Exercise 20. (Convexity Criterion II) Let f : [a, b] −→ R be a cont<strong>in</strong>uous function which are differentiable<br />

twice <strong>in</strong> (a, b). Show that the follow<strong>in</strong>gs are equivalent.<br />

(1) f ′′ (x) ≥ 0 for all x ∈ (a, b).<br />

(2) f is convex on (a, b).<br />

When we deduce (5) ⇒ (4) ⇒ (3) ⇒ (2) <strong>in</strong> the proposition, we didn’t use the cont<strong>in</strong>uity <strong>of</strong> f :<br />

Corollary 4.1.4. Let f : [a, b] −→ R be a function. Suppose that<br />

( )<br />

f(x) + f(y) x + y<br />

≥ f<br />

2<br />

2<br />

for all x, y ∈ [a, b]. Then, we have<br />

r 1 f(x 1 ) + · · · + r n f(x n ) ≥ f(r 1 x 1 + · · · + r n x n )<br />

for all x 1 , · · · , x n ∈ [a, b] <strong>and</strong> r 1 , · · · , r n ∈ Q + with r 1 + · · · + r n = 1.<br />

We close this section by present<strong>in</strong>g an well-known <strong>in</strong>ductive pro<strong>of</strong> <strong>of</strong> the weighted Jensen’s <strong>in</strong>equality. It<br />

turns out that we can completely drop the cont<strong>in</strong>uity <strong>of</strong> f.<br />

Second Pro<strong>of</strong>. It clearly holds for n = 1, 2. We now assume that it holds for some n ∈ N. Let x 1 , · · · , x n , x n+1 ∈<br />

ω<br />

[a, b] <strong>and</strong> ω 1 , · · · , ω n+1 > 0 with ω 1 + · · · + ω n+1 = 1. S<strong>in</strong>ce 1<br />

1−ω n+1<br />

+ · · · + ωn<br />

1−ω n+1<br />

= 1, it follows from the<br />

<strong>in</strong>duction hypothesis that<br />

ω 1 f(x 1 ) + · · · + ω n+1 f(x n+1 )<br />

(<br />

ω 1<br />

= (1 − ω n+1 )<br />

f(x 1 ) + · · · +<br />

1 − ω n+1<br />

(<br />

ω 1<br />

≥ (1 − ω n+1 )f<br />

x 1 + · · · +<br />

1 − ω n+1<br />

( [<br />

ω 1<br />

≥ f (1 − ω n+1 ) x 1 + · · · +<br />

1 − ω n+1<br />

= f(ω 1 x 1 + · · · + ω n+1 x n+1 ).<br />

)<br />

ω n<br />

f(x n ) + ω n+1 f(x n+1 )<br />

1 − ω n+1<br />

)<br />

ω n<br />

x n + ω n+1 f(x n+1 )<br />

1 − ω n+1<br />

]<br />

)<br />

ω n<br />

x n + ω n+1 x n+1<br />

1 − ω n+1<br />

44


4.2 Power Means<br />

Convexity is one <strong>of</strong> the most important concepts <strong>in</strong> analysis. Jensen’s <strong>in</strong>equality is the most powerful tool<br />

<strong>in</strong> theory <strong>of</strong> <strong>in</strong>equalities. In this section, we shall establish the Power Mean <strong>in</strong>equality by apply<strong>in</strong>g Jensen’s<br />

<strong>in</strong>equality <strong>in</strong> two ways. We beg<strong>in</strong> with two simple lemmas.<br />

Lemma 4.2.1. Let a, b, <strong>and</strong> c be positive real numbers. Let us def<strong>in</strong>e a function f : R −→ R by<br />

( a x + b x + c x )<br />

f(x) = ln<br />

,<br />

3<br />

where x ∈ R. Then, we obta<strong>in</strong> f ′ (0) = ln (abc) 1 3 .<br />

Pro<strong>of</strong>. We compute f ′ (x) = ax ln a+b x ln b+c x ln c<br />

a x +b x +c<br />

. Then, f ′ ln a+ln b+ln c<br />

(0) = x<br />

3<br />

= ln (abc) 1 3 .<br />

Lemma 4.2.2. Let f : R −→ R be a cont<strong>in</strong>uous function. Suppose that f is monotone <strong>in</strong>creas<strong>in</strong>g on (0, ∞)<br />

<strong>and</strong> monotone <strong>in</strong>creas<strong>in</strong>g on (−∞, 0). Then, f is monotone <strong>in</strong>creas<strong>in</strong>g on R.<br />

Pro<strong>of</strong>. We first show that f is monotone <strong>in</strong>creas<strong>in</strong>g on [0, ∞). By the hypothesis, it rema<strong>in</strong>s to show that<br />

f(x) ≥ f(0) for all x > 0. For all ɛ ∈ (0, x), we have f(x) ≥ f(ɛ). S<strong>in</strong>ce f is cont<strong>in</strong>uous at 0, we obta<strong>in</strong><br />

f(x) ≥ lim f(ɛ) = f(0).<br />

ɛ→0 +<br />

Similarly, we f<strong>in</strong>d that f is monotone <strong>in</strong>creas<strong>in</strong>g on (−∞, 0]. We now show that f is monotone <strong>in</strong>creas<strong>in</strong>g<br />

on R. Let x <strong>and</strong> y be real numbers with x > y. We want to show that f(x) ≥ f(y). In case 0 ∉ (x, y), we<br />

get the result by the hypothesis. In case x ≥ 0 ≥ y, it follows that f(x) ≥ f(0) ≥ f(y).<br />

Theorem 4.2.1. (Power Mean <strong>in</strong>equality for three variables ) Let a, b, <strong>and</strong> c be positive real numbers.<br />

We def<strong>in</strong>e a function M (a,b,c) : R −→ R by<br />

M (a,b,c) (0) = 3√ ( a r + b r + c r<br />

abc , M (a,b,c) (r) =<br />

3<br />

) 1<br />

r<br />

(r ≠ 0).<br />

Then, M (a,b,c) is a monotone <strong>in</strong>creas<strong>in</strong>g cont<strong>in</strong>uous function.<br />

First Pro<strong>of</strong>. Write M(r) = M (a,b,c) (r). We first establish that M is cont<strong>in</strong>uous. S<strong>in</strong>ce M is cont<strong>in</strong>uous at r<br />

for all r ≠ 0, it’s enough to show that<br />

lim M(r) = 3√ abc.<br />

r→0<br />

Let f(x) = ln ( a x +b x +c x<br />

3<br />

)<br />

, where x ∈ R. S<strong>in</strong>ce f(0) = 0, the lemma 2 implies that<br />

S<strong>in</strong>ce e x is a cont<strong>in</strong>uous function, this means that<br />

f(r) f(r) − f(0)<br />

lim = lim<br />

= f ′ (0) = ln 3√ abc .<br />

r→0 r r→0 r − 0<br />

lim M(r) = lim e f(r)<br />

r = e ln 3√ abc = 3√ abc.<br />

r→0 r→0<br />

Now, we show that M is monotone <strong>in</strong>creas<strong>in</strong>g. By the lemma 3, it will be enough to establish that M is<br />

monotone <strong>in</strong>creas<strong>in</strong>g on (0, ∞) <strong>and</strong> monotone <strong>in</strong>creas<strong>in</strong>g on (−∞, 0). We first show that M is monotone<br />

<strong>in</strong>creas<strong>in</strong>g on (0, ∞). Let x ≥ y > 0. We want to show that<br />

( a x + b x + c x<br />

After the substitution u = a y , v = a y , w = a z , it becomes<br />

3<br />

(u x x x<br />

y + v<br />

y + w<br />

y<br />

3<br />

) 1 (<br />

x a y + b y + c y ) 1<br />

y<br />

≥ .<br />

3<br />

) 1<br />

x<br />

≥<br />

( u + v + w<br />

3<br />

) 1<br />

y<br />

.<br />

45


S<strong>in</strong>ce it is homogeneous, we may normalize to u + v + w = 3. We are now required to show that<br />

where G(t) = t x y , where t > 0. S<strong>in</strong>ce<br />

x<br />

y<br />

G(u) + G(v) + G(w)<br />

3<br />

G(u) + G(v) + G(w)<br />

3<br />

≥ 1,<br />

Similarly, we may deduce that M is monotone <strong>in</strong>creas<strong>in</strong>g on (−∞, 0).<br />

≥ 1, we f<strong>in</strong>d that G is convex. Jensen’s <strong>in</strong>equality shows that<br />

( )<br />

u + v + w<br />

≥ G<br />

= G(1) = 1.<br />

3<br />

We’ve learned that the convexity <strong>of</strong> f(x) = x λ (λ ≥ 1) implies the monotonicity <strong>of</strong> the power means.<br />

Now, we shall show that the convexity <strong>of</strong> x ln x also implies the power mean <strong>in</strong>equality.<br />

Second Pro<strong>of</strong> <strong>of</strong> the Monotonicity. Write f(x) = M (a,b,c) (x). We use the <strong>in</strong>creas<strong>in</strong>g function theorem. By<br />

the lemma 3, it’s enough to show that f ′ (x) ≥ 0 for all x ≠ 0. Let x ∈ R − {0}. We compute<br />

f ′ (x)<br />

f(x) = d<br />

dx (ln f(x)) = − 1 ( a x<br />

x 2 ln + b x + c x )<br />

+ 1 1<br />

3 (ax ln a + b x ln b + c x ln c)<br />

3 x<br />

1<br />

3 (ax + b x + c x )<br />

or<br />

x 2 f ′ (x)<br />

f(x)<br />

( a x + b x + c x<br />

= − ln<br />

3<br />

)<br />

+ ax ln a x + b x ln b x + c x ln c x<br />

a x + b x + c x .<br />

To establish f ′ (x) ≥ 0, we now need to establish that<br />

( a<br />

a x ln a x + b x ln b x + c x ln c x ≥ (a x + b x + c x x + b x + c x )<br />

) ln<br />

.<br />

3<br />

Let us <strong>in</strong>troduce a function f : (0, ∞) −→ R by f(t) = t ln t, where t > 0. After the substitution p = a x ,<br />

q = a y , r = a z , it becomes<br />

( ) p + q + r<br />

f(p) + f(q) + f(r) ≥ 3f<br />

.<br />

3<br />

S<strong>in</strong>ce f is convex on (0, ∞), it follows immediately from Jensen’s <strong>in</strong>equality.<br />

As a corollary, we obta<strong>in</strong> the RMS-AM-GM-HM <strong>in</strong>equality for three variables.<br />

Corollary 4.2.1. For all positive real numbers a, b, <strong>and</strong> c, we have<br />

√<br />

a2 + b 2 + c 2<br />

≥ a + b + c ≥ 3√ 3<br />

abc ≥<br />

3<br />

3<br />

1<br />

a + 1 b + 1 .<br />

c<br />

Pro<strong>of</strong>. The Power Mean <strong>in</strong>equality states that M (a,b,c) (2) ≥ M (a,b,c) (1) ≥ M (a,b,c) (0) ≥ M (a,b,c) (−1).<br />

Us<strong>in</strong>g the convexity <strong>of</strong> x ln x or the convexity <strong>of</strong> x λ<br />

the power means for n positive real numbers.<br />

(λ ≥ 1), we can also establish the monotonicity <strong>of</strong><br />

Theorem 4.2.2. (Power Mean <strong>in</strong>equality) Let x 1 , · · · , x n > 0. The power mean <strong>of</strong> order r is def<strong>in</strong>ed by<br />

( ) 1<br />

M (x1,··· ,x n)(0) = n√ x<br />

r r r<br />

x 1 · · · x n , M (x1,··· ,x n)(r) = 1 + · · · + x n<br />

(r ≠ 0).<br />

n<br />

Then, M (x1,··· ,x n) : R −→ R is cont<strong>in</strong>uous <strong>and</strong> monotone <strong>in</strong>creas<strong>in</strong>g.<br />

We conclude that<br />

Corollary 4.2.2. (Geometric Mean as a Limit) Let x 1 , · · · , x n > 0. Then,<br />

(<br />

n√ r ) 1<br />

r<br />

x1<br />

r<br />

+ · · · + x n x1 · · · x n = lim<br />

.<br />

r→0 n<br />

Theorem 4.2.3. (RMS-AM-GM-HM <strong>in</strong>equality) For all x 1 , · · · , x n > 0, we have<br />

√<br />

x12 + · · · + x n<br />

2<br />

n<br />

≥ x 1 + · · · + x n<br />

n<br />

≥ n√ n<br />

x 1 · · · x n ≥<br />

1<br />

x 1<br />

+ · · · + 1 .<br />

x n<br />

46


4.3 Majorization Inequality<br />

We say that a vector x = (x 1 , · · · , x n ) majorizes another vector y = (y 1 , · · · , y n ) if<br />

(1) x 1 ≥ · · · ≥ x n , y 1 ≥ · · · ≥ y n ,<br />

(2) x 1 + · · · + x k ≥ y 1 + · · · + y k for all 1 ≤ k ≤ n − 1,<br />

(3) x 1 + · · · + x n = y 1 + · · · + y n .<br />

Theorem 4.3.1. (Majorization Inequality) Let f : [a, b] −→ R be a convex function.<br />

(x 1 , · · · , x n ) majorizes (y 1 , · · · , y n ), where x 1 , · · · , x n , y 1 , · · · , y n ∈ [a, b]. Then, we obta<strong>in</strong><br />

Suppose that<br />

f(x 1 ) + · · · + f(x n ) ≥ f(y 1 ) + · · · + f(y n ).<br />

For example, we can m<strong>in</strong>imize cos A + cos B + cos C, where ABC is an acute triangle. Recall that − cos x<br />

is convex on ( ) (<br />

0, π 2 . S<strong>in</strong>ce π<br />

2 , π 2 , 0) majorize (A, B, C), the majorization <strong>in</strong>equality implies that<br />

( π<br />

) ( π<br />

)<br />

cos A + cos B + cos C ≥ cos + cos + cos 0 = 1.<br />

2 2<br />

Also, <strong>in</strong> a triangle ABC, the convexity <strong>of</strong> tan ( ) 2 x<br />

4 on [0, π] <strong>and</strong> the majorization <strong>in</strong>equality show that<br />

21 − 12 √ ( 3 = 3 tan 2 π<br />

) ( ) ( ) ( )<br />

A B C ( ≤ tan 2 + tan 2 + tan 2 ≤ tan 2 π<br />

)<br />

+ tan 2 0 + tan 2 0 = 1.<br />

12<br />

4<br />

4<br />

4<br />

4<br />

(IMO 1999/2) Let n be an <strong>in</strong>teger with n ≥ 2.<br />

Determ<strong>in</strong>e the least constant C such that the <strong>in</strong>equality<br />

⎛<br />

∑<br />

x i x j (x 2 i + x 2 j) ≤ C ⎝ ∑<br />

1≤i


4.4 Support<strong>in</strong>g L<strong>in</strong>e Inequality<br />

There is a simple way to f<strong>in</strong>d new bounds for given differentiable functions. We beg<strong>in</strong> to show that every<br />

support<strong>in</strong>g l<strong>in</strong>es are tangent l<strong>in</strong>es <strong>in</strong> the follow<strong>in</strong>g sense.<br />

Proposition 4.4.1. (Characterization <strong>of</strong> Support<strong>in</strong>g L<strong>in</strong>es) Let f be a real valued function.<br />

m, n ∈ R. Suppose that<br />

Let<br />

(1) f(α) = mα + n for some α ∈ R,<br />

(2) f(x) ≥ mx + n for all x <strong>in</strong> some <strong>in</strong>terval (ɛ 1 , ɛ 2 ) <strong>in</strong>clud<strong>in</strong>g α, <strong>and</strong><br />

(3) f is differentiable at α.<br />

Then, the support<strong>in</strong>g l<strong>in</strong>e y = mx + n <strong>of</strong> f is the tangent l<strong>in</strong>e <strong>of</strong> f at x = α.<br />

Pro<strong>of</strong>. Let us def<strong>in</strong>e a function F : (ɛ 1 , ɛ 2 ) −→ R by F (x) = f(x) − mx − n for all x ∈ (ɛ 1 , ɛ 2 ). Then, F is<br />

differentiable at α <strong>and</strong> we obta<strong>in</strong> F ′ (α) = f ′ (α)−m. By the assumption (1) <strong>and</strong> (2), we see that F has a local<br />

m<strong>in</strong>imum at α. So, the first derivative theorem for local extreme values implies that 0 = F ′ (α) = f ′ (α)−m so<br />

that m = f ′ (α) <strong>and</strong> that n = f(α)−mα = f(α)−f ′ (α)α. It follows that y = mx+n = f ′ (α)(x−α)+f(α).<br />

(Nesbitt, 1903) For all positive real numbers a, b, c, we have<br />

cyclic<br />

a<br />

b + c +<br />

b<br />

c + a +<br />

c<br />

a + b ≥ 3 2 .<br />

Pro<strong>of</strong> 13. We may normalize to a + b + c = 1. Note that 0 < a, b, c < 1. The problem is now to prove<br />

∑<br />

f(a) ≥ 3 (<br />

f(a) + f(b) + f(c) 1<br />

⇔ ≥ f , where f(x) =<br />

2 3<br />

3)<br />

x<br />

1 − x .<br />

The equation <strong>of</strong> the tangent l<strong>in</strong>e <strong>of</strong> f at x = 1 3<br />

x ∈ (0, 1). It follows from the identity<br />

f(x) − 9x − 1<br />

4<br />

9x−1<br />

is given by y =<br />

4<br />

. We claim that f(x) ≥ 9x−1<br />

4<br />

for all<br />

=<br />

(3x − 1)2<br />

4(1 − x) .<br />

Now, we conclude that<br />

∑<br />

cyclic<br />

a<br />

1 − a ≥ ∑<br />

cyclic<br />

9a − 1<br />

4<br />

= 9 4<br />

∑<br />

a − 3 4 = 3 2 .<br />

cyclic<br />

The above argument can be generalized. If a function f has a support<strong>in</strong>g l<strong>in</strong>e at some po<strong>in</strong>t on the graph<br />

<strong>of</strong> f, then f satisfies Jensen’s <strong>in</strong>equality <strong>in</strong> the follow<strong>in</strong>g sense.<br />

Theorem 4.4.1. (Support<strong>in</strong>g L<strong>in</strong>e Inequality) Let f : [a, b] −→ R be a function. Suppose that α ∈ [a, b]<br />

<strong>and</strong> m ∈ R satisfy<br />

f(x) ≥ m(x − α) + f(α)<br />

for all x ∈ [a, b]. Let ω 1 , · · · , ω n > 0 with ω 1 + · · · + ω n = 1. Then, the follow<strong>in</strong>g <strong>in</strong>equality holds<br />

ω 1 f(x 1 ) + · · · + ω n f(x n ) ≥ f(α)<br />

for all x 1 , · · · , x n ∈ [a, b] such that α = ω 1 x 1 + · · · + ω n x n . In particular, we obta<strong>in</strong><br />

f(x 1 ) + · · · + f(x n )<br />

n<br />

≥ f<br />

( s<br />

n)<br />

,<br />

where x 1 , · · · , x n ∈ [a, b] with x 1 + · · · + x n = s for some s ∈ [na, nb].<br />

Pro<strong>of</strong>. It follows that ω 1 f(x 1 )+· · ·+ω n f(x n ) ≥ ω 1 [m(x 1 −α)+f(α)]+· · ·+ω 1 [m(x n −α)+f(α)] = f(α).<br />

We can apply the support<strong>in</strong>g l<strong>in</strong>e <strong>in</strong>equality to deduce Jensen’s <strong>in</strong>equality for differentiable functions.<br />

48


Lemma 4.4.1. Let f : (a, b) −→ R be a convex function which is differentiable twice on (a, b). Let y = l α (x)<br />

be the tangent l<strong>in</strong>e at α ∈ (a, b). Then, f(x) ≥ l α (x) for all x ∈ (a, b).<br />

Pro<strong>of</strong>. Let α ∈ (a, b). We want to show that the tangent l<strong>in</strong>e y = l α (x) = f ′ (α)(x − α) + f(α) is the<br />

support<strong>in</strong>g l<strong>in</strong>e <strong>of</strong> f at x = α such that f(x) ≥ l α (x) for all x ∈ (a, b). However, by Taylor’s theorem, we<br />

can f<strong>in</strong>d a θ x between α <strong>and</strong> x such that<br />

f(x) = f(α) + f ′ (α)(x − α) + f ′′ (θ x )<br />

(x − α) 2 ≥ f(α) + f ′ (α)(x − α).<br />

2<br />

(Weighted Jensen’s <strong>in</strong>equality) Let f : [a, b] −→ R be a cont<strong>in</strong>uous convex function which is<br />

differentiable twice on (a, b). Let ω 1 , · · · , ω n > 0 with ω 1 +· · ·+ω n = 1. For all x 1 , · · · , x n ∈ [a, b],<br />

ω 1 f(x 1 ) + · · · + ω n f(x n ) ≥ f(ω 1 x 1 + · · · + ω n x n ).<br />

Third Pro<strong>of</strong>. By the cont<strong>in</strong>uity <strong>of</strong> f, we may assume that x 1 , · · · , x n ∈ (a, b). Now, let µ = ω 1 x 1 +· · ·+ω n x n .<br />

Then, µ ∈ (a, b). By the above lemma, f has the tangent l<strong>in</strong>e y = l µ (x) = f ′ (µ)(x − µ) + f(µ) at x = µ<br />

satisfy<strong>in</strong>g f(x) ≥ l µ (x) for all x ∈ (a, b). Hence, the support<strong>in</strong>g l<strong>in</strong>e <strong>in</strong>equality shows that<br />

ω 1 f(x 1 ) + · · · + ω n f(x n ) ≥ ω 1 f(µ) + · · · + ω n f(µ) = f(µ) = f(ω 1 x 1 + · · · + ω n x n ).<br />

We note that the cos<strong>in</strong>e function is concave on [ 0, π 2<br />

]<br />

<strong>and</strong> convex on<br />

[ π<br />

2 , π] . Non-convex functions can be<br />

locally convex <strong>and</strong> have support<strong>in</strong>g l<strong>in</strong>es at some po<strong>in</strong>ts. This means that the support<strong>in</strong>g l<strong>in</strong>e <strong>in</strong>equality is<br />

a powerful tool because we can also produce Jensen-type <strong>in</strong>equalities for non-convex functions.<br />

(Theorem 6) In any triangle ABC, we have cos A + cos B + cos C ≤ 3 2 .<br />

Third Pro<strong>of</strong>. Let f(x) = − cos x. Our goal is to establish a three-variables <strong>in</strong>equality<br />

f(A) + f(B) + f(C)<br />

3<br />

≥ f<br />

( π<br />

3<br />

)<br />

,<br />

where A, B, C ∈ (0, π) with A + B + C = π. We compute f ′ (x) = s<strong>in</strong> x. The equation <strong>of</strong> the tangent l<strong>in</strong>e <strong>of</strong><br />

f at x = π 3 is given by y = √ ( )<br />

3<br />

2 x −<br />

π<br />

3 −<br />

1<br />

2<br />

. To apply the support<strong>in</strong>g l<strong>in</strong>e <strong>in</strong>equality, we need to show that<br />

√<br />

3<br />

(<br />

− cos x ≥ x − π )<br />

− 1 2 3 2<br />

for all x ∈ (0, π). This is a one-variable <strong>in</strong>equality! We omit the pro<strong>of</strong>.<br />

Problem 29. (Japan 1997) Let a, b, <strong>and</strong> c be positive real numbers. Prove that<br />

(b + c − a) 2 (c + a − b)2 (a + b − c)2<br />

(b + c) 2 +<br />

+ a2 (c + a) 2 +<br />

+ b2 (a + b) 2 + c 2 ≥ 3 5 .<br />

Pro<strong>of</strong>. Because <strong>of</strong> the homogeneity <strong>of</strong> the <strong>in</strong>equality, we may normalize to a + b + c = 1. It takes the form<br />

(1 − 2a) 2 (1 − 2b)2 (1 − 2c)2<br />

(1 − a) 2 +<br />

+ a2 (1 − b) 2 +<br />

+ b2 (1 − c) 2 + c 2 ≥ 3 5 ⇔ 1<br />

2a 2 − 2a + 1 + 1<br />

2b 2 − 2b + 1 + 1<br />

2c 2 − 2c + 1 ≤ 27 5 .<br />

We f<strong>in</strong>d that the equation <strong>of</strong> the tangent l<strong>in</strong>e <strong>of</strong> f(x) =<br />

1<br />

2x 2 −2x+1 at x = 1 3<br />

for all x > 0. It follows that<br />

( 54<br />

f(x) −<br />

25 x + 27 )<br />

= − 2(3x − 1)2 (6x + 1)<br />

25 25(2x 2 ≤ 0.<br />

− 2x + 1)<br />

∑<br />

f(a) ≤ ∑<br />

cyclic<br />

cyclic<br />

54<br />

25 a + 27<br />

25 = 27 5 .<br />

54<br />

is given by y =<br />

25 x + 27<br />

25<br />

<strong>and</strong> that<br />

49


Chapter 5<br />

Problems, Problems, Problems<br />

Each problem that I solved became a rule, which served afterwards to solve other problems. Rene Descartes<br />

5.1 Multivariable <strong>Inequalities</strong><br />

M 1. (IMO short-listed 2003) Let (x 1 , x 2 , · · · , x n ) <strong>and</strong> (y 1 , y 2 , · · · , y n ) be two sequences <strong>of</strong> positive real<br />

numbers. Suppose that (z 1 , z 2 , · · · , z n ) is a sequence <strong>of</strong> positive real numbers such that<br />

z i+j 2 ≥ x i y j<br />

for all 1 ≤ i, j ≤ n. Let M = max{z 2 , · · · , z 2n }. Prove that<br />

( ) 2 ( ) ( )<br />

M + z2 + · · · + z 2n x1 + · · · + x n y1 + · · · + y n<br />

≥<br />

.<br />

2n<br />

n<br />

n<br />

M 2. (Bosnia <strong>and</strong> Herzegov<strong>in</strong>a 2002) Let a 1 , · · · , a n , b 1 , · · · , b n , c 1 , · · · , c n<br />

Prove the follow<strong>in</strong>g <strong>in</strong>equality :<br />

( n<br />

∑<br />

a i<br />

3<br />

) ( n<br />

∑<br />

b i<br />

3<br />

) ( n<br />

∑<br />

c i<br />

3<br />

) ( n<br />

) 3<br />

∑<br />

≥ a i b i c i .<br />

i=1 i=1 i=1<br />

i=1<br />

be positive real numbers.<br />

M 3. (C 1 2113, Marc<strong>in</strong> E. Kuczma) Prove that <strong>in</strong>equality<br />

n∑<br />

i=1<br />

a i<br />

n ∑<br />

i=1<br />

b i ≥<br />

for any positive real numbers a 1 , · · · , a n , b 1 , · · · , b n<br />

n∑<br />

n∑ a i b i<br />

(a i + b i )<br />

a i + b i<br />

i=1<br />

M 4. (Yogoslavia 1998) Let n > 1 be a positive <strong>in</strong>teger <strong>and</strong> a 1 , · · · , a n , b 1 , · · · , b n be positive real numbers.<br />

Prove the follow<strong>in</strong>g <strong>in</strong>equality.<br />

⎛ ⎞<br />

⎝ ∑ 2<br />

a i b j<br />

⎠ ≥ ∑ ∑<br />

a i a j b i b j .<br />

i≠j<br />

i≠j i≠j<br />

M 5. (C2176, Sefket Arslanagic) Prove that<br />

i=1<br />

where a 1 , · · · , a n , b 1 , · · · , b n > 0<br />

1 CRUX with MAYHEM<br />

((a 1 + b 1 ) · · · (a n + b n )) 1 n ≥ (a1 · · · a n ) 1 n + (b1 · · · b n ) 1 n<br />

50


M 6. (Korea 2001) Let x 1 , · · · , x n <strong>and</strong> y 1 , · · · , y n be real numbers satisfy<strong>in</strong>g<br />

Show that<br />

<strong>and</strong> determ<strong>in</strong>e when equality holds.<br />

x 1 2 + · · · + x n 2 = y 1 2 + · · · + y n 2 = 1<br />

∣ n 2<br />

∣ 1 − ∑ ∣∣∣∣<br />

x i y i ≥ (x 1 y 2 − x 2 y 1 ) 2<br />

i=1<br />

M 7. (S<strong>in</strong>gapore 2001) Let a 1 , · · · , a n , b 1 , · · · , b n be real numbers between 1001 <strong>and</strong> 2002 <strong>in</strong>clusive. Suppose<br />

that<br />

n∑ n∑<br />

a 2 i = b 2 i .<br />

Prove that<br />

Determ<strong>in</strong>e when equality holds.<br />

i=1<br />

n∑<br />

i=1<br />

3<br />

a i<br />

≤ 17<br />

b i 10<br />

i=1<br />

n∑<br />

a 2 i .<br />

M 8. (Abel’s <strong>in</strong>equality) Let a 1 , · · · , a N , x 1 , · · · , x N be real numbers with x n ≥ x n+1 > 0 for all n. Show<br />

that<br />

|a 1 x 1 + · · · + a N x N | ≤ Ax 1<br />

where<br />

i=1<br />

A = max{|a 1 |, |a 1 + a 2 |, · · · , |a 1 + · · · + a N |}.<br />

M 9. (Ch<strong>in</strong>a 1992) For every <strong>in</strong>teger n ≥ 2 f<strong>in</strong>d the smallest positive number λ = λ(n) such that if<br />

then<br />

0 ≤ a 1 , · · · , a n ≤ 1 2 , b 1, · · · , b n > 0, a 1 + · · · + a n = b 1 + · · · + b n = 1<br />

b 1 · · · b n ≤ λ(a 1 b 1 + · · · + a n b n ).<br />

M 10. (C2551, Panos E. Tsaoussoglou) Suppose that a 1 , · · · , a n are positive real numbers. Let e j,k =<br />

n − 1 if j = k <strong>and</strong> e j,k = n − 2 otherwise. Let d j,k = 0 if j = k <strong>and</strong> d j,k = 1 otherwise. Prove that<br />

(<br />

n∑ n∏<br />

n∏ n<br />

) 2<br />

∑<br />

e j,k a 2 k ≥ d j,k a k<br />

j=1 k=1<br />

j=1<br />

M 11. (C2627, Walther Janous) Let x 1 , · · · , x n (n ≥ 2) be positive real numbers <strong>and</strong> let x 1 + · · · + x n .<br />

Let a 1 , · · · , a n be non-negative real numbers. Determ<strong>in</strong>e the optimum constant C(n) such that<br />

n∑<br />

j=1<br />

a j (s n − x j )<br />

x j<br />

k=1<br />

⎛<br />

n∏<br />

≥ C(n) ⎝<br />

j=1<br />

⎞ 1<br />

n<br />

a j<br />

⎠ .<br />

M 12. (Hungary-Israel B<strong>in</strong>ational Mathematical Competition 2000) Suppose that k <strong>and</strong> l are two<br />

given positive <strong>in</strong>tegers <strong>and</strong> a ij (1 ≤ i ≤ k, 1 ≤ j ≤ l) are given positive numbers. Prove that if q ≥ p > 0,<br />

then<br />

⎛<br />

) ⎞ q 1 ⎛ ⎛ ⎞ p ⎞ 1<br />

q<br />

p<br />

p<br />

⎝<br />

l∑<br />

j=1<br />

( k∑<br />

i=1<br />

a ij<br />

p<br />

⎠<br />

≤<br />

⎜<br />

⎝<br />

k∑ l∑<br />

⎝<br />

M 13. (Kantorovich <strong>in</strong>equality) Suppose x 1 < · · · < x n are given positive numbers. Let λ 1 , · · · , λ n ≥ 0<br />

<strong>and</strong> λ 1 + · · · + λ n = 1. Prove that (<br />

∑ n<br />

) ( n<br />

)<br />

∑ λ i<br />

λ i x i ≤ A2<br />

x i G 2 ,<br />

where A = x 1+x n<br />

2<br />

<strong>and</strong> G = √ x 1 x n .<br />

i=1<br />

i=1<br />

51<br />

i=1<br />

j=1<br />

a ij<br />

q<br />

⎠<br />

q<br />

⎟<br />

⎠<br />

.


M 14. (Czech-Slovak-Polish Match 2001) Let n ≥ 2 be an <strong>in</strong>teger. Show that<br />

for all nonnegative reals a 1 , · · · , a n .<br />

(a 1 3 + 1)(a 2 3 + 1) · · · (a n 3 + 1) ≥ (a 1 2 a 2 + 1)(a 2 2 a 3 + 1) · · · (a n 2 a 1 + 1)<br />

M 15. (C1868, De-jun Zhao) Let n ≥ 3, a 1 > a 2 > · · · > a n > 0, <strong>and</strong> p > q > 0. Show that<br />

a 1 p a 2 q + a 2 p a 3 q + · · · + a n−1 p a n q + a n p a 1 q ≥ a 1 q a 2 p + a 2 q a 3 p + · · · + a n−1 q a n p + a n q a 1<br />

p<br />

M 16. (Baltic Way 1996) For which positive real numbers a, b does the <strong>in</strong>equality<br />

x 1 x 2 + x 2 x 3 + · · · + x n−1 x n + x n x 1 ≥ x 1 a x 2 b x 3 a + x 2 a x 3 b x 4 a + · · · + x n a x 1 b x 2<br />

a<br />

holds for all <strong>in</strong>tegers n > 2 <strong>and</strong> positive real numbers x 1 , · · · , x n .<br />

M 17. (IMO short List 2000) Let x 1 , x 2 , · · · , x n be arbitrary real numbers. Prove the <strong>in</strong>equality<br />

x 1<br />

1 + x 1<br />

2 + x 2<br />

1 + x 12 + x 2<br />

2 + · · · + x n<br />

1 + x 12 + · · · + x n<br />

2 < √ n.<br />

M 18. (MM 2 1479, Donald E. Knuth) Let M n be the maximum value <strong>of</strong> the quantity<br />

x n<br />

(1 + x 1 + · · · + x n ) 2 + x 2<br />

(1 + x 2 + · · · + x n ) 2 + · · · + x 1<br />

(1 + x n ) 2<br />

over all nonnegative real numbers (x 1 , · · · , x n ). At what po<strong>in</strong>t(s) does the maximum occur ? Express M n <strong>in</strong><br />

terms <strong>of</strong> M n−1 , <strong>and</strong> f<strong>in</strong>d lim n→∞ M n .<br />

M 19. (IMO 1971) Prove the follow<strong>in</strong>g assertion is true for n = 3 <strong>and</strong> n = 5 <strong>and</strong> false for every other<br />

natural number n > 2 : if a 1 , · · · , a n are arbitrary real numbers, then<br />

n∑ ∏<br />

(a i − a j ) ≥ 0.<br />

i=1 i≠j<br />

M 20. (IMO 2003) Let x 1 ≤ x 2 ≤ · · · ≤ x n be real numbers.<br />

(a) Prove that<br />

⎛<br />

⎞<br />

⎝<br />

∑<br />

2<br />

|x i − x j | ⎠ ≤ 2(n2 − 1)<br />

3<br />

1≤i,j≤n<br />

∑<br />

1≤i,j≤n<br />

(x i − x j ) 2 .<br />

(b) Show that the equality holds if <strong>and</strong> only if x 1 , x 2 , · · · , x n is an arithmetic sequence.<br />

M 21. (Bulgaria 1995) Let n ≥ 2 <strong>and</strong> 0 ≤ x 1 , · · · , x n ≤ 1. Show that<br />

<strong>and</strong> determ<strong>in</strong>e when there is equality.<br />

(x 1 + x 2 + · · · + x n ) − (x 1 x 2 + x 2 x 3 + · · · + x n x 1 ) ≤<br />

M 22. (MM1407, M. S. Klamk<strong>in</strong>) Determ<strong>in</strong>e the maximum value <strong>of</strong> the sum<br />

[ n<br />

2<br />

]<br />

,<br />

x 1 p + x 2 p + · · · + x n p − x 1 q x 2 r − x 2 q x 3 r − · · · x n q x 1 r ,<br />

where p, q, r are given numbers with p ≥ q ≥ r ≥ 0 <strong>and</strong> 0 ≤ x i ≤ 1 for all i.<br />

M 23. (IMO Short List 1998) Let a 1 , a 2 , · · · , a n be positive real numbers such that<br />

a 1 + a 2 + · · · + a n < 1.<br />

Prove that<br />

a 1 a 2 · · · a n (1 − (a 1 + a 2 + · · · + a n ))<br />

(a 1 + a 2 + · · · + a n )(1 − a 1 )(1 − a 2 ) · · · (1 − a n ) ≤ 1<br />

n n+1 .<br />

2 Mathematics Magaz<strong>in</strong>e<br />

52


M 24. (IMO Short List 1998) Let r 1 , r 2 , · · · , r n be real numbers greater than or equal to 1. Prove that<br />

1<br />

r 1 + 1 + · · · + 1<br />

r n + 1 ≥ n<br />

(r 1 · · · r n ) 1 n + 1 .<br />

M 25. (Baltic Way 1991) Prove that, for any real numbers a 1 , · · · , a n ,<br />

∑<br />

1≤i,j≤n<br />

a i a j<br />

i + j − 1 ≥ 0.<br />

M 26. (India 1995) Let x 1 , x 2 , · · · , x n be positive real numbers whose sum is 1. Prove that<br />

x 1<br />

+ · · · + x √<br />

n n<br />

≥<br />

1 − x 1 1 − x n n − 1 .<br />

M 27. (Turkey 1997) Given an <strong>in</strong>teger n ≥ 2, F<strong>in</strong>d the m<strong>in</strong>imal value <strong>of</strong><br />

5<br />

5<br />

5<br />

x 1 x 2 x n<br />

+<br />

+ · · ·<br />

x 2 + x 3 + · · · + x n x 3 + · · · + x n + x 1 x 1 + x 3 + · · · + x n−1<br />

for positive real numbers x 1 , · · · , x n subject to the condition x 1 2 + · · · + x n 2 = 1.<br />

M 28. (Ch<strong>in</strong>a 1996) Suppose n ∈ N, x 0 = 0, x 1 , · · · , x n > 0, <strong>and</strong> x 1 + · · · + x n = 1. Prove that<br />

1 ≤<br />

n∑<br />

i=1<br />

x i<br />

√ 1 + x0 + · · · + x i−1<br />

√<br />

xi + · · · + x n<br />

< π 2<br />

M 29. (Vietnam 1998) Let x 1 , · · · , x n be positive real numbers satisfy<strong>in</strong>g<br />

Prove that<br />

1<br />

x 1 + 1998 + · · · + 1<br />

x n + 1998 = 1<br />

1998 .<br />

(x 1 · · · x n ) 1 n<br />

n − 1<br />

≥ 1998<br />

M 30. (C2768 Mohammed Aassila) Let x 1 , · · · , x n be n positive real numbers. Prove that<br />

x<br />

√ 1<br />

x1 x 2 + x + x<br />

√ 2<br />

2 2 x2 x 3 + x + · · · + x<br />

√ n<br />

2 3 xn x 1 + x ≥ √ n 2 1 2<br />

M 31. (C2842, George Ts<strong>in</strong>tsifas) Let x 1 , · · · , x n be positive real numbers. Prove that<br />

(a) x 1 n + · · · + x n<br />

n<br />

nx 1 · · · x n<br />

(b) x 1 n + · · · + x n<br />

n<br />

x 1 · · · x n<br />

+ n(x 1 · · · x n ) 1 n<br />

x 1 + · · · + x n<br />

≥ 2,<br />

+ (x 1 · · · x n ) 1 n<br />

x 1 + · · · + x n<br />

≥ 1.<br />

M 32. (C2423, Walther Janous) Let x 1 , · · · , x n (n ≥ 2) be positive real numbers such that x 1 +· · ·+x n = 1.<br />

Prove that (1 + 1 ) (<br />

· · · 1 + 1 ) ( ) ( )<br />

n − x1 n − xn<br />

≥ · · ·<br />

x 1 x n 1 − x 1 1 − x n<br />

Determ<strong>in</strong>e the cases <strong>of</strong> equality.<br />

M 33. (C1851, Walther Janous) Let x 1 , · · · , x n (n ≥ 2) be positive real numbers such that<br />

x 1 2 + · · · + x n 2 = 1.<br />

Prove that<br />

2 √ n − 1<br />

n<br />

5 √ n − 1 ≤ ∑ 2 + x i<br />

≤ 2√ n + 1<br />

5 + x i 5 √ n + 1 .<br />

i=1<br />

53


M 34. (C1429, D. S. Mitir<strong>in</strong>ovic, J. E. Pecaric) Show that<br />

n∑ x i<br />

≤ n − 1<br />

x i2 + x i+1 x i+2<br />

i=1<br />

where x 1 , · · · , x n are n ≥ 3 positive real numbers. Of course, x n+1 = x 1 , x n+2 = x 2 . 3<br />

M 35. (Belarus 1998 S. Sobolevski) Let a 1 ≤ a 2 ≤ · · · ≤ a n be positive real numbers. Prove the<br />

<strong>in</strong>equalities<br />

n<br />

(a)<br />

1<br />

a 1<br />

+ · · · + 1 ≥ a 1<br />

· a1 + · · · + a n<br />

,<br />

a n<br />

a n n<br />

n<br />

(b)<br />

1<br />

a 1<br />

+ · · · + 1 ≥ 2k<br />

a n<br />

1 + k 2 · a1 + · · · + a n<br />

,<br />

n<br />

where k = an<br />

a 1<br />

.<br />

M 36. (Hong Kong 2000) Let a 1 ≤ a 2 ≤ · · · ≤ a n be n real numbers such that<br />

Show that<br />

a 1 + a 2 + · · · + a n = 0.<br />

a 1 2 + a 2 2 + · · · + a n 2 + na 1 a n ≤ 0.<br />

M 37. (Pol<strong>and</strong> 2001) Let n ≥ 2 be an <strong>in</strong>teger. Show that<br />

n∑<br />

( n<br />

n∑<br />

x i i + ≥ ix i<br />

2)<br />

for all nonnegative reals x 1 , · · · , x n .<br />

M 38. (Korea 1997) Let a 1 , · · · , a n be positive numbers, <strong>and</strong> def<strong>in</strong>e<br />

(a) If n is even, show that<br />

(b) If n is odd, show that<br />

i=1<br />

A = a 1 + · · · + a n<br />

n<br />

i=1<br />

, G = (a 1 · · ·n) 1 n , H =<br />

n<br />

1<br />

a 1<br />

+ · · · + 1<br />

a n<br />

( ) n<br />

A<br />

A<br />

H ≤ −1 + 2 .<br />

G<br />

A<br />

H ≤ −n − 2<br />

( ) n<br />

2(n − 1) A<br />

+ .<br />

n n G<br />

M 39. (Romania 1996) Let x 1 , · · · , x n , x n+1 be positive reals such that<br />

Prove that<br />

x n+1 = x 1 + · · · + x n .<br />

n∑ √<br />

xi (x n+1 − x i ) ≤ √ x n+1 (x n+1 − x i )<br />

i=1<br />

M 40. (C2730, Peter Y. Woo) Let AM(x 1 , · · · , x n ) <strong>and</strong> GM(x 1 , · · · , x n ) denote the arithmetic mean<br />

<strong>and</strong> the geometric mean <strong>of</strong> the positive real numbers x 1 , · · · , x n respectively. Given positive real numbers<br />

a 1 , · · · , a n , b 1 , · · · , b n , (a) prove that<br />

For each real number t ≥ 0, def<strong>in</strong>e<br />

GM(a 1 + b 1 , · · · , a n + b n ) ≥ GM(a 1 , · · · , a n ) + GM(b 1 , · · · , b n ).<br />

f(t) = GM(t + b 1 , t + b 2 , · · · , t + b n ) − t<br />

(b) Prove that f is a monotonic <strong>in</strong>creas<strong>in</strong>g function, <strong>and</strong> that<br />

lim f(t) = AM(b 1, · · · , b n )<br />

t→∞<br />

3 x<br />

Orig<strong>in</strong>al version is to show that supPn<br />

i<br />

i=1 x 2 = n − 1.<br />

i +x i+1 x i+2<br />

54


M 41. (C1578, O. Johnson, C. S. Goodlad) For each fixed positive real number a n , maximize<br />

over all positive real numbers a 1 , · · · , a n−1 .<br />

M 42. (C1630, Isao Ashiba) Maximize<br />

a 1 a 2 · · · a n<br />

(1 + a 1 )(a 1 + a 2 )(a 2 + a 3 ) · · · (a n−1 + a n )<br />

a 1 a 2 + a 3 a 4 + · · · + a 2n−1 a 2n<br />

over all permutations a 1 , · · · , a 2n <strong>of</strong> the set {1, 2, · · · , 2n}<br />

M 43. (C1662, M. S. Klamk<strong>in</strong>) Prove that<br />

2r+1<br />

x 1<br />

+ x 2 2r+1<br />

2r+1 xn 4 r<br />

+ · · · ≥<br />

s − x 1 s − x 2 s − x n (n − 1)n 2r−1 (x 1x 2 + x 2 x 3 + · · · + x n x 1 ) r<br />

where n > 3, r ≥ 1 2 , x i ≥ 0 for all i, <strong>and</strong> s = x 1 + · · · + x n . Also, F<strong>in</strong>d some values <strong>of</strong> n <strong>and</strong> r such that the<br />

<strong>in</strong>equality is sharp.<br />

M 44. (C1674, M. S. Klamk<strong>in</strong>) Given positive real numbers r, s <strong>and</strong> an <strong>in</strong>teger n > r s<br />

, f<strong>in</strong>d positive real<br />

numbers x 1 , · · · , x n so as to m<strong>in</strong>imize<br />

( 1<br />

x<br />

r + 1<br />

1 x<br />

r + · · · + 1 )<br />

(1 + x<br />

2 x<br />

r<br />

1 ) s (1 + x 2 ) s · · · (1 + x n ) s .<br />

n<br />

M 45. (C1691, Walther Janous) Let n ≥ 2. Determ<strong>in</strong>e the best upper bound <strong>of</strong><br />

over all x 1 , · · · , x n ∈ [0, 1].<br />

x 1<br />

x 2 x 3 · · · x n + 1 + x 2<br />

x 1 x 3 · · · x n + 1 + · · · + x n<br />

x 1 x 2 · · · x n−1 + 1<br />

M 46. (C1892, Marc<strong>in</strong> E. Kuczma) Let n ≥ 4 be an <strong>in</strong>teger. F<strong>in</strong>d the exact upper <strong>and</strong> lower bounds for<br />

the cyclic sum<br />

n∑<br />

i=1<br />

x i<br />

x i−1 + x i + x i+1<br />

over all n-tuples <strong>of</strong> nonnegative numbers x 1 , · · · , x n such that x i−1 + x i + x i+1 > 0 for all i. Of course,<br />

x n+1 = x 1 , x 0 = x n . Characterize all cases <strong>in</strong> which either one <strong>of</strong> these bounds is atta<strong>in</strong>ed.<br />

M 47. (C1953, M. S. Klamk<strong>in</strong>) Determ<strong>in</strong>e a necessary <strong>and</strong> sucient condition on real constants r 1 , · · · , r n<br />

such that<br />

x 1 2 + x 2 2 + · + x n 2 ≥ (r 1 x 1 + r 2 x 2 + · · · + r n x n ) 2<br />

holds for all real numbers x 1 , · · · , x n .<br />

M 48. (C2018, Marc<strong>in</strong> E. Kuczma) How many permutations (x 1 , · · · , x n ) <strong>of</strong> {1, 2, · · · , n} are there such<br />

that the cyclic sum<br />

|x 1 − x 2 | + |x 2 − x 3 | + · · · + |x n−1 − x n | + |x n − x 1 |<br />

is (a) a m<strong>in</strong>imum, (b) a maximum ?<br />

M 49. (C2214, Walther Janous) Let n ≥ 2 be a natural number.<br />

C = C(n) such that for all x 1 , · · · , x n ≥ 0 we have<br />

n∑ √ ∏<br />

xi ≤ √ n (x i + C)<br />

i=1<br />

i=1<br />

Show that there exists a constant<br />

Determ<strong>in</strong>e the m<strong>in</strong>imum C(n) for some values <strong>of</strong> n. (For example, C(2) = 1.)<br />

55


M 50. (C2615, M. S. Klamk<strong>in</strong>) Suppose that x 1 , · · · , x n are non-negative numbers such that<br />

∑<br />

xi<br />

2 ∑ (x i x i+1 ) 2 =<br />

n(n + 1)<br />

2<br />

where e the sums here <strong>and</strong> subsequently are symmetric over the subscripts {1, · · · , n}. (a) Determ<strong>in</strong>e the<br />

maximum <strong>of</strong> ∑ x i . (b) Prove or disprove that the m<strong>in</strong>imum <strong>of</strong> ∑ √<br />

n(n+1)<br />

x i is<br />

2<br />

.<br />

M 51. (Turkey 1996) Given real numbers 0 = x 1 < x 2 < · · · < x 2n , x 2n+1 = 1 with x i+1 − x i ≤ h for<br />

1 ≤ i ≤ n, show that<br />

1 − h<br />

n∑<br />

< x 2i (x 2i+1 − x 2i−1 ) < 1 + h .<br />

2<br />

2<br />

i=1<br />

M 52. (Pol<strong>and</strong> 2002) Prove that for every <strong>in</strong>teger n ≥ 3 <strong>and</strong> every sequence <strong>of</strong> positive numbers x 1 , · · · , x n<br />

at least one <strong>of</strong> the two <strong>in</strong>equalities is satsified :<br />

n∑<br />

i=1<br />

x i<br />

x i+1 + x i+2<br />

≥ n 2 ,<br />

Here, x n+1 = x 1 , x n+2 = x 2 , x 0 = x n , x −1 = x n−1 .<br />

n ∑<br />

i=1<br />

x i<br />

x i−1 + x i−2<br />

≥ n 2 .<br />

M 53. (Ch<strong>in</strong>a 1997) Let x 1 , · · · , x 1997 be real numbers satisfy<strong>in</strong>g the follow<strong>in</strong>g conditions:<br />

− 1 √<br />

3<br />

≤ x 1 , · · · , x 1997 ≤ √ 3, x 1 + · · · + x 1997 = −318 √ 3<br />

Determ<strong>in</strong>e the maximum value <strong>of</strong> x 1 12 + · · · + x 1997 12 .<br />

M 54. (C2673, George Baloglou) Let n > 1 be an <strong>in</strong>teger. (a) Show that<br />

for all a 1 , · · · , a n ∈ [1, ∞) if <strong>and</strong> only if n ≥ 4.<br />

(b) Show that<br />

(1 + a 1 · · · a n ) n ≥ a 1 · · · a n (1 + a 1 n−2 ) · · · (1 + a 1 n−2 )<br />

1<br />

a 1 (1 + a<br />

n−2 2 ) + 1<br />

a 2 (1 + a<br />

n−2 3 ) + · · · + 1<br />

a n (1 + a<br />

n−2 1 ) ≥<br />

for all a 1 , · · · , a n > 0 if <strong>and</strong> only if n ≤ 3.<br />

(c) Show that<br />

1<br />

a 1 (1 + a<br />

n−2 1 ) + 1<br />

a 2 (1 + a<br />

n−2 2 ) + · · · + 1<br />

a n (1 + a<br />

n−2 n ) ≥<br />

for all a 1 , · · · , a n > 0 if <strong>and</strong> only if n ≤ 8.<br />

n<br />

1 + a 1 · · · a n<br />

n<br />

1 + a 1 · · · a n<br />

M 55. (C2557, Gord S<strong>in</strong>namon,Hans He<strong>in</strong>ig) (a) Show that for all positive sequences {x i }<br />

n∑<br />

⎛<br />

k∑ j∑<br />

n∑ k∑<br />

x i ≤ 2 ⎝<br />

k=1 j=1 i=1<br />

k=1<br />

j=1<br />

⎞2<br />

x j<br />

⎠<br />

(b) Does the above <strong>in</strong>equality rema<strong>in</strong> true without the factor 2? (c) What is the m<strong>in</strong>imum constant c that<br />

can replace the factor 2 <strong>in</strong> the above <strong>in</strong>equality?<br />

M 56. (C1472, Walther Janous) For each <strong>in</strong>teger n ≥ 2, F<strong>in</strong>d the largest constant C n such that<br />

∑<br />

n<br />

C n<br />

i=1<br />

|a i | ≤ ∑<br />

for all real numbers a 1 , · · · , a n satisfy<strong>in</strong>g ∑ n<br />

i=1 a i = 0.<br />

1≤i


M 57. (Ch<strong>in</strong>a 2002) Given c ∈ ( 1<br />

2 , 1) . F<strong>in</strong>d the smallest constant M such that, for any <strong>in</strong>teger n ≥ 2 <strong>and</strong><br />

real numbers 1 < a 1 ≤ a 2 ≤ · · · ≤ a n , if<br />

then<br />

1<br />

n<br />

k=1<br />

n∑<br />

n∑<br />

ka k ≤ c a k ,<br />

k=1<br />

where m is the largest <strong>in</strong>teger not greater than cn.<br />

k=1<br />

n∑<br />

m∑<br />

a k ≤ M ka k ,<br />

M 58. (Serbia 1998) Let x 1 , x 2 , · · · , x n be positive numbers such that<br />

Prove the <strong>in</strong>equality<br />

k=1<br />

x 1 + x 2 + · · · + x n = 1.<br />

a x1−x2<br />

x 1 + x 2<br />

+ ax2−x3<br />

x 2 + x 3<br />

+ · · · axn−x1<br />

x n + x 1<br />

≥ n2<br />

2 ,<br />

holds true for every positive real number a. Determ<strong>in</strong>e also when the equality holds.<br />

M 59. (MM1488, He<strong>in</strong>z-Jurgen Seiffert) Let n be a positive <strong>in</strong>teger. Show that if 0 < x 1 ≤ x 2 ≤ x n ,<br />

then<br />

⎛ ⎞<br />

n∏<br />

n∑ j∏<br />

(1 + x i ) ⎝<br />

1<br />

⎠ ≥ 2 n (n + 1)<br />

x k<br />

i=1<br />

with equality if <strong>and</strong> only if x 1 = · · · = x n = 1.<br />

j=0 k=1<br />

M 60. (Len<strong>in</strong>grad Mathematical Olympiads 1968) Let a 1 , a 2 , · · · , a p be real numbers. Let M = max S<br />

<strong>and</strong> m = m<strong>in</strong> S. Show that<br />

(p − 1)(M − m) ≤ ∑<br />

1≤i,j≤n<br />

|a i − a j | ≤ p2<br />

(M − m)<br />

4<br />

M 61. (Len<strong>in</strong>grad Mathematical Olympiads 1973) Establish the follow<strong>in</strong>g <strong>in</strong>equality<br />

8∑<br />

i=0<br />

( π<br />

) ( ( π<br />

))<br />

2 i cos<br />

2 i+2 1 − cos<br />

2 i+2 < 1 2 .<br />

M 62. (Len<strong>in</strong>grad Mathematical Olympiads 2000) Show that, for all 0 < x 1 ≤ x 2 ≤ . . . ≤ x n ,<br />

x 1 x 2<br />

+ x 2x 3<br />

+ · · · + x n 1<br />

x 1<br />

+ x nx 1<br />

≥<br />

x 3 x 4 x 2 x 2<br />

M 63. (Mongolia 1996) Show that, for all 0 < a 1 ≤ a 2 ≤ . . . ≤ a n ,<br />

( ) ( ) ( ) ( ) ( )<br />

a1 + a 2 a2 + a 3 an + a 1 a1 + a 2 + a 3 a2 + a 3 + a 4<br />

· · ·<br />

≤<br />

· · ·<br />

2 2<br />

2<br />

3<br />

3<br />

n∑<br />

i=1<br />

x i<br />

(<br />

an + a 1 + a 2<br />

3<br />

)<br />

.<br />

57


5.2 Problems for Putnam Sem<strong>in</strong>ar<br />

P 1. Putnam 04A6 Suppose that f(x, y) is a cont<strong>in</strong>uous real-valued function on the unit square 0 ≤ x ≤<br />

1, 0 ≤ y ≤ 1. Show that<br />

∫ 1<br />

0<br />

(∫ 1<br />

2<br />

f(x, y)dx)<br />

dy +<br />

(∫ 1<br />

≤<br />

0<br />

∫ 1<br />

∫ 1<br />

f(x, y)dx dy) 2<br />

+<br />

(∫ 1<br />

2<br />

f(x, y)dy)<br />

dx<br />

0 0<br />

∫ 1 ∫ 1<br />

0 0<br />

0 0<br />

(f(x, y)) 2 dx dy.<br />

P 2.<br />

Putnam 04B2 Let m <strong>and</strong> n be positive <strong>in</strong>tegers. Show that<br />

(m + n)! m! n!<br />

<<br />

(m + n)<br />

m+n<br />

m m n n .<br />

P 3.<br />

Putnam 03A2 Let a 1 , a 2 , . . . , a n <strong>and</strong> b 1 , b 2 , . . . , b n be nonnegative real numbers. Show that<br />

(a 1 a 2 · · · a n ) 1/n + (b 1 b 2 · · · b n ) 1/n ≤ [(a 1 + b 1 )(a 2 + b 2 ) · · · (a n + b n )] 1/n .<br />

P 4.<br />

Putnam 03A3 F<strong>in</strong>d the m<strong>in</strong>imum value <strong>of</strong><br />

| s<strong>in</strong> x + cos x + tan x + cot x + sec x + csc x|<br />

for real numbers x.<br />

P 5.<br />

Putnam 03A4 Suppose that a, b, c, A, B, C are real numbers, a ≠ 0 <strong>and</strong> A ≠ 0, such that<br />

|ax 2 + bx + c| ≤ |Ax 2 + Bx + C|<br />

for all real numbers x. Show that<br />

|b 2 − 4ac| ≤ |B 2 − 4AC|.<br />

P 6. Putnam 03B6 Let f(x) be a cont<strong>in</strong>uous real-valued function def<strong>in</strong>ed on the <strong>in</strong>terval [0, 1]. Show<br />

that ∫ 1 ∫ 1<br />

∫ 1<br />

|f(x) + f(y)| dx dy ≥ |f(x)| dx.<br />

P 7. Putnam 02B3 Show that, for all <strong>in</strong>tegers n > 1,<br />

0<br />

0<br />

1<br />

2ne < 1 (1<br />

e − − 1 ) n<br />

< 1 n ne .<br />

P 8. Putnam 01A6 Can an arc <strong>of</strong> a parabola <strong>in</strong>side a circle <strong>of</strong> radius 1 have a length greater than 4?<br />

P 9. Putnam 99A5 Prove that there is a constant C such that, if p(x) is a polynomial <strong>of</strong> degree 1999,<br />

then<br />

∫ 1<br />

|p(0)| ≤ C |p(x)| dx.<br />

−1<br />

P 10. Putnam 99B4 Let f be a real function with a cont<strong>in</strong>uous third derivative such that f(x), f ′ (x), f ′′ (x), f ′′′ (x)<br />

are positive for all x. Suppose that f ′′′ (x) ≤ f(x) for all x. Show that f ′ (x) < 2f(x) for all x.<br />

P 11. Putnam 98B4 Let a m,n denote the coefficient <strong>of</strong> x n <strong>in</strong> the expansion <strong>of</strong> (1 + x + x 2 ) m . Prove that<br />

for all <strong>in</strong>tegers k ≥ 0,<br />

⌊ 2k 3 ⌋<br />

∑<br />

0 ≤ (−1) i a k−i,i ≤ 1.<br />

i=0<br />

0<br />

58


P 12.<br />

Putnam 98B1 F<strong>in</strong>d the m<strong>in</strong>imum value <strong>of</strong><br />

for x > 0.<br />

(<br />

x +<br />

1 6 ( )<br />

x)<br />

− x 6 + 1 x − 2<br />

( ) 6<br />

x +<br />

1 3 ( )<br />

x + x3 + 1 x 3<br />

P 13. Putnam 96B2 Show that for every positive <strong>in</strong>teger n,<br />

( 2n − 1<br />

e<br />

) 2n−1<br />

2<br />

( 2n + 1<br />

< 1 · 3 · 5 · · · (2n − 1) <<br />

e<br />

) 2n+1<br />

2<br />

.<br />

P 14. Putnam 96B3 Given that {x 1 , x 2 , . . . , x n } = {1, 2, . . . , n}, f<strong>in</strong>d, with pro<strong>of</strong>, the largest possible<br />

value, as a function <strong>of</strong> n (with n ≥ 2), <strong>of</strong><br />

x 1 x 2 + x 2 x 3 + · · · + x n−1 x n + x n x 1 .<br />

P 15. Putnam 91B6 Let a <strong>and</strong> b be positive numbers. F<strong>in</strong>d the largest number c, <strong>in</strong> terms <strong>of</strong> a <strong>and</strong> b,<br />

such that<br />

a x b 1−x s<strong>in</strong>h ux u(1 − x)<br />

≤ a + bs<strong>in</strong>h<br />

s<strong>in</strong>h u s<strong>in</strong>h u<br />

for all u with 0 < |u| ≤ c <strong>and</strong> for all x, 0 < x < 1.<br />

P 16. (CMJ 4 416, Joanne Harris) For what real values <strong>of</strong> c is<br />

for all real x?<br />

e x + e −x<br />

2<br />

≤ e cx2 .<br />

P 17. (CMJ420, Edward T. H. Wang) It is known [Daniel I. A. Cohen, Basic <strong>Techniques</strong> <strong>of</strong> Comb<strong>in</strong>atorial<br />

Theory, p.56] <strong>and</strong> easy to show that 2 n < ( )<br />

2n<br />

n < 2 2n for all <strong>in</strong>tegers n > 1. Prove that the stronger<br />

<strong>in</strong>equalities<br />

2 2n−1 ( ) 2n<br />

√ < < 22n<br />

√ n n n<br />

hold for all n ≥ 4.<br />

P 18. (CMJ379, Mohammad K. Azarian) Let x be any real number. Prove that<br />

n∑<br />

n∑<br />

(1 − cos x)<br />

s<strong>in</strong>(kx)<br />

cos(kx)<br />

∣ ∣ ∣ ∣ ≤ 2.<br />

P 19. (CMJ392 Robert Jones) Prove that<br />

(1 + 1 ) (<br />

x 2 x s<strong>in</strong> 1 )<br />

> 1 for x ≥ √ 1 .<br />

x<br />

5<br />

k=1<br />

k=1<br />

P 20. (CMJ431 R. S. Luthar) Let 0 < φ < θ < π 2<br />

. Prove that<br />

[(1 + tan 2 φ)(1 + s<strong>in</strong> 2 φ)] csc2 φ < [(1 + tan 2 θ)(1 + s<strong>in</strong> 2 θ)] csc2 θ .<br />

P 21. (CMJ451, Mohammad K. Azarian) Prove that<br />

provided 0 < α < π 2 .<br />

4 The College Mathematics Journal<br />

π sec2 α cos 2 α + π csc2 α s<strong>in</strong> 2 α ≥ π 2 ,<br />

59


P 22. (CMJ446, Norman Schaumberger) If x, y, <strong>and</strong> z are the radian measures <strong>of</strong> the angles <strong>in</strong> a<br />

(non-degenerate) triangle, prove that<br />

π s<strong>in</strong> 3 π ≥ x s<strong>in</strong> 1 x + y s<strong>in</strong> 1 y + z s<strong>in</strong> 1 z .<br />

P 23. (CMJ461, Alex Necochea) Let 0 < x < π 2<br />

<strong>and</strong> 0 < y < 1. Prove that<br />

√<br />

1 − y2 − cos x<br />

x − arcs<strong>in</strong> y ≤<br />

,<br />

y<br />

with equality hold<strong>in</strong>g if <strong>and</strong> only if y = s<strong>in</strong> x.<br />

P 24. (CMJ485 Norman Schaumberger) Prove that<br />

(1) if a ≥ b > 1 or 1 > a ≥ b > 0, then a bb b aa ≥ a ba b ab ; <strong>and</strong><br />

(2) if a > 1 > b > 0, then a bb b aa ≤ a ba b ab .<br />

P 25. (CMJ524 Norman Schaumberger) Let a, b, <strong>and</strong> c be positive real numbers. Show that<br />

( ) a ( ) b ( ) c a + b b + c c + a<br />

a a b b c c ≥<br />

≥ b a c b a c .<br />

2 2 2<br />

P 26. (CMJ567 H.-J. Seiffert) Show that for all dit<strong>in</strong>ct positive real numbers x <strong>and</strong> y,<br />

(√ x +<br />

√ y<br />

2<br />

) 2<br />

< x − y<br />

2 s<strong>in</strong>h x−y<br />

x+y<br />

< x + y .<br />

2<br />

P 27. (CMJ572, George Baloglou <strong>and</strong> Robert Underwood) Prove or disprove that for θ ∈ ( − π 4 , ) π<br />

4 ,<br />

1<br />

cosh θ ≤ √ .<br />

1−tan 2 θ<br />

P 28. (CMJ603, Juan-Bosco Romero Marquez) Let a <strong>and</strong> b be dist<strong>in</strong>ct positive real numbers <strong>and</strong> let<br />

n be a positive <strong>in</strong>teger. Prove that<br />

√<br />

√<br />

a + b b<br />

≤ n+1 − a n+1<br />

n<br />

2 (n + 1)(b − a) ≤ n an + b n<br />

.<br />

2<br />

P 29. (MM 5 904, Norman Schaumberger) For x > 2, prove that<br />

( ) x<br />

ln ≤<br />

x − 1<br />

∞∑<br />

j=0<br />

( )<br />

1 x − 1<br />

x 2j ≤ ln .<br />

x − 2<br />

P 30. (MM1590, Constant<strong>in</strong> P. Niculescu) For given a, 0 < a < π 2<br />

, determ<strong>in</strong>e the m<strong>in</strong>imum value <strong>of</strong><br />

α ≥ 0 <strong>and</strong> the maximum value <strong>of</strong> β ≥ 0 for which<br />

( x<br />

a<br />

) α s<strong>in</strong> x<br />

( x<br />

) β<br />

≤<br />

s<strong>in</strong> a ≤ .<br />

a<br />

(This generalize the well-known <strong>in</strong>equality due to Jordan, which asserts that 2x π ≤ s<strong>in</strong> x ≤ 1 on [0, π 2 ].)<br />

P 31. (MM1597, Constant<strong>in</strong> P. Niculescu) For every x, y ∈ ( 0, √ π<br />

2<br />

)<br />

with x ≠ y, prove that<br />

(<br />

ln<br />

) 2<br />

1 − s<strong>in</strong> xy<br />

≥ ln<br />

1 + s<strong>in</strong> xy<br />

1 − s<strong>in</strong> x2 1 − s<strong>in</strong> y2<br />

ln<br />

1 + s<strong>in</strong> x2 1 + s<strong>in</strong> y 2 .<br />

P 32. (MM1599, Ice B. Risteski) Given α > β > 0 <strong>and</strong> f(x) = x α (1 − x) β . If 0 < a < b < 1 <strong>and</strong><br />

f(a) = f(b), show that f ′ (α) < −f ′ (β).<br />

5 Mathematics Magaz<strong>in</strong>e<br />

60


P 33. (MM Q197, Norman Schaumberger) Prove that if b > a > 0, then ( )<br />

a a<br />

b ≥<br />

e a<br />

≥ ( )<br />

a b.<br />

e b b<br />

P 34. (MM1618, Michael Golomb) Prove that 0 < x < π,<br />

x π − x (3<br />

π + x < s<strong>in</strong> x < − x )<br />

x π − x<br />

π π + x .<br />

P 35. (MM1634, Constant<strong>in</strong> P. Niculescu) F<strong>in</strong>d the smallest constant k > 0 such that<br />

for every a, b, c > 0.<br />

ab<br />

a + b + 2c +<br />

bc<br />

b + c + 2a +<br />

ca<br />

≤ k(a + b + c)<br />

c + a + 2b<br />

P 36. (MM1233, Robert E. Shafer) Prove that if x > −1 <strong>and</strong> x ≠ 0, then<br />

1 + x + x2<br />

x 2<br />

2 − 120<br />

1+x+ 31<br />

x 4<br />

< [ln(1 + x)] 2 <<br />

1 + x + x2<br />

252 x2<br />

P 37. (MM1236, Mihaly Bencze) Let the functions f <strong>and</strong> g be def<strong>in</strong>ed by<br />

f(x) =<br />

x 2<br />

π 2 x<br />

2π 2 + 8x 2 <strong>and</strong> g(x) = 8x<br />

4π 2 + πx 2<br />

.<br />

2 − x 4<br />

240<br />

1+x+ 1 20 x2<br />

for all real x. Prove that if A, B, <strong>and</strong> C are the angles <strong>of</strong> an acuted-angle triangle, <strong>and</strong> R is its circumradius<br />

then<br />

f(A) + f(B) + f(C) < a + b + c < g(A) + g(B) + g(C).<br />

4R<br />

P 38. (MM1245, Fouad Nakhli) For each number x <strong>in</strong> open <strong>in</strong>terval (1, e) it is easy to show that there<br />

is a unique number y <strong>in</strong> (e, ∞) such that ln y<br />

y<br />

= ln x<br />

x<br />

. For such an x <strong>and</strong> y, show that x + y > x ln y + y ln x.<br />

P 39. (MM Q725, S. Kung) Show that (s<strong>in</strong> x)y ≤ s<strong>in</strong>(xy), where 0 < x < π <strong>and</strong> 0 < y < 1.<br />

P 40. (MM Q771, Norman Schaumberger) Show that if 0 < θ < π 2 , then s<strong>in</strong> 2θ ≥ (tan θ)cos 2θ .<br />

61


References<br />

AB K. S. Kedlaya, A < B, http://www.unl.edu/amc/a-activities/a4-for-students/s-<strong>in</strong>dex.html<br />

AI D. S. Mit<strong>in</strong>ović (<strong>in</strong> cooperation with P. M. Vasić), Analytic <strong>Inequalities</strong>, Spr<strong>in</strong>ger<br />

AK F. F. Abi-Khuzam, A Trigonometric Inequality <strong>and</strong> its Geometric Applications, Mathematical <strong>Inequalities</strong><br />

<strong>and</strong> Applications, Vol. 3, No. 3 (2000), 437-442<br />

AMN A. M. Nesbitt, Problem 15114, Educational Times (2) 3(1903), 37-38<br />

AP A. Padoa, Period. Mat. (4)5 (1925), 80-85<br />

Au99 A. Storozhev, AMOC Mathematics Contests 1999, Australian Mathematics Trust<br />

DP D. Pedoe, Th<strong>in</strong>k<strong>in</strong>g Geometrically, Amer. Math. Monthly 77(1970), 711-721<br />

EC E. Cesáro, Nouvelle Correspondence Math. 6(1880), 140<br />

ESF Elementare Symmetrische Funktionen,<br />

http://hydra.nat.uni-magdeburg.de/math4u/var/PU4.html<br />

EWW-KI Eric W. Weisste<strong>in</strong>. ”Kantorovich Inequality.” From MathWorld–A Wolfram Web Resource.<br />

http://mathworld.wolfram.com/KantorovichInequality.html<br />

EWW-AI Eric W. Weisste<strong>in</strong>. ”Abel’s Inequality.” From MathWorld–A Wolfram Web Resource.<br />

http://mathworld.wolfram.com/AbelsInequality.html<br />

GC G. Chang, Prov<strong>in</strong>g Pedoe’s Inequality by Complex Number Computation, Amer. Math. Monthly<br />

89(1982), 692<br />

GI O. Bottema, R. ˜Z. Djordjević, R. R. Janić, D. S. Mitr<strong>in</strong>ović, P. M. Vasić, Geometric <strong>Inequalities</strong>,<br />

Wolters-Noordh<strong>of</strong>f Publish<strong>in</strong>g, Gron<strong>in</strong>gen 1969<br />

HFS H. F. S<strong>and</strong>ham, Problem E819, Amer. Math. Monthly 55(1948), 317<br />

IN I. Niven, Maxima <strong>and</strong> M<strong>in</strong>ima Without Calculus, MAA<br />

IV Ilan Vardi, Solutions to the year 2000 International Mathematical Olympiad<br />

http://www.lix.polytechnique.fr/Labo/Ilan.Vardi/publications.html<br />

JC Ji Chen, Problem 1663, Crux Mathematicorum 18(1992), 188-189<br />

JfdWm J. F. Darl<strong>in</strong>g, W. Moser, Problem E1456, Amer. Math. Monthly 68(1961) 294, 230<br />

JmhMh J. M. Habeb, M. Hajja, A Note on Trigonometric Identities, Expositiones Mathematicae 21(2003),<br />

285-290<br />

KBS K. B. Stolarsky, Cubic Triangle <strong>Inequalities</strong>, Amer. Math. Monthly (1971), 879-881<br />

KYL K<strong>in</strong> Y. Li, Majorization Inequality, Mathematical Excalibur, (5)5 (2000), 2-4<br />

KWL Kee-Wai Liu, Problem 2186, Crux Mathematicorum with Mathematical Mayhem, 23(1997), 71-72<br />

LC1 L. Carlitz, An Inequality Involv<strong>in</strong>g the Area <strong>of</strong> Two Triangles, Amer. Math. Monthly 78(1971), 772<br />

LC2 L. Carlitz, Some <strong>Inequalities</strong> for Two Triangles, Amer. Math. Monthly 80(1973), 910<br />

MB L. J. Mordell, D. F. Barrow, Problem 3740, Amer. Math. Monthly 44(1937), 252-254<br />

MC M. Cipu, Problem 2172, Crux Mathematicorum with Mathematical Mayhem, 23(1997), 439-440<br />

MCo M. Col<strong>in</strong>d, Educational Times 13(1870), 30-31<br />

62


MEK Marc<strong>in</strong> E. Kuczma, Problem 1940, Crux Mathematicorum with Mathematical Mayhem, 23(1997),<br />

170-171<br />

MP M. Petrović, Ra˜cunanje sa brojnim razmacima, Beograd 1932, 79<br />

MEK2 Marc<strong>in</strong> E. Kuczma, Problem 1703, Crux Mathematicorum 18(1992), 313-314<br />

NC A note on convexity, Crux Mathematicorum with Mathematical Mayhem, 23(1997), 482-483<br />

ONI T. Andreescu, V. Cirtoaje, G. Dosp<strong>in</strong>escu, M. Lascu, Old <strong>and</strong> New <strong>Inequalities</strong><br />

PF P. Flor, Über e<strong>in</strong>e Ungleichung von S. S. Wagner, Elem. Math. 20, 136(1965)<br />

RAS R. A. Satnoianu, A General Method for Establish<strong>in</strong>g Geometric <strong>Inequalities</strong> <strong>in</strong> a Triangle, Amer. Math.<br />

Monthly 108(2001), 360-364<br />

RI K. Wu, Andy Liu, The Rearrangement Inequality, ??<br />

RS R. Sondat, Nouv. Ann. Math. (3) 10(1891), 43-47<br />

SR S. Rab<strong>in</strong>owitz, On The Computer Solution <strong>of</strong> Symmetric Homogeneous Triangle <strong>Inequalities</strong>, Proceed<strong>in</strong>gs<br />

<strong>of</strong> the ACM-SIGSAM 1989 International Symposium on Symbolic <strong>and</strong> Algebraic Computation<br />

(ISAAC 89), 272-286<br />

SR2 S. Reich, Problem E1930, Amer. Math. Monthly 73(1966), 1017-1018<br />

TD Titu Andreescu, Dor<strong>in</strong> Andrica, Complex Numbers from A to ... Z, Birkhauser<br />

TF G. B. Thomas, Jr., Ross L. F<strong>in</strong>ney Calculus <strong>and</strong> Analytic Geometry 9th ed, Addison-Wesley Publish<strong>in</strong>g<br />

Company<br />

TJM T. J. Mildorf, Olympiad <strong>Inequalities</strong>, http://web.mit.edu/tmildorf/www/<br />

TZ T. Andreescu, Z. Feng, 103 Trigonometry Problems From the Tra<strong>in</strong><strong>in</strong>g <strong>of</strong> the USA IMO Team, Birkhauser<br />

WJB W. J. Blundon, Canad. Math. Bull. 8(1965), 615-626<br />

WJB2 W. J. Blundon, Problem E1935, Amer. Math. Monthly 73(1966), 1122<br />

WR Walter Rud<strong>in</strong>, Pr<strong>in</strong>ciples <strong>of</strong> Mathematical Analysis, 3rd ed, McGraw-Hill Book Company<br />

ZsJc Zun Shan, Ji Chen, Problem 1680, Crux Mathematicorum 18(1992), 251<br />

63

Hooray! Your file is uploaded and ready to be published.

Saved successfully!

Ooh no, something went wrong!